Vous êtes sur la page 1sur 506

Extensor Tendon Compartments

Author: Derek Moore


Topic updated on 11/08/14 10:54am

Extensor Pollicis Brevis.

Abductor Pollicis Longus.

Extensor Carpi Radialis Longus.

Extensor Carpi Radialis Brevis.

Extensor Pollicis Longus

Extensor Indicis

Extensor Digitorum

Extensor Digiti Minimi

Extensor Carpi Ulnaris

Extensor Ligaments
Lumbrical tendon passes volar to transverse metacarpal ligament
Interossei tendons pass dorsal to transverse metacarpal ligament
Retinacular Ligaments
Function
o retain and position common extensor mechanism during PIP and DIP flexion
o similar to sagittal band function
Anatomic Components
o oblique band (oblique retinacular ligament of Landsmeer)
function
links motion of DIP and PIP
anatomy
origin: from lateral volar aspect of proximal phalanx,
insertion: to lateral terminal extensor dorsally (crosses collateral
ligaments)
biomechanics
with PIP flexion, ligament relaxes to allow DIP flexion
with PIP extension, ligament tights to facilitate DIP extension
pathology
contracture causes volar displacement of lateral bands and a
resulting Boutonniere Deformity
o transverse band
function
with PIP flexion, pull lateral bands volarly over PIP
with PIP extension, prevents excessive dorsal translation of lateral
bands
anatomy
origin: from edge of flexor tendon sheath at PIP
insertion: lateral border of conjointed lateral bands
pathology
attenuation leads to dorsal translation of lateral bands and a
resulting Swan Neck Deformity
contracture (with attenuation of triangular ligament) leads to volar
translation of lateral bands and resulting boutonniere deformity
Digital Cutaneous Ligaments
Function
o tether skin to deeper layers of fascia and bone to prevent excessive mobility of
skin and improve grip
o stabilize the digital neurovascular bundle with finger flexion and extension
Anatomic Components
o Cleland's ligaments (remember "C" for ceiling)
dorsal to digital nerves
not involved in Dupuytren's disease
o Grayson's ligament (remember "G" for ground)
volar to digital nerves
Expansion Hood

Function
o works to extend PIP and DIP joint
Anatomic Components
o central slip
functions to extend PIP
inserts into base of middle phalanx
o lateral band
functions to extend DIP
inserts into distal phalanx
lumbricals, extensor indicis, dorsal and palmar interossei insert on
lateral band
MCP Joint Collateral Ligaments
Function
o stabilize MCP joint during motion
MCP joint "cam" nature leads to inconstant arc of motion because of joint
asymmetry
caused by "snoopy head" configuration of metacarpal head
collaterals looser in extension, tighten during increasing flexion
as MP joint flexes, proximal phalanx moves further away from metacarpal
head, tightening all the ligaments
Anatomic Components
o radial collateral ligaments (RCL) are more horizontal than ulnar collateral ligaments
(UCL)
o RCL and UCL have 2 parts each: proper and accessory ligaments
accessory ligament
fan shaped
more volar
tight in extension
attachment
from metacarpal head at center of rotation
to palmar plate and deep transverse metacarpal ligament
clinical test
adduction/abduction stress in extension
proper ligament
cord like
more dorsal
tight in 30 degrees of flexion
attachment
from posterior tubercle of metacarpal head (dorsal to mid axis)
to proximal phalanx base
clinical test
adduction/abduction stress in 30 degrees flexion to isolate proper

ligaments
Deep Transverse Metacarpal Ligament
Function
o prevents metacarpal heads from splaying apart (abduction)
o allows some dorsal-volar translation
Anatomic components
o connects 2nd to 5th metacarpal heads together at volar plate of the MP joint
Natatory Ligament (Superficial Transverse Metacarpal Ligament)
Function
o resists abduction
Anatomic components
o most superficial MP joint ligament
o origin: from distal to the MP joint
o insertion: proximal phalanx of all 5 fingers (runs in the web space)
Sagittal Bands
Function
o keep extensor mechanism tracking in the midline during flexion of MP joint
Anatomy
o origin: palmar plate
o insertion: extensor mechanism (curves around radial and ulnar side of MP joint)
Triangular ligament
Function
o counteracts pull of oblique retinacular ligament, preventing lateral subluxation of the
common extensor mechanism
Anatomy
o triangular in shape
o located on dorsal side of extensor mechanism, distal to PIP joint
Volar Plate
Function
o prevent hyperextension
Anatomy
o thickening of joint capsule volar to the MP joint
o in the thumb, sesamoid bones are located here
o origin: metacarpal head
o insertion: periarticular surface of proximal phalanx , via checkrein ligaments
Biomechanics
o loose in flexion
folds into metacarpal neck during flexion

tight in extension

Extensor Carpi Ulnaris

Ligaments of the Fingers


Author: Colin Woon
Topic updated on 09/01/14 8:53pm

Flexor Pulley System

Annular ligaments
o A2 and A4 are critical to prevent bowstringing
most biomechanically important
o A1, A3, and A5 overlie the MP, PIP and DIP joints respectively
originate from palmar plate
o A1 pulley most commonly involved in trigger finger
Cruciate pulleys
o function to prevent sheath collapse and expansion during
digital motion
o 3 total at the level of the joints

Oblique pulley
o originates at proximal half of proximal phalanx
o most important pulley in thumb
o facilitates full excursion of flexor pollicis longus
o prevents bowstringing of flexor pollicis longus
Annular pulleys

A1 pulley
at the level of the volar plate at the MCP joint
~6mm in length
A2 pulley
contributes least to arc of motion of thumb

Question:

1. OBQ08.274) Which of the following flexor tendon annular pulleys originate from palmar
plates overlying joints?
The pulley system governs the moment arm, excursion and joint rotation produced by the
flexor tendons. The A2 and A4 pulleys are the most biomechanically important to these
functions. A2 and A4 arise from the periosteum of the proximal half of the proximal
phalanx, and the midportion of the middle phalanx, respectively. A1, A3 and A5 are joint
pulleys arising from the palmar plates of the MP, PIP, and DIP joints respectively. C1,
C2, and C3 are thin, condensable, cruciate sections of the flexor sheath which permit the
annular pulleys to approximate each other during flexion.

Blood Supply to Hand


Author: Colin Woon

Source Arteries
median artery (occasionally)
Radial artery
o runs between brachioradialis and FCR
o enters the dorsum of the carpus by passing between FCR and APL/EPB
tendons (in the snuffbox)
o gives off superficial palmar branch (communicates with superficial arch)
o finally passes between 2 heads of 1st dorsal interosseous to form the deep
palmar arch
Ulnar artery
o runs under flexor carpi ulnaris
o lateral to ulnar nerve at the wrist
o enters the hand through Guyon's canal
o lies on the transverse carpal ligament
Supplemental arteries
o anterior interosseous artery
o posterior interosseous artery

Superficial Arch
o
o
o

deep to palmar fascia


distal to the deep arch
surface marking

at the level of a line drawn across the palm parallel to the distal
edge of the fully abducted thumb
Blood supply
o predominant supply is ulnar artery
o minor supply from superficial branch of radial artery

Branches of superficial arch (from ulnar to radial)


o 1st branch
is the deep branch that provides the minor supply to the deep
palmar arch
o 2nd branch
is the ulnar digital artery of the little finger
the proper digital artery to the ulnar side of the little finger
arises directly from the superficial arch
o 3rd, 4th, 5th, and 6th branches
are the common palmar digital arteries
in the palm, the digital arteries are volar to the digital nerves
in the digits, the digital arteries are dorsal to the digital
nerves
in the digits, the neurovascular bundle is volar to
Cleland's ligament
o multiple branches to intrinsic muscles and skin

The superficial arch is complete (branches to all digits) in 80% of individuals

DEEP ARCH

deep to the flexor tendons (FDS, FDP)


o proximal to the superficial arch
o at the level of the base of the metacarpals
o surface marking
1 fingerbreadth proximal to a line drawn across the palm
parallel to the distal edge of the fully abducted thumb
1 fingerbreadth proximal to the superficial arch
Blood supply
o predominant supply is the deep branch of theradial artery
o minor supply from the deep branch of the ulnar artery
Branches of the deep arch (from radial to ulnar)
o princeps pollicis
runs between 1st dorsal interosseus and adductor pollicis
o branch to the radial side of the index finger
the proper digital artery to the radial side of the IF arises
directly from the deep arch
o branches to the 3 common digital arteries in the 2nd, 3rd, and
4th web spaces
The deep arch is complete (branches to all digits) in 97% of
individuals
o

DIGITAL ARTERIES

Common digital arteries arise from the superficial palmar arch


Divide into proper digital arteries at the web spaces
Gives dorsal branches distal to the PIP joints
Dominant arteries are found on the median side of the digit (closer to midline)
o in the index finger, the ulnar digital artery is dominant
o in the little finger, the radial digital artery is dominant
in the middle and ring fingers, ulnar and radial digital arteries are
dominant respectively, but dominance is less obvious
DORSAL ARTERIES
Blood supply
o posterior interosseous artery
o dorsal perforating branch of anterior interosseous artery
Form a dorsal carpal arch which gives rise to dorsal metacarpal arteries
o useful for dorsal metacarpal artery flaps
o 1st and 2nd dorsal metacarpal artery are more consistent than 3rd and
4th
VEINS
Deep veins
o veins follow the deep arterial system as venae comitantes
Superficial veins
o found at the hand dorsum
o contribute to the basilic and cephalic vein system

Wrist Ligaments & Biomechanics


Wrist Planes of Motion
Joints involved
o radiocarpal
o intercarpal
Three axes of motion
o flexion-extension
o radial-ulnar deviation
o prono-supination
Normal and function motion
o flexion (65 normal, 10 functional)
40% radiocarpal, 60% midcarpal
o extension (55 normal, 35 functional)
66% radiocarpal, 33% midcarpal
o radial deviation (15 normal, 10 functional)
90% midcarpal

ulnar deviation (35 normal, 15 functional)


50% radiocarpal, 50% midcarpal

Wrist Biomechanics
Three biomechanic concepts have been proposed:
Link concept

three links in a chain composed of radius, lunate and capitate


head of capitate acts as center of rotation
proximal row (lunate) acts as a unit and is an intercalated
segment with no direct tendon attachments
distal row functions as unit

advantage
efficient motion (less motion at each link)
strong volar ligaments enhance stability
o disadvantage
more links increases instability of the chain
scaphoid bridges both carpal rows
resting forces/radial deviation push the scaphoid into
flexion and push the triquetrum into extension
ulnar deviation pushes the scaphoid into extension
Column concept
o

Lateral (mobile) column


comprises scaphoid, trapezoid and trapezium
scaphoid is center of motion and function is mobile

central (flexion-extension) column


comprises lunate, capitate and hamate
luno-capitate articulation is center of motion
motion is flexion/extension
o medial (rotation) column
comprises triquetrum and distal carpal row
motion is rotation
Rows concept
o comprises proximal and distal rows
scaphoid is a bridge between rows
o motion occurs within and between rows
o

Carpal Relationships
Carpal collapse
o normal ratio of carpal height to 3rd metacarpal height is 0.54
Ulnar translation
o normal ratio of ulna-to-capitate length to 3rd metacarpal height is 0.30
Load transfer
o distal radius bears 80% of load
o distal ulna bears 20% of load
ulna load bearing increases with ulnar lengthening
ulna load bearing decreases with ulnar shortening
Wrist Ligaments
The ligaments of the wrist include
o extrinsic ligaments
bridge carpal bones to the radius or metacarpals
include volar and dorsal ligaments
o intrinsic ligaments
originate and insert on carpal bones
the most important intrinsic ligaments are the scapholunate
interosseous ligament andlunotriquetral interosseous ligament
Characteristics
o volar ligaments are secondary stabilizers of the scaphoid
o volar ligaments are stronger than dorsal ligaments
o dorsal ligaments converge on the triquetrum
Space of Poirier
o center of a double "V" shape convergence of ligaments
o central weak area of the wrist in the floor of the carpal tunnel at the
level of the proximal capitate
o between the volar radioscaphocapitate ligament and volar long
radiolunate ligament (radiolunotriquetral ligament)
wrist palmar flexion
area of weakness disappears

o
o

wrist dorsiflexion
area of weakness increases
in perilunate dislocations, this space allows the distal carpal row to
separate from the lunate

in lunate dislocations, the lunate escapes into this space

Extrinsic Ligaments
Volar radiocarpal ligaments

radial collateral

radioscaphocapitate

at risk for injury with excessively large radial styloid


from radial styloid to capitate, creating a sling to support
the waist of the scaphoid
preserve when doing proximal row carpectomy
acts as primary stabilizer of the wrist after PRC and
prevents ulnar drift
o radioscapholunate
Ligament of Testut and Kuentz
only functions as neurovascular conduit
not a true ligament
does not add mechanical strength
o long radiolunate
also called radiolunotriquetral or volar radiolunate
ligament
counteracts ulnar-distal translocation of the lunate
abnormal in Madelung's deformity
o short radiolunate
stabilizes lunate
Volar ulnocarpal ligaments
o ulnotriquetral
o ulnolunate
o ulnocapitate
DORSAL LIGAMENT

radiotriquetral
o must also be disrupted for VISI deformity to form (in
combination with rupture of lunotriquetral interosseous ligament
rupture)
dorsal intercarpal (DIC)
radiolunate
radioscaphoid
Intrinsic (Interosseous) ligaments
Proximal row
o scapholunate ligament

primary stabilizer of scapholunate joint


composed of 3 components
dorsal portion
thickest and strongest
prevents translation
volar portion
prevents rotation
proximal portion
no significant strength
disruption leads to lunate extension when the scaphoid
flexes

creating DISI deformity


o lunotriquetral ligament
composed of 3 components
dorsal
volar
proximal
disruption leads to lunate flexion when the scaphoid is
normally aligned
creating VISI deformity (in combination with rupture
of dorsal radiotriquetral rupture)
Distal row
o trapeziotrapezoid ligament
o trapeziocapitate ligament
o capitohamate ligament
Palmar midcarpal
o scaphotrapeziotrapezoid
o scaphocapitate
o triquetralcapitate
o triquetralhamate

Motion of the Fingers

Motion of the Fingers

Inspection
Skin
discoloration
erythema (cellulitis)
white (arterial insufficiency)
blue/purple (venous congestion)
black spots (melanoma)
o trophic changes (i.e. increased hair growth or altered sweat production)
can represent derangement of sympathetic nervous system
o scars/wounds
Swelling
Muscle atrophy
o thenar atrophy
median nerve involvement
caused by carpal tunnel syndrome
o interossei atrophy
ulnar nerve involvement
caused by cubital tunnel or cervical radiculopathy
o subcutaneous atrophy
locally post-steroid injection
Deformity
o asymmetry
o angulation
o rotation
o absence of normal anatomy (previous amputation)
o

cascade sign
fingers converge toward the scaphoid tubercle when flexed at
the MCPJ and PIPJ
if one or more fingers do not converge, then trauma to the digits
has likely altered normal alignment

Palpation
Masses (ganglions, nodules)
Temperature
o warm: infection, inflammation
o cool: vascular pathology
Tenderness
Crepitus (fracture)
Clicking or snapping (tendonitis)
Joint effusion (infection, inflammation, trauma)
Range of Motion
Active and passive
Finger
o

MCP: 0 extension to 85 of flexion

PIP: 0 extension to 110 of flexion


DIP: 0 extension to 65 of flexion

Wrist
o
o
o

60 flexion
60 extension
50 radioulnar deviation arc

Neurovascular Exam
Sensation
o two-point discrimination
Motor
o radial nerve: test thumb IP joint extension against resistence
o median nerve
recurrent motor branch: palmar abduction of thumb
anterior interosseous branch: flexion of thumb IP and index DIP
("A-OK sign")
o ulnar nerve: cross-fingers or abduct fingers against resistence
Vascular
o radial pulse
o ulnar pulse
o Allen's test
o capillary refill

Palpation

grind test
used to test for pathology at the thumb carpometacarpal
joint (CMC)
examiners applies axial load to first metacarpal and
rotates or "grinds" it
positive findings: pain, crepitus, instability
o Finkelstein's
used to test for DeQuervain's tenosynovitis
patient makes fist with fingers overlying thumb
examiner gently ulnarly deviates the wrist
positive findings: pain along the 1st compartment
Range of motion
o flexor profundus
used to test continuity of FDP tendons
MCP + PIP joints held in extension while patient asked to
flex FDP, thereby isolating FDP (from FDS) as the only
tendon capable of flexing the finger
o flexor sublimus
used to test for continuity of FDS tendon
MCP, PIP and DIP of all fingers held in extension with
hand flat and palm up; the finger to be tested is then
allowed to flex at PIP joint.
o Bunnel's test
examiner passively flexes PIPJ twice
first with MCP in extension
next with MCP held in flexion
intrinsic tightness present if PIP can be flexed easily
when MCP is flexed but NOT when MCP is extended
extrinsic tightness present if PIP can be flexed easily
when MCP is extended but NOT when MCP is flexed
Stability assessment
o scaphoid shift test (Watson's test)
tests for scapholunate ligament tear
examiner places thumb on distal pole of scaphoid on
palmar side of wrist and applies constant pressure as the
wrist is radially and ulnarly deviated
dorsal wrist pain or "clunk" may indicate instability
o lunotriquetral ballottement
tests for lunotriquetral ligament tear
examiner secures the pisotriquetral unit with the thumb
and index finger of one hand and the lunate with the
other hand
anterior and posterior stresses are placed on the LT joint
o

positive findings are increased laxity and accompanying


pain
o midcarpal instability
examiner stabilizes distal radius and ulna with nondominant hand and moves patients wrist from radial
deviation to ulnar deviation, whilst applying an axial load
a positive test occurs when a clunk is felt when the wrist
is ulnarly deviated
o ulnar carpal abutement
tests for TFCC tear or ulnar-carpal impingement
examiner ulnarly deviates wrist with axial compression
positive if test reproduces pain or a 'pop' or 'click' is
heard
o Gamekeeper's
tests for ulnar collateral ligament tear at MCP of thumb
examiner stresses first MCPJ into radial deviation with
MCPJ in fully flexed and extended positions
positive test if > 30 degrees of laxity in both positions (or
gross laxity compared to other side)
Nerve assessment
o Tinel's
tests for carpal tunnel syndrome
examiner percusses with two fingers over distal palmar
crease in the midline
positive if patient reports paresthesias in median nerve
distribution
o Phalen's
tests for carpal tunnel syndrome
with the hands pointed up, the patient's wrist is allowed to
flex by gravity in palmar flexion for 2 minutes maximum
positive if patient reports paresthesias in median nerve
distribution
o Froment's sign
tests for ulnar nerve motor weakness
patient asked to hold a piece of paper between thumb
and radial side of index
positive if as the paper is pulled away by the examiner
the patient flexes the thumb IP joint in an attempt to hold
on to paper
o Wartenberg's sign
tests ulnar nerve motor weakness
patient asked to hold fingers fully adducted with MCP,
PIP, and DIP joints fully extended

positive if small finger drifts away from others into


abduction
Jeanne's sign
tests for ulnar nerve motor weakness
ask patient to demosntrate key pinch
positive finding if patients first MCP joint is
hyperextended

Vascular Evaluation of the Hand


Three-phase bone scan
o Phase I (2 minutes)
shows an extremity anteriogram
o Phase II (5-10 minutes)
shows cellulits and synovial inflammation
o Phase III (2-3 hours)
shows bone images
RSD diagnosed with positive phase III that does not
correlate with positive Phase I and Phase II
o Phase IV (24 hours)
can differentiate osteomyelitis from adjacent cellulitis
Duplex scan
o is helpful for arterial intimal lesions (true and false aneurysms)
Arteriogram
o remains gold standard for embolic disease
o downside is it is invasive with risks
Ultrasound duplex
o imaging is becoming more sensitive and specific
Segmental limb pressures

Nerve Conduction Studies


Definition
o comprises nerve conduction velocity (NCV) studies and
electromyography (EMG)
o used to localize areas of compression and neuropathy
o distinguish
lower vs upper motor neuron lesions
spinal root, trunk, division, cord or peripheral nerve lesion
o determine severity and prognosis
neuropraxia has good prognosis
axonotmesis/neurotmesis has poor prognosis

demonstrate denervation, reinnervation, aberrant reinnervation,


motor end plate lesion
o valuable in worker's compensation patients with secondary
gain issues
Indications
o carpal tunnel syndrome
o cubital tunnel syndrom
o cervical radiculopathy
o lumbar radiculopathy
o nerve dysfunction of the shoulder (e.g., scapular winging)
o

Flexor Carpi Radialis Tendinitis


Authors: Colin Woon, David Abbasi
Topic updated on 09/07/13 1:46pm

Introduction
A condition characterized by inflammation of the FCR tendon sheath
Demographics
o incidence
uncommon
o risk factors
repetitive wrist flexion
golfers and racquet sports
manual labor
Pathoanatomy
o primary stenosing tenosynovitis within the fibroosseous tunnel
(see Anatomy)
o secondary tendinitis associated with
scaphoid fracture
scaphoid cysts
distal radius fracture
scaphoid-trapezium-trapezoid joint arthritis
thumb CMC joint arthritis
Prognosis
o prognosis is poor if the following are present
history of overuse
worker's compensation
failure to respond to local injection
long duration of symptoms
Anatomy
Flexor carpi radialis musculotendinous unit

o
o

FCR muscle
bipennate
FCR tendon
enveloped by sheath from musculotendinous origin to trapezium
no fibrous sheath distal to trapezium
enters fibroosseous tunnel at the proximal border of the
trapezium
boundaries

radial = body of the trapezium


palmar = trapezial crest, transverse carpal

ligament
ulnar = retinacular septum from transverse carpal
ligament (separates FCR from carpal tunnel)
dorsal = reflection of retinacular septum on
trapezium body
space
within the tunnel
the FCR tendon occupies 90% of space
is in direct contact with the roughened
surface of the trapezium
more prone to constriction, tendinitis,
attrition, rupture
proximal to the tunnel
the FCR tendon occupies 50-65% of space
within FCR sheath proximal to the tunnel
less prone to constriction
but more prone to mechanical irritation from
osteophytes
insertion

small slip (1-2mm) inserts into trapezial crest


80% of remaining tendon inserts into 2nd metacarpal
20% of remaining tendon inserts into 3rd metacarpal

Presentation
Symptoms

volar radial aspect of the wrist


Physical exam
o tenderness over volar radial forearm along FCR tendon at
distal wrist flexion crease
o provocative test
resisted wrist flexion triggers pain
resisted radial wrist deviation triggers pain
o

Imaging
Radiographs
o findings
in primary tendinitis, radiographs are unremarkable
in secondary tendinitis, the following may be present
healed scaphoid fracture
healed distal radius fracture
exostosis or arthritis of scaphotrapezoid joint or
thumb CMC
MRI
o views
best seen on T2
o findings
increased signal around FCR sheath on T2 image

may find associated conditions in secondary tendinitis


ganglion
scaphoid cyst
Studies
Diagnostic injection

injection of local anesthetic along FCR sheath relieves symptoms

Differentials
Thumb CMC arthritis
Scaphoid cyst
Ganglion
De Quervain's tenosynovitis

Treatment
Nonoperative
o immobilization, NSAIDS, steroid injection
indications
first line of treatment
technique
direct steroid injection in proximity, but not into
tendon
outcomes
usually effective for primary tendinitis
unsuccessful in secondary tendinitis if other lesions
are present (e.g. osteophytes)
Operative
o surgical release of FCR tendon sheath
indications
rarely needed but can be effective in recalcitrant
cases
Surgical Technique

Surgical release of FCR tendon sheath


o

approach
volar longitudinal incision starting proximal to the wrist crease,
extending over proximal thenar eminence

care taken to avoid


palmar cutaneous branch of median nerve
lateral antebrachial cutaneous nerve
superficial sensory radial nerve
o

technique
elevate and reflect thenar muscles radially
expose FCR sheath
open FCR sheath proximally in the distal forearm, and extend to
the trapezial crest
at the trapezial crest, the tendon enters the FCR tunnel
at this point, incise the sheath along the ulnar margin, taking
care not to injure the tendon
mobilize tendon from trapezoidal groove (releasing trapezial
insertion)

Complications
Complications of disease
o FCR attrition and rupture
Complications of surgical release
o cutaneous nerve injury
palmar cutaneous branch of median nerve
lateral antebrachial cutaneous nerve
superficial sensory radial nerve
o injury to deep palmar arch
o injury to FPL tendon (lies superficial to FCR tendon)
o injury to FCR tendon within the tunnel
decompression is easy proximal to the tunnel (incision of FCR

sheath)
within FCR fibroosseous tunnel, take care to avoid cutting FCR
tendon

Extensor Tendon Conditions

Extensor Tendon Injuries


Author: Daniel Hatch
Topic updated on 12/15/14 5:02pm

Introduction

Injury can be caused by laceration, trauma, or overuse


Epidemiology
o most commonly injured digit is the long finger
o zone VI is the most frequently injured zone
Mechanism
o Zone I
forced flexion of extended DIP joint
o Zone II
dorsal laceration or crush injury
o Zone V
commonly from "fight bite"
sagittal band rupture ("flea flicker injury")
forced extension of flexed digit

most common in long finger


Classification

Zone I

Zone II
Zone III

Zone IV
Zone V

Zone VI

Zones of Extensor Tendon Injuries


Disruption of terminal extensor tendon
distal to or at the DIP joint of the fingers and
IP joint of the thumb (EPL)
Mallet Finger

Disruption of tendon over middle phalanx or


proximal phalanx of thumb (EPL)
Disruption over the PIP joint of digit (central
slip) or MCP joint of thumb (EPL and EPB
Boutonniere deformity

Disruption over the proximal phalanx of digit


or metacarpal of thumb (EPL and EPB)
Disruption over MCP joint of digit or CMC
joint of thumb (EPL and EPB)
"Fight bite" common
Sagittal band rupture

Disruption over the metacarpal


Nerve and vessel injury likely
Zone VII Disruption at the wrist joint
Must repair retinaculum to prevent
bowstringing
Tendon repair followed by immobilization
with wrist in 40 extension and MCP joint in
20 flexion for 3-4 weeks
Zone VIII Disruption at the distal forearm
Zone VIII Extensor muscle belly
Usually from penetrating trauma
Often have associated neurologic injury
Tendon repair followed by immobilization
with elbow in flexion and wrist in extension

Presentation
Zone I
Inability to extend at the DIP joint
Zone III
o Elson test
flex the patient's PIP joint over a table 90 degrees and ask them to
extend against resistance
if central slip is intact, DIP will remain supple
if central slip disrupted, DIP will be rigid
Zone V
o extensor lag and flexion loss common
o sagittal band rupture
rupture of stronger radial fibers of sagittal band may lead to
extensor tendon subluxation
finger held in flexed position at MCP joint with no active extension
o

Imaging
Radiographs
o AP and lateral of digit to verify no bony avulsion (boney mallet)
Treatment

Nonoperative
o immobilization with early protected motion
indications
lacerations < 50% of tendon in all zones if patient can
extend digit against resistance
o DIP extension splinting
indications
acute (<12 weeks) Zone 1 injury (mallet finger)
nondisplaced bony mallet
chronic mallet finger (>12 weeks) if joint supple,
congruent
techniques
full-time splinting for six weeks

part-time splinting for four to six weeks


avoid hyperextension, which may cause skin necrosis
maintain PIP motion
outcomes
noncompliance is a common problem
PIP extension splinting
indications
closed central slip injury (zone III)
techniques
full-time splinting for six weeks
part-time splinting for four to six weeks
maintain DIP flexion
MCP extension splinting
indications

closed zone V sagittal band rupture


techniques
full-time splinting for four to six weeks

Operative
o immediate I&D
indications
fight bite to MCP joint
techniques
close loosely or in delayed fashion
treat with culture-specific antibiotics, although Eikenella
corrodens is a common mouth organism
o

tendon repair
indications
laceration > 50% of tendon width in all zones
fixation of bony avulsion
indications
boney mallet finger with P3 volar subluxation
techniques
closed reduction and percutaneous pinning through DIP joint
extension block pinning
ORIF if it involves >50% of the articular surface

tendon reconstruction
indications
chronic tendon injury or when repair not possible

central slip reconstruction


techniques
tendon graft
extensor turndown
lateral band mobilization
transverse retinacular ligament
FDS slip

EIP to EPL tendon transfer


indications
chronic EPL rupture

Surgical Techniques

Tendon Repair
o incision technique
utilize laceration, when present, and extend incision as
needed to gain appropriate exposure
longitudinal incision may be utilized across joints on the
dorsum of digits, unlike the palmar side
o suture technique
# of suture strands that cross the repair site is more important
than the number of grasping loops
in general strength increases with increasing number of
sutures crossing the repair site, thickness of the suture, and

o
o

locking of the stitch


4-6 strands provide adequate strength for early active motion
circumferential epitendinous suture
Optional for reinforcement
repair failure
tendon repairs are weakest between postoperative day 6 and
12
repair usually fails at knots

Tendon Reconstruction
o usually done as two stage procedure
first a silicon tendon implant is placed to create a favorable
tendon bed
wait 3-4 months and then place biologic tendon graft
only perform single stage reconstruction if flexor sheath is
pristine and digit has full ROM
o available grafts include
palmaris longus (absent in 15% of population)
most common
plantaris (absent in 19%)
indicated if longer graft is needed
long toe extensor
o pulley reconstruction
one pulley should be reconstructed proximal and distal to each
joint
methods include belt loop method and FDS tail method
Tenolysis
o indications
adhesion formation with loss of finger flexion
wait for soft tissue stabilization (> 3 months) and full passive
motion of all joints
o postoperative
o follow with extensive therapy
Complications
Adhesion formation
o leads to loss of finger flexion
o common in zone IV and VII and older patients
o prevented with early protected ROM and dynamic splinting (zone IV)
o treatment
extensor tenolysis with early motion indicated after failure of
nonoperative management, usually 3-6 months
tenolysis contraindicated if done in conjunction with other procedures
that require joint immobilization
Tendon rupture
o causes include poor suture material or surgical technique, aggressive

therapy, and noncompliance


o incidence
5%
most frequently during first 7 to 10 days post-op
o treatment
early recognition may allow revision repair
tendon reconstruction for late rupture or rupture with excessive
scarring
Swan neck deformity
o caused by prolonged DIP flexion with dorsal subluxation of lateral bands and
PIP joint hyperextension
o treatment
Fowler central slip tenotomy
spiral oblique ligament reconstruction
Boutonniere deformity (DIP hyperextension)
o caused by central slip disruption and lateral band volar subluxation
o treatment
dynamic splinting or serial casting for maximal passive motion
terminal extensor tenotomy, PIP volar plate release

Mallet Finger
Author: Ujash Sheth
Topic updated on 12/16/15 2:29am

Introduction
A finger deformity caused by disruption
of the terminal extensor tendon distal to
DIP joint
o the disruption may be bony or
tendinous
Epidemiology
o risk factors
usually occur in the work environment or during
participation in sports
o demographics
common in young to middle-aged males and older
females
o body location
most frequently involves long, ring and small fingers of
dominant hand
Pathophysiology
o mechanism of injury
traumatic impaction blow
usually caused by a traumatic impaction blow (i.e.
sudden forced flexion) to the tip of the finger in the
extended position.

forces the DIP joint into forced flexion


dorsal laceration
a less common mechanism of injury is a sharp or
crushing-type laceration to the dorsal DIP joint
Classification
Doyle's Classification
Doyle's Classification of Mallet Finger Injuries
Type I Closed injury with or without small dorsal avulusion fracture
Type II Open injury (laceration)
Type III Open injury (deep abrasion involving skin and tendon substance)
Type IV Mallet fracture
A = distal phalanx physeal injury (pediatrics)
B = fracture fragment involving 20% to 50% of articular surface (adult)
C = fracture fragment >50% of articular surface (adult)

Presentation
Symptoms
o primary symptoms
painful and swollen DIP joint following impaction injury to
finger
often in ball sports
Physical exam
o inspection
fingertip rest at ~45 of flexion
o motion
lack of active DIP extension
Imaging
Radiographs
o findings
usually see bony avulsion of distal phalanx

may be a ligamentous injury with normal bony anatomy

Treatment
Nonoperative
o

extension splinting of DIP joint for 6-8 weeks

indications
acute soft tissue injury (< than 12 weeks)
nondisplaced bony mallet injury
technique
maintain free movement of the PIP joint
worn for 6-8 weeks
volar splinting has less complications than dorsal
splinting
avoid hyperextension
begin progressive flexion exercises at 6 weeks
Operative
o CRPP vs ORIF
indications
absolute indications

volar subluxation of distal phalanx


relative indications
>50% of articular surface involved
>2mm articular gap
surgical reconstruction of terminal tendon
indications
chronic injury (> 12 weeks) with healthy joint
outcomes
tendon reconstruction has a high complication rate
(~ 50%)
DIP arthrodesis
indications
painful, stiff, arthritic DIP joint
Swan neck deformity correction
indications
Swan neck deformity present

Techniques

CRPP vs ORIF
o approach
dorsal midline incision
o fixation
simple pin fixation
dorsal blocking pin
Surgical reconstruction of terminal tendon
o repair
this may be done with direct repair/tendon
advancement, tenodermodesis, or spiral oblique
retinacular ligament reconstruction
Swan neck deformity correction
o techniques to correct Swan neck deformity include
lateral band tenodesis
FDS tenodesis
Fowler central slip tenotomy
minimal Swan Neck deformities may correct with
treatment of the DIP pathology alone
Complications
Extensor lag
o a slight residual extensor lag of < 10 may be present at
completion of closed treatment
Swan neck deformities
o occurs due to
attenuation of volar plate and transverse retinacular

ligament at PIP joint


dorsal subluxation of lateral bands
resulting PIP hyperextension
contracture of triangular ligament maintains deformity
(OBQ12.2) A 42-year-old sustains a left finger injury while attempting to catch a baseball for his
son. He presents with left, long finger pain and an inability to extend his middle finger at the
distal interphalangeal joint. A radiograph after closed reduction and splinting is shown in Figure
A. What is the best course of treatment?

1.

Reduction and pinning

2.

Repeat splinting of the distal interphalangeal joint in extension

3.

Splinting of the distal and proximal interphalangeal joints in extension

4.

Observation

5.

Fusion of the distal interphalangeal joint

PREFERRED RESPONSE 1
The radiograph depicts a bony mallet injury with volar subluxation of the distal phalanx after
splinting of the DIP joint in extension, which is an indication for reduction and pinning.
A mallet deformity is caused by disruption of the terminal extensor tendon distal to DIP joint.
Occasionally, a bony avulsion of the distal phalanx is noted on radiographs. "Bony" mallet
fingers will rarely require surgical fixation. It is important to attempt to splint a bony mallet injury
and get a new radiograph prior to making the decision for operative treatment. Indications for
surgical management of this condition include volar subluxation of the distal phalanx even after
DIP splinting.
Stern et al. found a higher long-term complication rate with surgical treatment of mallet injuries.
He also noted 15 degrees more DIP flexion at follow-up in the splinting group compared to the
surgical group.
Pegoli et al. describe an extension block technique for treatment of this injury with good results.
Their indications for surgery included the presence of a large bone fragment, and palmar
subluxation or the loss of joint congruity of the distal interphalangeal joint.
Theivendran et al. review the surgical treatment of DIP joint fractures and state that 30%
articular involvement is an indication for operative treatment.
Figure A shows a lateral radiograph with a large intra-articular bony avulsion fragment and volar
subluxation of the distal phalanx.

Incorrect Answers:
Answer 2,3,4: This patient meets the indications for ORIF and nonoperative modalities would
not be appropriate.
Answer 5: A DIP fusion in a young patient would not be appropriate.

(OBQ12.85) A 27-year-old male presents with finger pain 2 days after suffering an injury while
playing basketball. Physical exam shows swelling of the distal interphalangeal joint with no
evidence of open injury. A radiograph is shown in Figure A. Which of the following is the most
appropriate treatment at this time?

FIGURES: A
1.

Extension splinting of DIP joint for 6-8 weeks

2.

Closed reduction and percutaneous pinning

3.

Open reduction and internal fixation

4.

DIP arthrodesis

5.

Swan neck deformity correction

PREFERRED RESPONSE 1
The clinical presentation is consistent with a non-displaced bony mallet finger without joint
subluxation. Extension splinting of the DIP joint for 6-8 weeks is the most appropriate treatment.
A mallet finger is a deformity caused by disruption of the terminal extensor tendon distal to DIP
joint. Treatment is dictated by the degree of displacement and acuity of injury. Acute injuries
with minimal displacement and no joint subluxation are treated with extension bracing for 6-8
weeks. ORIF or closed reduction and percutaneous fixation is indicated for chronic injuries or
acute injuries with volar displacement of the distal phalanx, a >2mm articular step-off, or when a
majority (>50%) of the articular surface is involved.
Pegoli et al. report the results of extension block Kirschner wire fixation for the treatment of
mallet fractures of the distal phalanx in 65 consecutive patients. Their results showed 46%
excellent, 32% good, 20% fair and 2% poor results. The recommend the following indications for
operative treatment: presence of a large bone fragment, palmar subluxation, or the loss of joint
congruity of the distal interphalangeal joint.
Theivendran et al. report operative fixation is indicated when more than 30% of the articular
surface is involved with or without subluxation of the joint. They summarize the management

options for intra-articular distal interphalangeal fractures, placing particular emphasis on surgical
treatment.
Figure A shows a non-displaced bony mallet Injury. Illustration A shows an example of an
extension splint used for non-operative management of mallet injuries. Illustration V is a video
showing the surgical technique for a Mallet finger.
Incorrect Answers:
Answer 2: Closed reduction and percutaneous pinning is indicated for a displaced mallet finger
injury with joint subluxation.
Answer 3: Open reduction and internal fixation is indicated for displaced, subluxed mallet finger
injuries that can not be reduced closed.
Answer 4: DIP arthrodesis is indicated in patients with a painful, stiff, arthritic DIP joint.
Answer 5: Swan neck deformity correction is indicated for a chronic mallet finger that has led to
a swan neck deformity.
Illustrations: A

Sagittal Band Rupture (traumatic


extensor tendon dislocation)
Author: Colin

Woon

Topic updated on 12/16/15 2:49am

Introduction
Sagittal band (SB) rupture leads to
dislocation of the extensor tendon
o also known as "boxer's
knuckle"
Epidemiology
o demographics
more common in
pugilists
index and middle
finger in professionals
ring and little finger in amateurs
o location
the middle finger is most commonly involved
index 14%
middle 48%
ring 7%
little 31%
the radial SB is more commonly involved
radial:ulnar = 9:1
Mechanisms
o traumatic
forceful resisted flexion or extension
laceration of extensor hood

direct blow to MCP joint


atraumatic
inflammatory (e.g. rheumatoid arthritis)

spontaneously during routine activities


Associated conditions
o MCP joint collateral ligament injuries

Anatomy
Extensor mechanism comprises
o tendons
EDC/EIP/EDM
lumbricals
interossei
o retinacular system
sagittal bands
the sagittal bands are part of a closed cylindrical
tube (or girdle) that surrounds the metacarpal head
and MCP along with the palmar plate

origin
volar plate and intermetacarpal ligament at
the metacarpal neck
insertion
extensor mechanism (curving around radial

and ulnar side of MCP joint)


retinacular ligaments
triangular ligament

Sagittal band
o function
the SB is the primary stabilizer of the extensor tendon at
the MCP joint

juncturae tendinum are the secondary


stabilizers

resists ulnar deviation of the tendon, especially during


MCP flexion
prevents tendon bowstringing during MCP joint
hyperextension
biomechanics
ulnar sagittal band
partial or complete sectioning does not lead to
extensor tendon dislocation
radial sagittal band
distal sectioning does not produce extensor tendon
instability
complete sectioning leads to extensor dislocation
sectioning of 50% of the proximal SB leads to
extensor tendon subluxation
extensor tendon
instability after sectioning is greater with wrist
flexion

instability after sectioning is greater in the central


digits (than border digits)
the least stable tendon is the middle finger
the most stable tendon is the little finger
junctura tendinum stabilize the small
finger
Classification

Rayan and Murray Classification of Closed SB Injury

Type
Type I

Description
SB injury without extensor tendon instability

Type II SB injury with tendon subluxation


Type III SB injury with tendon dislocation

Image

Presentation
Symptoms
o MCP soreness
Physical exam
o tendon snapping
o ulnar deviation of the digits at the MCP joint (rheumatoid
arthritis)
o inability to initiate extension
o pseudo-triggering
o extensor tendon dislocation into intermetacarpal gully
most unstable during MCP flexion with wrist flexed
least unstable during MCP flexion with wrist extended
o provocative test
pain when extending MCP joint against resistance (with
both IP joints extended)
Imaging
Radiographs
o required views
hand PA, lateral, oblique
o optional view
Brewerton view
AP with dorsal surface of fingers touching the
cassette and MCP joints flexed 45deg
stress view
to rule out collateral ligament avulsion/injury
o findings
exclude mechanical/bony pathology limiting extension, or
predisposing to sagittal band rupture
may show dropped fingers and ulnar deviation in
rheumatoid arthritis

Ultrasound (dynamic)
o indications
when swelling obscures the physical exam
o findings
subluxation of EDC tendon relative to metacarpal head
on MCP flexion

MRI
o

indications
to establish diagnosis of SB disruption (radial or ulnar
SB)
may show underlying etiology e.g. synovitis in rheumatoid
arthritis
views
axial images at the level of the long MCP
with MCP joint flexed for maximum EDC tendon
displacement
findings

poor definition, focal discontinuity and focal thickening in


acute injury
subluxation of extensor tendon in radial direction due
ulnar SB defect

dislocation of extensor tendon into ulnar intermetacarpal


gully radial SB defect

Differentials
MCP joint collateral ligament injury
EDC tendon rupture
Trigger finger
Junctura tendinum disruption
Congenital sagittal band deficiency
MCP joint arthritis
Treatment
Nonoperative

extension splint for 4-6 weeks


indications
acute injuries (within one week)

Operative
o direct repair (Kettlekamp)
indications
chronic injuries (more than one week) where
primary repair is possible
professional athlete

extensor centralization procedure


indications
chronic injuries (more than one week) where
primary repair is NOT possible
professional athlete

Techniques
Extensor Centralization Procedures
o various techniques described including
trapdoor flap
ulnar based partial thickness capsular flap created
tendon placed deep to flap
flap resutured to capsule

Kilgore tendon slip


distally based slip of EDC tendon on radial side
looped around radial collateral ligament
sutured to itself after tensioning to centralize

tendon
Carroll tendon slip
distally based slip of EDC tendon on ulnar side
routed deep to affected tendon and around radial
collateral ligament
sutured to itself after tensioning to centralize
tendon

McCoy tendon slip


proximally based slip of EDC tendon
looped around lumbrical insertion
sutured to itself after tensioning to centralize
tendon

Watson EDC tendon transfer


distally based slip of EDC tendon slip
looped under deep transverse metacarpal ligament
weaved to remaining EDC tendon after tensioning

to centralize tendon

Wheeldon junctural reinforcement


for a middle finger radial SB rupture, the juncturae
tendinum (JT) of the ring finger is divided close to
the ring finger,
bring JT over the extensor tendon
attach JT to the torn SB

fascial strips or free tendon graft


Qbank (2 Questions)
(OBQ12.262) A 28-year-old NFL running back complains of continued hand pain three days
following an injury sustained while being tackled. He was splinted on the field. He has tenderness
over the long finger metacarpal head, with subluxation of the extensor tendon into the
intermetacarpal area during active metacarpophalangeal joint flexion. A representative MRI is
shown in Figure A. What is the next best step in management of this patient? Review Topic

FIGURES: A

1.

Observation alone

2.

Continued splinting in flexion

3.

Continued splinting in extension

4.

Open repair of the disrupted junctura tendinae

5.

Open repair of the disrupted sagittal band

PREFERRED RESPONSE 5
Based on the history and physical exam findings this patient has sustained a traumatic rupture of
the sagittal band. In this professional athlete, the next best step would be to perform an open
repair of the sagittal band. This will allow for earlier aggressive rehabilitation and a quicker return
to sport.
Sagittal band ruptures may be traumatic (as in this case) or attritional in nature (as in rheumatoid
arthritis). A direct blow to the MCP leads to forced flexion of the digit and subsequent
stretching/rupture of the affected structure. On physical exam the tendons are most unstable with
the wrist flexed; MCP flexion will lead to dislocation of the tendon into the intermetacarpal gutter.
Acute injuries may be treated with extension bracing for 4-6 weeks, but in professional athletes,
direct open repair of the sagittal band is indicated.
Catalano et al. review sagittal band injuries treated with a thermally molded plastic splint that
held the MCP in ~25-35 degrees of hyperextension. Patients were evaluated over 14 months; out
of 11 sagittal band injuries, splinting was successful in eight of them. They recommend initial
nonsurgical management with custom splinting.
Hame et al. review the results of the management of sagittal band injuries in the professional
athlete. The lesion commonly found was the disruption of the extensor mechanism with
predictable sagittal band tears. In their series, all patients regained full range of motion and
returned to their respective sports. They recommend surgical intervention in elite athletes in the
form of extensor tendon centralization and sagittal band repair.
Figure A shows a T1 weighted axial cut of the affected hand; subluxation of the tendon (arrow)
can be identified with disruption of the sagittal band (arrowhead).
The video provided briefly reviews injury to the sagittal band.
Incorrect Answers
Answer 1: Observation is not indicated in this patient
Answer 2, 3: Splinting in extension would be an acceptable option in the non-athlete, but direct
repair is indicated in a professional athlete
Answer 4: The junctura tendinae are not injured in this patient

Snapping ECU
Author: David

Abbasi

Topic updated on 12/17/14 12:25pm

Introduction

Overuse of wrist can lead to spectrum


of ECU tendonitis and instability
Pathoanatomy
o ECU subluxation is secondary to attenuation and rupture of the
ECU subsheath in the 6th dorsal compartment
o ECU subluxates in volar and ulnar direction
o subluxation and snapping can lead to ECU tendonitis
Anatomy
Extensor tendon compartments

Compartment 1 (De Quervain's Tenosynovitis )

APL
EBP
o

Compartment 2 (Intersection syndrome )


ECRL

ECRB
Compartment 3

EPL
Compartment 4

EIP
EDC
o Compartment 5 (Vaughn-Jackson Syndrome )
EDM
o Compartment 6 (Snapping ECU )
ECU
ECU tendon
o ECU subsheath is part of the TFCC that is most critical to ECU stability

Presentation
Symptoms
o pain and snapping over dorsal ulnar wrist
Physical exam
o extension and supination of the wrist elicit a painful snap
o ECU tendon reduces with pronation
Imaging
Radiographs
o unremarkable
Ultrasound
o can dynamically assess ECU stability
MRI
o can show tendonitis, TFCC pathology, or degenerative tears of ECU

Treatment
Nonoperative
o wrist splint or long arm cast
indications
first line of treatment
technique
arm immobilized in pronation and slight radial
deviation
Operative
o ECU subsheath reconstruction +- wrist arthroscopy

indications
if nonoperative management fails
technique
direct repair in acute cases
chronic cases may require a extensor retinaculum
flap for ECU subsheath reconstruction
wrist arthroscopy shows concurrent TFCC tears in
50% of cases

De Quervain's Tenosynovitis
Author: Michael

Hughes MD

Topic updated on 06/02/15 4:50pm

Introduction
A stenosing tenosynovial inflammation of the 1st dorsal
compartment which includes
o abductor pollicis longus (APL)
o extensor pollicis brevis (EPB)
Epidemiology
o common in
woman 30-50 years
racquet sports
Pathophysiology
o causes include
idiopathic
overuse
golfers and racquet sports
post-traumatic
postpartum
Anatomy
Extensor tendon compartments

Compartment 1 (De Quervain's Tenosynovitis)

Compartment 2 (Intersection syndrome )


ECRL
ECRB
Compartment 3
EPL
Compartment 4
EIP
EDC
Compartment 5 (Vaughn-Jackson Syndrome )
EDM
Compartment 6 (Snapping ECU )
ECU

o
o

o
o

APL
EPB

Presentation

Symptoms
o radial sided wrist pain
Physical exam
o Finkelstein provocative maneuver
ulnar deviated wrist with thumb clenched in fist
tenderness over 1st dorsal compartment at level of radial
styloid
location of tenderness differentiates from
intersection syndrome (tenderness 5cm proximal to
wrist joint)
Imaging
Radiographs
o recommended views
radiographs not indicated

Treatment
Nonoperative
o rest, NSAIDS, thumb spica splint, steroid injection
indications
first line of treatment
technique
steroid injections into first dorsal compartment
Operative
o surgical release of 1st dorsal compartment
indications
severe symptoms and nonoperative management
has failed
Surgical Techniques

Surgical release of 1st dorsal compartment


o approach
transverse incision with release on dorsal side of 1st
compartment to prevent volar subluxation of the tendon
has variable anatomy with APL usually having at
least 2 tendon slips and its own fibro-osseous
compartment
a distinct EPB sheath is often encountered dorsally
Complications

Sensory branch of radial nerve injury


Neuroma formation
Failure to decompress with recurrence
o may be caused by failure to recognize and decompress EPB or
APL lying in separate subsheath/compartment

Complex regional pain syndrome


Qbank (2 Questions)
(OBQ12.253) A 45-year-old patient presents with recurrence of radial sided wrist pain after
undergoing a first dorsal compartment release about 3 months ago. The surgery was completed
by one of your partners; operative reports indicate that the sheath was incised on the dorsal
edge. On physical exam she is found to have normal appearing skin, a negative Tinels sign, and
a positive Finklestein test. What is the most likely cause of the recurrence of her symptoms?
Review Topic
1.

Development of neuroma

2.

Complex regional pain syndrome

3.

Failure to decompress the EPB sub-sheath

4.

Failure to decompress the EPL sub-sheath

5.

Failure to decompress the APB sub-sheath

PREFERRED RESPONSE 3
Based on the history and clinical findings this patient has de Quervains tenosynovitis. The
recurrence of her symptoms can be attributed to a failure to recognize and decompress the EPB
sub-sheath.
De Quervains tenosynovitis is a stenosing inflammatory condition of the first dorsal compartment
of the wrist (APL/EPB). Surgical release of the compartment is indicated after conservative
measures have failed. At the time of the operation, the incision is made on the dorsal side of the
sheath to prevent volar subluxation of the tendons. Failure to identify and release a distinct EPB
sub-sheath or a separate fibro-osseous compartment of the APL can lead to a recurrence of
symptoms.
Alegado et al. report a case of a patient with dysesthesias in the superficial radial nerve
distribution 3 months after undergoing first dorsal compartment release for de Quervains
tenosynovitis. They found a persistent fibrous remnant of the dorsal aspect of the sheath causing
elevation of the superficial radial nerve. They recommend sheath excision or incision of the
sheath at its dorsal attachment to avoid this complication.
Ashurst et al. report a case of a patient presenting with bilateral de Quervains tenosynovitis
secondary to excessive text messaging. Conservative measures afforded the patient complete
symptomatic recovery. They recommend limitation of texting, in conjunction with other standard
treatments, to treat text messaging- associated de Quervains tenosynovitis
Ilyas et al. review the etiology, diagnosis and management of De Quervains tenosynovitis. Nonsurgical management is largely successful and includes splinting and cortisone injections. In
refractory cases, surgical release of the first dorsal compartment is completed. They recommend
meticulous care of the radial sensory nerve and identification of all separate sub-sheaths.
Illustration A shows an operative photo in a patient with multiple APL slips and an EPB that is
hidden within a sub-sheath. Video V gives a brief overview of de Quervains tenosynovitis.

Incorrect Answers
Answer 1: Given the negative Tinels sign on physical exam, the patient is less likely to have a
neuroma.
Answer 2: Her history, symptoms and lack of skin changes are not consistent with complex
regional pain syndrome.
Answers 4, 5: The first dorsal compartment is composed of the APL/EPB. The EPL is in the third
dorsal compartment and the APB is in the thenar compartment.

(OBQ08.9) A 31-year-old mother of a 2-month-old infant complains of radial sided wrist pain.
Corticosteroid injections should be directed into what anatomic area? Review Topic
1.

First carpometacarpal joint

2.

Carpal tunnel

3.

First dorsal compartment near the radial styloid

4.

A1 pulley of thumb

5.

At the crossing of the first and second dorsal compartments

PREFERRED RESPONSE 3
There is an association between the postpartum state and de Quervains tenosynovitis. De
Quervains is a pathologic process of the 1st dorsal (extensor) compartment which contains the
extensor pollicis brevis and abductor pollicis longus tendons. The best choice is #3 because of
the very common and known association of postpartum state and de Quervains as well as the
potential for resolution with appropriately placed steroid injection. Answer #1 refers to basal joint
arthritis which is typically seen in older patients. Answer #2 refers to carpal tunnel syndrome,
which would present with paresthesias in the median nerve distribution. Answer #4 refers to a
trigger thumb. Answer #5 alludes to intersection syndrome which is generally more proximal to
the wrist and results from inflammation at crossing point of 1st dorsal compartment (APL and
EPB) and 2nd dorsal compartment (ECRL, ECRB). To review, the wrist extensor compartments
(from radial to ulnar) are: 1) APL & EPB; 2) ECRL & ECRB (common radial wrist extensors); 3)
EPL; 4) EIP & EDC; 5) EDM; 6) ECU.

Intersection Syndrome
Author: Jan

Szatkowski

Topic updated on 09/25/15 3:29pm

Introduction
Due to inflammation at crossing point of 1st dorsal
compartment (APL and EPB ) and 2nd dorsal compartment (ECRL,
ECRB)
Epidemiology
o common in
rowers
weight lifters
Pathophysiology
o mechanism is repetitive wrist extension
Anatomy
Extensor tendon compartments

Compartment 1 (De Quervain's Tenosynovitis )

Compartment 2 (Intersection syndrome)


ECRL
ECRB
Compartment 3
EPL
Compartment 4
EIP
EDC
Compartment 5 (Vaughn-Jackson Syndrome )
EDM
Compartment 6 (Snapping ECU )
ECU

o
o

o
o

APL
EPB

Presentation
Symptoms
o pain over dorsal forearm and wrist
Physical exam
o tenderness on dorsoradial forearm
approximately 5cm proximal to the wrist joint
o provocative tests

crepitus over area with resisted wrist


extension and thumb extension
Imaging
Radiographs
o not required for the diagnosis or treatment of intersection syndrome

Treatment
Nonoperative
o rest, wrist splinting, steroid injections
indications
first line of treatment
technique
injection aimed into 2nd dorsal
compartment (ECRL, ECRB)
Operative
o surgical debridement and release
indications
rarely indicated in recalcitrant cases
technique
release of the 2nd dorsal compartment
approximately 6 cm proximal to radial styloid
Qbank (1 Questions)
(OBQ07.235) A collegiate rower complains of dorsal wrist pain for 6 weeks refractory to NSAIDs
and bracing. Maximal tenderness is palpated on the dorsoradial forearm approximately 5 cm
proximal to the wrist. Pain is exacerbated with resisted wrist extension. Radiographs are
unremarkable. A steroid injection should be directed into the compartment containing which of
the following structures? Review Topic
1.

APL and EPB tendons

2.

ECRL and ECRB tendons

3.

EPL tendon

4.

APL and ECRB tendons

5.

Brachoradialis tendon

PREFERRED RESPONSE 2
The clinical scenario is consistent with intersection syndrome, a inflammatory response to
overuse at the site of the second dorsal compartment crossing under the first dorsal
compartment approximately 5 cm proximal to the wrist. An anatomical depiction is provided in
illustration A. Injections of the second dorsal compartment, which includes ECRL and ECRB,
may relieve symptoms and quell inflammation. Intersection must be differentiated from
DeQuervain's syndrome, which is tenosynovitis of the first dorsal compartment. Injections of the
first dorsal compartment, which includes APL and EPB, are part of the treatment algorithm for
Dequervain's. Wood et al summarizes the evaluation and treatment of sports-related wrist

injuries. Grundberg et al demonstrates the pathologic abnormality of intersection syndrome is


stenosing tenosynovitis of the second compartment explaining the rationale behind steroid
injections into the sheath.

Wrist Trauma

Scaphoid Fracture
Author: David

Abbasi

Topic updated on 02/26/16 5:40am

Introduction
Scaphoid is most frequently fractured carpal bone
Epidemiology
o incidence
accounts for up to 15% of acute wrist injuries
o location
incidence of fracture by location
waist -65%
proximal third - 25%
distal third - 10%
distal pole is most common location
in kids due to ossification sequence
Pathoanatomy
o most common mechanism of injury is axial load across hyperextended and radially deviated wrist
common in contact sports
o transverse fracture patterns are considered more stable than
vertical or oblique oriented fractures
Associated conditions
o SNAC (Scaphoid Nonunion Advanced Collapse)
Prognosis
o incidence of AVN with fracture location
proximal 5th AVN rate of 100%
proximal 3rd AVN rate of 33%
Anatomy
Articular surface
o > 75% of scaphoid bone is covered by articular cartilage
Blood supply

major blood supply is dorsal carpal branch (branch of the radial


artery)
enters scaphoid in a nonarticular ridge on the dorsal

surface and supplies proximal 80% of scaphoid via


retrograde blood flow
o minor blood supply from superficial palmar arch (branch of
volar radial artery)
enters distal tubercle and supplies distal 20% of scaphoid
Motion
o both intrinsic and extrinsic ligaments attach and surround the
scaphoid
o the scaphoid flexes with wrist flexion and radial deviation and it
extends during wrist extension and ulnar deviation (same as
proximal row)
Also see Wrist Ligaments and Biomechanics for more detail
Presentation
Physical exam
o anatomic snuffbox tenderness dorsally
o scaphoid tubercle tenderness volarly
o pain with resisted pronation
Imaging
Radiographs
o recommended views
AP and lateral

scaphoid view
30 degree wrist extension, 20 degree ulnar
deviation

45 pronation view
o findings
if radiographs are negative and there is a high clinical
suspicion
should repeat radiographs in 14-21 days
Bone scan
o effective to diagnose occult fractures at 72 hours
specificity of 98%, and sensitivity of 100%, PPV 85% to
93% when done at 72 hours

MRI
o
o

most sensitive method to diagnose of occult fractures within 24


hours
allows immediate identification of fractures and ligamentous
injuries in addition to assessment of vascular status of bone
(vascularity of proximal pole)

CT scan with 1mm cuts


o less effective than bone scan and MRI to diagnose occult
fracture
o can be used to evaluate location of fracture, size of fragments,
extent of collapse, and progression of nonunion or union after
surgery
Treatment
Nonoperative
o thumb spica cast immobilization
indications
stable nondisplaced fracture (majority of fractures)
if patient has normal xrays but there is a high level
of suspicion can immobilize in thumb spica and
reevaluate in 12 to 21 days
technique
start immobilization early (nonunion rates increase
with delayed immobilization of > 4 weeks after
injury)
long arm spica vs short arm casting is
controversial
with no consensus
duration of casting depends on location of fracture
distal-waist for 3 months
mid-waist for 4 months
proximal third for 5 months
athletes should not return to play until
imaging shows a healed fracture
may opt to augment with pulsed electromagnetic
field (studies show beneficial in delayed union)
outcomes
scaphoid fractures with <1mm displacement have

union rate of 90%


Operative
o ORIF vs percutaneous screw fixation

indications
in unstable fractures as shown by
proximal pole fractures
displacement > 1 mm
15 scaphoid humpback deformity
radiolunate angle > 15 (DISI)
intrascaphoid angle of > 35
scaphoid fractures associated with perilunate
dislocation
comminuted fractures
unstable vertical or oblique fractures
in non-displaced waist fractures
to allow decreased time to union, faster
return to work/sport, similar total costs
compared to casting
outcomes
union rates of 90-95% with operative treatment of
scaphoid fractures
CT scan is helpful for evaluation of union
Technique

ORIF vs percutaneous screw fixation


o approach
dorsal approach
indicated in proximal pole fractures
care must be taken to preserve the blood supply
when entering the dorsal ridge by limiting exposure
to the proximal half of the scaphoid
percutaneous has higher risk of unrecognized
screw penetration of subchondral bone
volar approach
indicated in waist and distal pole fractures and
fractures with humpback flexion deformities
allows exposure of the entire scaphoid
uses the interval between the FCR and the radial
artery
arthroscopic assisted approach
has also been described

fixation
rigidity is optimized by long screw placed down the
central axis of the scaphoid
radial styloidectomy
should be performed if there is evidence of impaction
osteoarthritis between radial styloid and scaphoid

Complications
Scaphoid Nonunion
o treatment
inlay (Russe) bone graft
indications
if minimal deformity and there is no adjacent
carpal collapse or excessive flexion deformity
(humpback scaphoid)
outcomes
92% union rate

interposition (Fisk) bone graft


indications
if there is adjacent carpal collapse and
excessive flexion deformity (humpback
scaphoid)
technique
an opening wedge graft that is designed to
restore scaphoid length and angulation
outcomes
results show 72-95% union rates
vascular bone graft from radius
indications
gaining popularity and a good option for
proximal pole fractures with osteonecrosis
confirmed by MRI
technique
1-2 intercompartmental supraretinacular
artery (branch of radial artery) is harvested
to provide vascularized graft from dorsal
aspect of distal radius

SNAC wrist (scaphoid nonunion advanced collapse)

Qbank (10 Questions)


(OBQ09.36) A 22-year-old male snowboarder falls on an outstretched hand and presents with
the radiograph shown in Figure A. Which of the following techniques is MOST important in
optimizing biomechanical fixation? Review Topic
FIGURES: A
1.

Using a longer screw placed in the central axis of the scaphoid

2.

Using a supplementary K-wire transfixing the distal pole of the scaphoid to the capitate

3.

Using a longer screw placed in the dorsal axis of the scaphoid

4.

Using a larger diameter screw placed in the dorsal axis of the scaphoid

5.

Using a larger diameter screw placed in the volar axis of the scaphoid

PREFERRED RESPONSE 1
Several studies have shown a longer screw placed in the central axis of the scaphoid optimizes
biomechanical fixation of scaphoid waist fractures. Many studies have discussed the amount of
compression generated by various internal fixation screws (e.g headless vs. headed, variable
pitch, partially vs. fully threaded, cannulated vs. noncannulated), but it is believed that rigidity of
fixation is probably the most important factor in promoting healing of scaphoid fractures.
The first reference by McCallister et al is a cadaveric, biomechanical study that demonstrated a
centrally placed screw had 43% more stiffness than an eccentrically placed screw. They
recommend using surgical techniques that optimize central placement and screw length, such as
using a cannulated screw.
The study by Dodds et al supported these findings and added that a longer screw with 2mm of
bone coverage provided greater stability than a shorter screw. A more centrally placed screw is
generally longer and has more length of screw on each side of the fracture than does a
peripherally placed screw due to the anatomic dimensions of the scaphoid.
(OBQ09.56) An open dorsal approach for antegrade screw fixation of a nondisplaced scaphoid
waist fracture differs in which of the following ways compared to a percutaneous dorsal
approach? Review Topic
1.

Decreased risk of proximal pole AVN

2.

Increased risk of posterior interosseous nerve injury

3.

Decreased risk of injury to the APL tendon

4.

Increased risk of injury to the EPL tendon

5.

Decreased risk of screw prominence above subchondral bone

PREFERRED RESPONSE 5
Scaphoid screw fixation should be just below the subchondral bone; this is best judged by direct
visualization.
Adamany et al in an anatomic study using fluoroscopy to insert a scaphoid screw via a
percutaneous approach found that the scaphoid screw "was prominent (above the subchondral
bone) in 2 of 12 specimens and flush with or buried in the remaining 10 specimens." As a result,

they recommend using a limited dorsal incision to verify full seating of the screw. In addition, they
found the percutaneous approach was within 2.2-3.1 mm of the PIN, EDC, and EIP. Thus, all of
these structures are at increased risk of injury in a percutaneous approach. The APL tendon is
not in the surgical field. Illustration A shows the AIN(arrowhead) is deep in relation to pronator
quadratus. Sensory remnant of posterior interosseous nerve (straight thick arrow) is adjacent to
interosseous membrane. White arrow is median nerve. Shaded open arrow is ulnar nerve, and
long thin arrow is superficial radial nerve.
Tumilty et al inserted a Herbert screw through a dorsal approach in 12 cadaveric wrists. They
then imaged them with AP/Lateral xrays, and 360 degree fluoroscopic views. The wrists were
then dissected to evaluated for subchondral penetration, and plain x-ray films were accurate in 5
of 6 specimens. Fluoroscopy was accurate in all 6. They concluded that fluoroscopy during
placement of the Herbert screw may decrease the rate of subchondral penetration.

Lunate Dislocation (Perilunate


dissociation)
Author: Mark

Karadsheh

Topic updated on 12/24/14 11:44am

Introduction
High energy injury with poor functional outcomes
Commonly missed (~25%) on initial presentation
Categories
o perilunate dislocation
lunate stays in position while carpus dislocates
4 types
transcaphoid-perilunate
perilunate
transradial-styloid
transcaphoid-trans-capitate-perilunar

lunate dislocation
lunate forced volar or dorsal while carpus remains
aligned

Mechanism
o traumatic, high energy
o occurs when wrist extended and ulnarly deviated
leads to intercarpal supination
Pathoanatomy
o sequence of events
scapholunate ligament disrupted -->
disruption of capitolunate articulation -->
disruption of lunotriquetral articulation -->
failure of dorsal radiocarpal ligament -->
lunate rotates and dislocates, usually into carpal tunnel

dislocation can course through


greater arc

ligamentous disruptions with associated fractures


of the radius, ulnar, or carpal bones

lesser arc
purely ligamentous
Anatomy
Normal wrist anatomy

Osseous
o proximal row
scaphoid
lunate
triquetrum
pisiform
o distal row
trapezium
trapezoid
capitate
hamate
Ligaments
o interosseous ligaments
run between the carpal bones
scapholunate interosseous ligament

lunotriquetral interosseous ligament

major stabilizers of the proximal carpal row


intrinsic ligaments
ligaments the both originate and insert among the carpal bones
dorsal intrinsic ligaments
volar intrinsic ligaments
extrinsic ligaments
connect the forearm bones to the carpus
volar extrinsic carpal ligaments
dorsal extrinsic carpal ligaments

Classification

Mayfield Classification

Stage I

scapholunate dissociation

Stage II

+ lunocapitate disruption

Stage III

+ lunotriquetral disruption, "perilunate"

Stage IV

lunate dislocated from lunate fossa (usually volar)


associated with median nerve compression

Presentation
Symptoms
o acute wrist swelling and pain
o median nerve symptoms may occur in ~25% of patients
most common in Mayfield stage IV where the lunate
dislocates into the carpal tunnel
Imaging
Radiographs
o required views
PA/lateral wrist radiographs
o findings
AP

break in Gilula's arc


lunate and capitate overlap
lunate appears triangular "piece-of-pie sign"

lateral
loss of colinearity of radius, lunate, and capitate
SL angle >70 degrees

MRI

usually not required for diagnosis

Treatment
Nonoperative
o closed reduction and casting
indications
no indications when used as definitive
management
outcomes
universally poor functional outcomes with nonoperative management
recurrent dislocation is common
Operative
o emergent closed reduction/splinting followed by open
reduction, ligament repair, fixation, possible carpal tunnel
release
indications
all acute injuries <8 weeks old
outcomes
emergent closed reduction leads to
decreased risk of median nerve damage
decreased risk of cartilage damage
return to full function unlikely
decreased grip strength and stiffness are common
o proximal row carpectomy
indications
chronic injury (defined as >8 weeks after initial
injury)

not uncommon, as initial diagnosis frequently


missed
total wrist arthrodesis
indications
chronic injuries with degenerative changes

Techniques
Closed Reduction
o

technique
finger traps, elbow at 90 degrees of flexion
hand 5-10 lbs traction for 15 minutes
dorsal dislocations are reduced through wrist extension,
traction, and flexion of wrist
apply sugar tong splint
follow with surgery

Open reduction, ligament repair and fixation +/- carpal tunnel release

approach (controversial)
dorsal approach
longitudinal incision centered at Lister's tubercle
excellent exposure of proximal carpal row and midcarpal joints
does not allow for carpal tunnel release
volar approach
extended carpal tunnel incision just proximal to volar wrist
crease
combined dorsal/volar
pros
added exposure
easier reduction
access to distal scaphoid fractures
ability to repair volar ligaments
carpal tunnel decompression
cons
some believe volar ligament repair not necessary
increased swelling
potential carpal devascularization
difficulty regaining digital flexion and grip
o technique
fix associated fractures
repair scapholunate ligament
suture anchor fixation
protect scapholunate ligament repair
controversy of k-wire versus intraosseous cerclage wiring
repair of lunotriquetral interosseous ligament
decision to repair based on surgeon preference as no studies
have shown improved results
o post-op
short arm thumb spica splint converted to short arm cast at first postop visit
duration of casting varies, but at least 6 weeks
Proximal row carpectomy
o technique
perform via dorsal and volar incisions if median nerve compression is
present
volar approach allows median nerve decompression with excision of
lunate
dorsal approach facilitates excision of the scaphoid and triquetrum
o

Qbank (1 Questions)
(OBQ09.227) A 35-year-old professional football player complains of severe wrist pain after
making a tackle. He reports paresthesias in his thumb and index finger. AP and lateral
radiographs of the wrist are shown in figures A and B respectively. What is the most appropriate
next step in management? Review Topic
FIGURES: A B
1.

short arm thumb spica cast

2.

long arm thumb spica cast

3.

urgent closed reduction and splinting

4.

MR arthrogram of the wrist to assess ligamentous injuries

5.

bone scan to assess vascularity

Hook of Hamate Fracture


Author: Richard

Yoon

Topic updated on 03/03/16 2:48pm

Introduction
Epidemiology
o incidence
2% of carpal fractures
o risk factors
often seen in
golf
baseball
hockey
Pathophysiology
o typically caused by a direct blow
grounding a golf club
checking a baseball bat
Associated conditions
o bipartite hamate
will have smooth cortical surfaces
Anatomy
Hamate
o one of carpal bones, distal and radial to the pisiform

articulates with
fourth and fifth metacarpals
capitate
triquetrum
hook of hamate
forms part of Guyon's canal, which is formed by
roof - superficial palmar carpal ligament
floor - deep flexor retinaculum, hypothenar muscles
ulnar border - pisiform and pisohamate ligament
radial border - hook of hamate
one of the palpable attachments of the flexor retinaculum
deep branch of ulnar nerve lies under the hook

Presentation

Symptoms
o hypothenar pain
o pain with activities requiring tight grip
Physical examination
o provocative maneuvers
tender to palpation over the hook of hamate

o
o

hook of hamate pull test:


hand held in ulnar deviation as patient flexes DIP
joints of the ulnar 2 digits, the flexor tendons act as
a deforming force on the fracture site, positive test
elicits pain
motion and strength
decreased grip strength
neurovascular exam
chronic cases
parasthesia in ring and small finger
motor weakness in intrinsics

Imaging
Radiographs
o recommended views
AP and carpal tunnel view

findings
fracture best seen on carpal tunnel view

CT
o

Treatment

indications
establish diagnosis if radiographs are negative

Nonoperative
o immobilization 6 weeks
indications
acute hook of hamate fractures
body of hamate fx (rare)
Operative
o

excision of hamate fracture fragment


indications
chronic hook of hamate fxs with non-union

ORIF

indications
ORIF is possible but has little benefit

Complications

Non-union
Scar sensitivity
Iatrogenic injury to ulnar nerve
Closed rupture of the flexor tendons to the small finger
Qbank (5 Questions)
(SBQ07.40) A 44-year-old man presents with ulnar-sided right wrist pain and mild constant
tingling in the fourth and fifth digits after injuring his wrist while playing golf. Although pain and
function have improved with conservative treatment 6 months following the injury, he still reports
difficulty with his golf game. Which of the following should initially be obtained in this patient to
aide in the diagnosis? Review Topic
1.

Bone scan of the wrist and hand

2.

EMG study of the affected extremity

3.

Carpal tunnel view radiograph

4.

CT scan of the distal forearm and wrist

5.

Contrast enhanced magnetic resonance angiogram

PREFERRED RESPONSE 3
This patients clinical presentation is most consistent with a chronic hook of the hamate fracture,
which should initially be evaluated with a carpal tunnel view radiograph. Hook of the hamate
fractures typically are associated with pain localized to the hypothenar eminence, and chronic
cases can be associated with neuropathy of the ulnar nerve. Excision of the hook through the
fracture site usually yields satisfactory results in the presence of chronic injuries.
Parker et al treated five patients with six hook of the hamate fractures over an eight year period.
All patients ultimately underwent hook resection and returned to their previous level of activity in
6 to 8 weeks after surgery without loss of function. Based on their case series, they concluded
that the entire hook should be resected to the base of the hamate as the primary form of
treatment in hook of the hamate fractures.

Illustration A: Patient positioning for carpal tunnel radiograph-wrist is extended 70 degrees, and
beam is angled 25-30 deg to the long axis of the hand(arrow).
Illustration B: Carpal tunnel view radiograph demonstrates a fracture at the base of the hook of
the hamate(black arrow) and normal pisotriquetral joint space.
Incorrect Answers:
1-Bone scans are not typically indicated in the diganostic setting of acute or chronic hook of the
hamate fractures.
2-Imaging should be obtained to rule out bony injury prior to obtaining an EMG study.
4-CT scans can used to confirm the diagnosis of a hook of the hamate fracture after obtaining a
carpal tunnel view radiograph.
5-Contrast enhanged MRA of the wrist is typically used to diagnose hypothenar hammer
syndrome or other vascular abnormalities.

(OBQ11.130) A 24-year-old racquetball player presents after accidentally striking his racket
against the wall during a match two months ago. He is tender to palpation over the hypothenar
mass, and his pain is aggrevated by grasping. A radiograph and CT scan of his wrist are shown
in Figures A and B. Which of the following treatment methods has been definitively shown in the
literature to have a favorable outcome, and a high chance to return to pre-injury activities in
patients with this injury? Review Topic
FIGURES: A B
1.

Activity restriction and continued monitoring

2.

Open reduction and internal fixation

3.

Casting for 6 weeks, followed by physical therapy

4.

Corticosteroid injection and immediate return to play

5.

Surgical excision

PREFERRED RESPONSE 5
The patients history and imaging are consistent with a subacute hook of the hamate fracture.
This is demonstrated by the carpal tunnel view radiograph in Figure A, and confirmed by the CT
scan of the wrist in Figure B. CT scan of the wrist is usually indicated to definitively diagnose
these fractures. Current literature supports the most favorable results and ability to return to preinjury activities with excision of the fracture fragment. There is little available literature reporting
the results of open reduction and internal fixation of these fractures.
Rettig et al review traumatic wrist injuries in athletes. With regards to treatment of hook of the
hamate fractures, they state that ORIF and excision are the two viable treatment options in
athletes. Of these, the literature supports fragment excision, which has an average return to sport
time of 7-10 weeks.

Welling et al determined which wrist fractures are not diagnosed with initial radiography, using
CT as a gold standard and identified specific fracture patterns. In their series, they found that
only 40% of hamate fractures were diagnosed on plain radiography, suggesting that CT should
be considered after a negative radiographic finding if clinically warranted.

Hamate Body Fracture


Author: Evan

Watts

Topic updated on 11/26/14 5:05pm

Introduction

A rare carpal fracture


Epidemiology
o incidence
<2% of all carpal fractures

Pathophysiology
o mechanism of injury
main cause for these lesions is a direct impact against a
hard surface with a clenched fist
Associated conditions
o may be associated with 4th or 5th metacarpal base fractures or
dislocations
present in ~ 15%

Anatomy
Hamate Bone
o osteology
triangular shaped carpal bone
composed of hook and body

o
o

location
most ulnar bone in the distal carpal row
articulation
4th and 5th metacarpals
capitate
triquetrum

Classification

Milch Classification of Hamate Fractures


Milch Classification
Type I

Hook of Hamate Fx (most common)

Type II

Body of Hamate Fx

Presentation
Symptoms
o ulnar-sided wrist pain and swelling
Physical exam
o inspection
focal tenderness over hamate
Imaging
Radiographs
o recommended views
oblique radiographs (30) are usually required to visualize
fracture

AP and lateral radiographs are less reliable


additional views
carpal tunnel view radiographs

CT
o

usually required to delineate fracture pattern and determine


operative plan

Treatment
Nonoperative
o immobilization
indications
rarely may be used for extra-articular nondisplaced
fracture
Operative
o ORIF
indications

most fracture are intra-articular and require open


reduction
technique
interfragmentary screws +/- k-wires for temporary
stabilization
Surgical Techniques
Open Reduction Internal Fixation
o approach
dorsal most common approach

o
o

fixation technique
fixation may be obtained with K wires or screws
postoperative care
immobilize for 6-8 weeks

Complications
Stiffness
Malunion
Infection

Pisiform Fracture
Author: Evan

Watts

Topic updated on 01/26/15 7:25pm

Introduction
A rare carpal fracture
Epidemiology
o incidence
<1% of carpal fractures

rare injury and often missed


Pathophysiology
o mechanism of injury
usually occurs by direct impact against a hard surface
fall on outstretched hand
Associated conditions
o 50% occur as isolated injuries
o 50% occur in association with other carpal fractures or distal
radius fractures
Anatomy
Pisiform Bone
o osteology
pea shaped, seasmoid bone
o location
most ulnar and palmar carpal bone in proximal row
located within the FCU tendon
o function
contributes to the stability of the ulnar column by
preventing triquetral subluxation
Presentation
Symptoms
o ulnar sided wrist pain after a fall
o grip weakness
Physical exam
o inspection
hypothenar tenderness and swelling
rule out associated injury to other carpal bones and distal

radius
Imaging
Radiographs
o recommended views
AP and lateral views of wrist

additional views
pronated oblique and supinated oblque views
carpal tunnel view
findings
best seen with 30 deg of wrist supination or utilizing the
carpal tunnel view

CT
o

indications
may be required to delineate fracture pattern and
determine treatment plan

MRI
o
o

indications
suspected carpal fracture with negative radiographs
findings
may show bone marrow edema within the pisiform
indicating fracture

Treatment
Nonoperative
o early immobilization
indications
first line of treatment
technique

short arm cast with 30 degrees of wrist flexion and


ulnar deviation for 6-8 weeks
outcomes
most often go on to heal without posttraumatic
osteoarthritis
Operative
o pisiformectomy
indications
severely displaced and symptomatic fractures
painful nonunion

outcomes
studies show a pisiformectomy is a reliable way to
relieve this pain and does not impair wrist function
Complications
Malunion
Non-union
Chronic ulnar sided pain
Decreased grip strength
Qbank (1 Questions)
(OBQ07.102) A 28-year-old man fell while ice skating 6 months ago and has had ulnar-sided
wrist pain ever since. The patient's lateral radiograph of the wrist is shown in Figure A and a CT
scan is shown in Figure B. What is the most appropriate treatment? Review Topic
FIGURES: A B
1.

Scapholunate ligament repair

2.

Excision of the hook hamate

3.

Excision of the pisiform

4.

Open reduction internal fixation of the hamate

5.

Open reduction internal fixation of the pisiform

PREFERRED RESPONSE 3
Based on clinical history and imaging shown, this patient has developed a pisiform fracture
nonunion. Treatment of symptomatic nonunions of the pisiform is by pisiformectomy
Fractures of the pisiform are rare. They often occur in conjunction with injuries to the distal radius
or carpus. Non-operative management with cast immobilization in 30 degrees of wrist flexion is
the first line of treatment. Symptomatic nonunions are treated with pisiformectomy.
Palmieri et al. performed pisiformectomies on 21 patients who had pisiform area pain that was
refractory to conservative management. Patients had a history of painful union or nonunion of
pisiform fractures, arthritis or FCU tendonitis. In all cases, wrist strength and mobility was
retained.
Lam et al. reviewed the effect of pisiform excision on wrist function in patients with piso-triquetral
dysfunction. After an average follow up of 65 months, 75% of patients had complete relief of
pisiform area symptoms. No differences in grip, wrist motion, strength or power were found in
comparison to the contralateral side.
Figure A shows a lateral radiograph of a pisiform fracture nonunion. Figure B shows an axial CT
scan sequence of the wrist. A pisiform fracture nonunion is identified with subtle comminution.
The pisotriquetral joint appears to be congruent.
Incorrect Answers
Answer 1: The scapholunate ligament is not affected in this clinical situation.
Answers 2, 4: Although the hook of hamate can be a source of ulnar sided pain, it is not

implicated in this clinical situation


Answer 5: An ORIF of the pisiform is not typically used for symptomatic pisiform fracture
nonunions

Seymour Fracture
Author: Amiethab

Aiyer

Topic updated on 03/11/16 2:30pm

Seymour Fracture
Definition
o juxta-epiphyseal fractures of the terminal phalanx
includes Salter Harris I or II or metaphyseal fractures 1-2mm from the
epiphyseal plate
o often includes nailbed laceration, nail plate subluxation, interposition of soft
tissue at fracture site (usually germinal matrix)
Epidemiology
o body location
middle finger most common
Pathophysiology
o mechanism of injury
caught in door
struck by baseball (similar mechanism to mallet finger)
o pathoanatomy
weakness of epiphysiometaphyseal junction
angulation of the diaphysis on the epiphysis
direction of the blow
different insertion sites of flexor and extensor tendons
extensor tendon and volar plate insert into epiphysis
flexor tenon inserts into metaphysis
Presentation
Physical exam
o apparent mallet deformity
o echymosis and swelling
o base of nail plate elevated above eponychial fold in open injuries
Radiographs
may appear normal on posteroanterior view
lateral view
o widened physis or displacement between epiphysis/metaphysis
o flexion deformity at fracture site
Treatment
stable, closed injuries
o closed reduction and splinting
unstable, closed injuries
o closed reduction and pinning across DIPJ
open injuries
o characterized by nail fold lateration, skin laceration proximal to nail fold,
elevation of nail plate superficial to eponychial fold

antibiotics, nail plate elevation and removal, nail bed repair, pinning (if
unstable), nail plate fixation

Finger Trauma

Metacarpal Fractures
Author: Joshua

Blomberg

Topic updated on 06/02/15 10:35am

Introduction
Metacarpal fractures
o divided into fractures of metacarpal head, neck, shaft
o treatment based on which metacarpal is involved and location
of fracture
o acceptable angulation varies by location
o no degree of malrotation is acceptable
Epidemiology
o incidence
metacarpal fractures account for 40% of all hand injuries
o demographics
men aged 10-29 have highest incidence of metacarpal
injuries
o

location

metacarpal neck is most common site of fracture


fifth metacarpal is most commonly injured
Mechanism of injury
o direct blow to hand or rotational injury with axial load
o high energy injuries (ie. automobile) may result in multiple
fractures
Associated conditions
o wounds may indicate open fractures or concomitant soft tissue
injury
tendon laceration
neurovascular injury
o compartment syndrome
closed injuries with multiple fractures or dislocations
crush injuries
Anatomy
Metacarpal anatomy

concave on palmar surface


o 1st, 4th, and 5th digits form mobile borders
o 2nd and 3rd digits form stiffer central pillar
index metacarpal is the most firmly fixed, while the thumb
metacarpal articulates with the trapezium and acts
independently from the others
o three palmar and four dorsal interossei muscles arise from
metacarpal shafts
Insertional anatomy
o extensor carpi radialis longus/brevis
insert on the base of metacarpal II, III (respectively);
assist with wrist extension and radial flexion of the wrist
o

extensor carpi ulnaris

abductor pollicis longus

opponens pollicis

opponens digiti minimi

inserts on the base of metacarpal V; extends and fixes


wrist when digits are being flexed; assists with ulnar
flexion of wrist
inserts on the trapezium and base of metacarpal I;
abducts thumb in frontal plane; extends thumb at
carpometacarpal joint
inserts on metacarpal I; flexes metacarpal I to oppose the
thumb to the fingertips
inserts on the medial surface of metacarpal V; Flexes
metacarpal V at carpometacarpal joint when little finger is
moved into opposition with tip of thumb; deepens palm of
hand.

Presentation
Physical exam
o inspect for open wounds and associated injuries
fight wounds over MCP joint are open until proven otherwise
extensor tendon can be lacerated and retracted
dorsal wounds over metacarpal fractures are almost always open
fractures
o deformity indicates location
deformity at metacarpal base may indicate CMC dislocation
shortening can be assessed by comparing contralateral hand
malrotation assessed by lining up fingernail in partial flexion and full
flexion if possible, compare to contralateral side
o motor examination
typically no motor deficits unless open wounds present
check integrity of flexor/extensor tendons in presence of open wounds
o neurovascular examination
dorsal wounds may affect dorsal sensory branch of radial/ulnar nerve
volar wounds can involve digital nerves

test for radial and ulnar border two-point discrimination on the injured
digit before any regional/hematoma block or attempted reduction

Imaging
Radiographs
o standard AP, oblique, and lateral films
o
oblique radiographs
for evaluation of CMC joint and improved visualization of
affected digit
30pronated lateral
to see 4th and 5th CMC fx/dislocation
30supinated view
to see 2nd and 3rd CMC fx/dislocation
o Brewerton view for metacarpal head fractures
o Roberts view for thumb CMC joint
CT
o indications
inconclusive radiographs of CMC fractures/dislocations
multiple CMC dislocations
complex metacarpal head fractures
General Treatment
Nonoperative
o immobilization
indications
must be stable pattern
no rotational deformity
acceptable angulation & shortening (see table)
Acceptable Shaft
Acceptable Shaft
Angulation (degrees) Shortening (mm)
Index & Long Finger

Acceptable neck
Angulation

10-20

2-5

10-15

Ring Finger

30

2-5

30-40

Little Finger

40

2-5

50-60

Operative
o operative treatment
general indications
intra-articular fxs

rotational malalignment of digit


significantly displaced fractures (see above criteria)
multiple metacarpal shaft fractures
loss inherent stability from border digit during
healing process
postoperative
early motion is critical
remove pins/ cast at ~ 4 weeks
Treatment - Metacarpal Head Fractures

Operative
o ORIF

indications
no degree of articular displacement acceptable
majority requires surgical fixation
external fixation
indications
severely comminuted fractures
MCP arthroplasty
indications
severely comminuted fractures
MCP fusion
indications
arthritis late disease

Techniques
o ORIF

approach
dorsal incision
either centrally split extensor apparatus or release and repair
sagittal band
fixation
hardware cannot protrude from joint surface
fix with multiple small screws in collateral recess, headless
screws, or k-wires
ideal fixation should allow for early motion

Complications
o stiffness
most common
prevented with early motion

Treatment - Metacarpal Shaft Fractures


Nonoperative
o immobilization
indications
nondisplaced metacarpal neck fractures
acceptable angulation (see above table)
no malrotation

shortening (aesthetic problem only)


immobilize MCP joints in 70-90 degrees of flexion
cast for 4 weeks
Operative
o ORIF vs. CRPP
indications
open fractures
unacceptable angulation (see above table)
any malrotation
multiple fractures
Techniques
o closed reduction percutaneous pinning
place antegrade through metacarpal base or retrograde
through collateral recess
remove pins at 4 weeks
o open reductions with lag screw
can use multiple lag screws for long spiral fractures
try to get at least two lag screws
o open reduction with dorsal plating
works best for transverse fractures
try to cover plate with periosteum to prevent tendon
irritation
begin early motion to prevent tendon irritations
Treatment - Metacarpal Neck Fractures
Nonoperative
o reduction and casting
acceptable degrees of apex dorsal angulation (varies by
study, see above table)
immobilize MCP joints in 70-90 degrees of flexion, leave
PIP joints free
cast for 4 weeks
reduce using Jahss technique
90 degrees MCP flexion, dorsal pressure through
proximal phalanx while stabilizing metacarpal shaft
Operative
o reduction and fixation
indications
unacceptable angulation (see above table)
open fractures
any malrotation
intraarticular fractures
Technique
o CRPP with MCP's flexed

antegrade through metacarpal base


retrograde through collateral recess
o

ORIF
perform if cannot get reduction for CRPP
difficult to plate because limited bone for distal fixation

MCP Dislocations
Author: Joshua

Blomberg

Topic updated on 12/19/14 3:54pm

Introduction

Epidemiology
o dorsal dislocations most common
o index finger most commonly involved
Mechanism
o a hyperextension injury
Classification
Simple vs. Complex

simple
volar plate not interposed in joint
treated with closed reduction
o complex
complex dislocations have interposition of volar plate
and/or sesamoids
in index finger flexor tendon displaces ulnarly and
lumbrical displaces radially which tighten around
metacarpal neck preventing reduction
in small finger flexor tendons and lumbrical
displace radially and the abductor digiti minimi and
flexor digiti minimi ulnarly preventing closed
reduction
may require open reduction
Kaplan's lesion (rare)
o most common in index finger
o complex dorsal dislocation of finger, irreducible
o metacarpal head buttonholes into palm (volarly)
o volar plate is interposed between base of proximal phalanx and
metacarpal head
o

Presentation

Physical exam
o skin dimpling often seen in complex dislocations but absent in
simple dislocations
Imaging
Radiographs
o lateral view best shows dislocation
o joint space widening may indicate interposition of volar plate
o useful to detect associated chip fractures
Treatment
Nonoperative
o closed reduction
indications
simple dislocations
technique
reduction technique involve applying direct
pressure over proximal phalanx while the wrist is
held in flexion to take tension off the intrinsic and
extrinsic flexors
avoid longitudinal traction and hyperextension
during closed reduction, may pull volar plate into
joint
Operative
o open reduction
indications
complex dislocations
Surgical Techniques
Open reduction
o approach

dorsal approach
split extensor tendon to expose joint
may be able to push volar plate out with freer
elevator
usually need to split volar plate to remove from
joint
use this approach for volar dislocations
volar approach
places neurovascular structures at risk
release A1 pulley to expose volar plate

Phalanx Dislocations
Author: Joshua

Blomberg

Topic updated on 02/05/16 3:18pm

Introduction
Common hand injuries can be broken into the following
o

PIP joint
dorsal dislocations
dorsal fracture-dislocations
volar dislocation
volar fracture-dislocation
rotatory dislocations
DIP joint
dorsal dislocations & fracture-dislocations

Associated conditions
o
o

swan neck deformity


nail bed injuries
associated with distal phalanx fractures

Imaging
Radiographs
o
o

finger xrays
must get true lateral of joint
hand xrays to rule out associated fractures
30pronated lateral to see 4th and 5th CMC x/dislocation
30supinated view to see 2nd and 3rd CMC fx/dislocation

Dorsal PIP Dislocations


Introduction
o more common than volar dislocation
o leads to injury to the volar plate and at least one collateral
ligament, and if untreated a swan neck deformity will result
Classification
o simple
middle phalanx in contact with condyles of proximal
phalanx
o complex
base of middle phalanx not in contact with condyle of
proximal phalanx, bayonet appearance

volar plate acts as block to reduction with longitudinal


traction
Treatment
o nonoperative
reduce and buddy tape to adjacent finger (3-6 weeks)
indications
dislocation is reducible
usually performed by patient
technique
if complex, reduce with hyperextension of
middle phalanx followed by palmar force
complications
a PIP flexion contracture
(pseudoboutonniere)
may develop but usually resolves with
therapy
swan neck deformity
occurs secondary to a volar plate injury
o operative
open reduction and extraction of the volar plate
indication
failed reduction
technique
in closed injuries incomplete reduction
usually due to volar plate interposition
in open injuries incomplete reduction usually
caused by dislocated FDP tendon
perform dorsal approach with incision
between central slip and lateral band
Dorsal PIP Fracture-Dislocations

Classification
o
o

Hastings classification (based on amount of P2 articular


surface involvement)
volar lip fractures are the most common fracture pattern
Type I-Stable
<30%-treat with dorsally based extension block
splint
Type II-Tenuous
30-50%-if reducible in flexion, dorsally based
extension block splint
Type III-Unstable
>50%-ORIF, hamate autograft, or volar plate

arthroplasty
Treatment
o nonoperative
dorsal extension block splinting
indications
if < 40% joint involved and stable
outcome
regardless of treatment, must achieve
adequate joint reduction for favorable longterm outcome
o operative
ORIF or CRPP
indications
if > 40% joint involved and unstable
technique
reduction of the middle phalanx on the
condyles of the proximal phalanx is the
primary goal
adequate volar exposure of the volar plate
requires resection of
proximal portion of C2 pulley
entire A3 pulley
distal C1 pulley
outcomes
articular surface reconstruction is desirable,
but not necessary for a good clinical outcome
PIP subluxation inhibits the gliding arc of the
joint and portends a poor clinical outcome
dynamic distraction external fixation
indications
highly comminuted "pilon" fracturedislocations
technique
follow with early mobilization
volar plate arthroplasty
indications
chronic injuries
arthrodesis
indications
chronic injuries
Volar PIP Dislocation & Fracture-dislocations

Introduction
o less common than dorsal dislocation
o leads to an injury to the central slip and at least one collateral
ligament, and a failure to treat will lead
to boutonneire deformity
Treatment
o dislocation only
nonoperative
splinting in extension for 6-8 weeks
indications
most PIP dislocations
o fracture-dislocation
nonoperative
splinting in extension for 6-8 weeks
indications
if < 40% joint involved and stable
operative
ORIF or CRPP
reduction of the middle phalanx on the
condyles of the proximal phalanx is the
primary goal
if > 40% joint involvement
Rotatory PIP dislocation

Introduction
o one of phalangeal condyles is buttonholed between central slip
and lateral band
Treatment
o nonoperative
only if reduction is successful
reduce by applying traction to finger with MP and PIP
joints in 90 degrees of flexion
flexion relaxes volarly displaced lateral band,
allowing it to slip back dorsally
reduction is confirmed with post-reduction true
lateral radiograph
o operative
open reduction
indications
required in most cases
Dorsal DIP Dislocations & Fracture-Dislocations
Treatment
o nonoperative

closed reduction, immobilization in slight flexion


with a dorsal splint for 2 weeks
indications
first line of treatment
tuft fractures require no specific treatment
can consider temporary splinting
operative
open reduction
indications
if two reduction attempts fail
technique
volar plate interposition is most common
block to reduction in irreducible closed DIP
joint dislocation
FDP may be blocking reduction if injury is
open
may require percutaneous pinning to support
nail bed repair
amputation
consider in highly comminuted injuries with
significant soft tissue loss

Qbank (8 Questions)
(OBQ11.63) A 39-year-old male sustained an index finger injury 6 months ago and has failed
eight weeks of splinting. A radiograph taken at the time of injury is shown in Figure A, and a
current radiograph is shown in Figure B. Which of the following is true regarding open reduction
and screw fixation of this injury? Review Topic
FIGURES: A B

1.

High risk of symptomatic implant

2.

Immobilization of the distal interphalangeal joint is required for 2 weeks post-operatively

3.

High rates of post-operative infection are common

4.

Open reduction via an approach through the nail bed leads to significant post-operative
nail deformity

5.

Range of motion of the DIP joint in the affected finger is usually less than 10 degrees
post-operatively

PREFERRED RESPONSE 1
Open reduction and internal fixation of distal phalanx fracture non-unions frequently requires the
post-operative removal of the fixation implant after complete fracture healing.
Chim et al followed 14 patients with non-union of fractures of the shaft of the distal phalanx who

were treated with open reduction and screw fixation. The implants required removal in 13/14
patients, and the mean post-operative range of motion of the DIP joints was 56 degrees. No
immobilization was required postoperatively, and bone grafting was only necessary in two
patients with severely comminuted fractures. Finally, the authors recommended approaching the
fracture through the nailbed for the best exposure, and found no postoperative nail growth
complications. Postoperative infections were not common in their series.
Mejis et al describe two patients with non-unions of the thumb distal phalanx treated with a single
compression screw using a minimally invasive approach. Both patients healed their fractures
using this technique.

Phalanx Fractures

Introduction
proximal phalanx
middle phalanx
distal phalanx

incidence
o most common injuries to the skeletal system
o account for 10% of all fractures
o distal phalanx is most common fractured bone in the hand

mechanism
o depends on age
10-29 years of age: sports is most common
40-69 year of age: machinery is most common
>70 year of age: falls are most common
pathoanatomy
o proximal phalanx fx
deformity is usually apex volar angulation due to
proximal fragment in flexion (from interossei)
distal fragment in extension (from central slip)
o middle phalanx
deformity is usually apex dorsal OR volar angulation
apex dorsal if fracture proximal to FDS insertion (from
extension of proximal fragment through pull of the central slip)
apex volar if fracture distal to FDS insertion (prolonged
insertion from just distal to the flare at the base to within a few
mm of the neck)

a fracture through the middle third may angulate in either


direction or not at all secondary to the inherent stability
provided by an intact and prolonged FDS insertion

nail bed injuries


o associated with distal phalanx fractures

Presentation
Symptoms
o pain
Physical exam
o local tenderness
o deformity
o look carefully for open wounds

Imaging
Radiographs
finger xrays
o must get true lateral of joint
hand xrays to rule out associated fractures
o 30pronated lateral to see 4th and 5th CMC x/dislocation
o 30supinated view to see 2nd and 3rd CMC fx/dislocation

Treatment - Proximal Phalanx Fracture


Nonoperative
buddy taping
o indications
extraarticular with < 10 angulation or < 2mm shortening and no
rotational deformity
o 3 weeks of immobilization followed by aggressive motion
reduction and splinting
o indications
most distal phalanx fx
Operative
CRPP vs. ORIF
o indications
irreducible or unstable fracture pattern

transverse fractures (all angulate volarly) with > 10 angulation or


2mm shortening or rotationally deformed
long oblique proximal phalanx fractures
techniques
crossed k-wires
Eaton-Belsky pinning through metacarpal head
minifragment fixation with plate and lag screws, or lag screws alone
lag screws alone indicated in presence of long oblique fracture

Treatment - Middle Phalanx Fracture

Nonoperative
o buddy taping
indications
extraarticular with < 10 angulation or < 2mm shortening and no
rotational deformity
technique
3 weeks of immobilization followed by aggressive motion
Operative
o CRPP vs. ORIF
indications
irreducible or unstable fracture pattern
transverse fractures with > 10 angulation or 2mm shortening or
rotationally deformed
techniques
crossed k-wires
collateral recess pinning
minifragment fixation with plate and lag screws

Treatment - Distal Phalanx Fracture

Nonoperative
reduction and splinting

indications
most cases
nail matrix may be incarcerated in fx and block reduction

Operative
o remove nail, repair nailbed, and replace nail to maintain epi fold
indications
when distal phalanx associated with a nailbed injury
see nail bed injuries
o ORIF +/- bone grafting
indications
non-unions

Complication
Loss of motion
o most common complication
o predisposing factors include prolonged immobilization, associated joint injury,
and extensive surgical dissection
o treat with rehab, and surgical release as a last resort
Malunion
o malrotation, angulation, shortening
o surgery indicated when associated with functional impairment
corrective osteotomy at malunion site (preferred)
metacarpal osteotomy (limited degree of correction)
Nonunion
o uncommon
o most are atrophic and associated with bone loss or neurovascular compromise
o surgical options
resection, bone grafting, plating
ray amputation or fusion

Question
A 34-year-old male sustains the closed finger injury shown in Figure A one week ago. He
undergoes closed reduction and pinning shown in Figure B to correct alignment. Which of
the following is responsible for the apex palmar fracture deformity noted on the preoperative
radiographs?
Figure

B.

A.

1.

Indirect pull of the central slip on the distal fragment and the interossei insertions at
the base of the proximal phalanx

2.

Intrinsic muscle fibrosis and intrinsic minus contracture

3.

PIP joint volar plate attenuation and extensor tendon disruption

4.

Rupture of the central slip with attenuation of the triangular ligament and palmar
migration of the lateral bands

5.

Flexor tendon disruption with associated overpull of the extensor mechanism

1.
Indirect pull of the central slip on the distal fragment and the interossei insertions at
the base of the proximal phalanx
82% (2484/3028)

2.

Intrinsic muscle fibrosis and intrinsic minus contracture

1% (45/3028)

3.

PIP joint volar plate attenuation and extensor tendon disruption

2% (49/3028)

4.
Rupture of the central slip with attenuation of the triangular ligament and palmar
migration of the lateral bands
9% (274/3028)

5.

Flexor tendon disruption with associated overpull of the extensor mechanism

5% (148/3028)

Preferred Respone
The clinical presentation is consistent with a transverse proximal phalanx fracture. These
fracture have an apex palmar angulated deformity under the indirect pull of the central slip on
the distal fragment and the interossei insertions at the base of the proximal phalanx.
If proximal phalanx fractures are allowed to heal with the apex palmar deformity, an extensor
lag will result. Therefore CRPP or ORIF is indicated in transverse fractures with > 10
angulation. To correct this deformity prior to surgical fixation, the MCP joint should be
flexed, which allows the extensor mechanism as a whole to function as a tension band to help
reduce the fracture. This is referred to as intrinsic plus splinting. Collateral ligament, capsule,

and intrinsic muscle attachments render transverse fractures in the proximal 6 to 9 mm of the
P1 base more stable than fractures located distally.
Henry provides a review of fractures of the proximal phalanx and metacarpals. He states that
most transverse or short oblique P1 fractures without comminution are best stabilized by two
0.045-inch K-wires placed longitudinally through the fully flexed MCP joint. A single wire
alone risks rotational malunion, but some fracture patterns may provide inherent rotational
stability that would allow use of one wire for angular control.
Figure A shows a transverse fracture of the proximal phalanx with apex volar angulation.
Figure B shows two K-wires placed transarticular through the MCP joint in a flexed (intrinsic
plus) posture to correct the deformity and stabilize the fracture.
Incorrect Answers:
Answer 2: Intrinsic muscle fibrosis and contracture is usually associated with chronic crush
injuries and significant soft tissue damage.
Answer 3: This is describing a swan neck deformity.
Answer 4: This is describing a Boutonnierre deformity.
Answer 5: Flexor tendon disruption is not likely in this closed injury pattern.

Digital Collateral Ligament Injury

Introduction
Trauma to the digit injuring the radial or ulnar collateral ligaments
Mechanism
o depends on the joint involved
o usually the result of a "jammed finger"
o doral or volar dislocation events can tear one or both of the collateral ligaments

Anatomy

Collateral ligaments of the digits


o located on the lateral aspect of the DIP, PIP and MCP joints

crucial for opposing pinch stability

Presentation

Symptoms
o Pain at involved joint
o Instabilty with pinch once pain resolved
Physical exam
o inspection
swelling at involved joint
deformity of joint
o provocative tests
varus and valgus stress tests

Imaging
Radiographs
o recommended views
AP, lateral, and oblique views of digit
varus/valgus stress views may aid in diagnosis
MRI
o indicated if equivocal physical exam findings

Treatment
Nonoperative
o buddy taping for 3 weeks
indications
simple tears
o buddy taping for 6 weeks
indications
complete tears
Operative
o collateral ligament repair
indications
radial ligament injuries of index finger (ligament needed for
pinch stability)

Nail Bed Injury

Introduction
Nail bed injuries are the result of direct trauma to the fingertip. Injury types include
subungual hematoma (details below)
nail bed laceration
nail bed avulsion
Epidemiology
nail bed injuries are included under the umbrella of fingertip injuries
o finger tip injuries are the most common hand injuries seen in the hospital
emergency department
Pathophysiology
mechanisms of injury include
o crushing fingertip between two objects
o catching finger in a closing door
o saw injury
o snowblower injury
o direct blow from a hammer
Associated conditions
DIP fractures or dislocations
Prognosis
early treatment of acute injuries results in the best outcomes with minimal morbidity

Anatomy

Nailbed and surrounding tissue

perionychium
o nail
o nailbed
o surrounding skin
paronychium
o lateral nail folds
hyponychium
o skin distal distal and palmar to the nail
eponychium
o dorsal nail fold
o proximal to nail fold
lunula
o white part of the proximal nail
matrix
o sterile
soft tissue deep to nail
distal to lunula
adheres to nail
o germinal
soft tissue deep to nail
proximal to sterile matrix
responsible for most of nail development
insertion of extensor tendon is approximately 1.2 to 1.4 mm proximal
to germinal matrix

Presentation
Symptoms
o pain
Physical exam
o examine for subungual hematoma
o inspect nail integrity

Imaging
Radiographs
o recommended
AP, lateral and oblique of finger
to rule out fracture of distal phalanx

Subungual Hematoma

Most commonly caused by a crushing-type injury


causes bleeding beneath nail
Treatment
drainage of hematoma by perforation

indications
less than 50% of nail involved
o techniques
puncture nail using sterile needle
electrocautery to perforate nail
nail removal, D&I, nail bed repair
o indications
> 50 % nail involved
o technique
nail bed repair (see techniques)
o

Nail Bed Lacerations

Laceration of the nail and underlying nail bed


usually present with the nail intact and a subungual hematoma greater than 50% of
nail surface area
Treatment
nail removal with D&I, nail bed repair
o indications
most cases
o modalities
tetanus and antibiotic prophylaxis

Avulsion Injury

Avulsion of nail and portion of underlying nail bed

Mechanism
usually caused by higher energy injuries
Associated conditions
commonly associated with other injuries including
o distal phalanx fracture
if present reduction is advocated
Treatment
nail removal, nail bed repair, +/- fx fixation
o indications

avulsion injury with minimal or no loss of nail matrix, with or without


fracture

technique
always give tetanus and antibiotics
fracture fixation depends on fracture type
nail removal, nail bed repair, split thickness graft vs. nail matrix transfer, +/- fx
fixation
o indications
avulsion or crush injury with significant loss of nail matrix
o technique
always give tetanus and antibiotics
nail matrix transfer from adjacent injured finger or nail matrix transfer
from second toe
fracture fixation depends on fracture type
o

Techniques
Nail bed repair
o nail removal
soak nail in Betadine while repairing nail bed
o nail bed repair
repair nail bed with 6-0 or smaller absorbable suture
RCT has demonstrated quicker repair time using 2-octylcyanoacrylate
(Dermabond) instead of suture with comparable cosmetic and
functional results
o splint eponychial fold
splint eponychial fold with original nail, aluminum, or non-adherent
gauze

Complications
Hook nail

A.

caused by advancement of the matrix to obtain coverage without adequate bony


support
A. Treatment

remove nail and trim matrix to level of bone


Split nail

caused by scarring of the matrix following injury to nail bed


o Treatment
excise scar tissue and replace nail matrix
graft may be needed

Questions
A 7-year-old boy sustains a ring finger injury after falling from his bike. The fingernail has
been torn transversely beneath the eponychium and the surgeon has removed the nail as
shown in Figure A. Radiographs are shown in Figure B. What is the next best step in
management?
Figures :

A.

B.

1.

Irrigation and debridement with alumafoam placement and immobilization

2.

Irrigation and debridement followed by percutaneous pinning and immobilization

3.

Irrigation and debridement followed by reduction, nail bed repair and immobilization

4.

Betadine soaks at home three times daily with intermittent alumafoam splint placement
and immobilization

5.

Alumafoam splint placement and immobilization

1.

Irrigation and debridement with alumafoam placement and immobilization

2% (41/1759)

2.

Irrigation and debridement followed by percutaneous pinning and immobilization

8% (147/1759)

3.

Irrigation and debridement followed by reduction, nail bed repair and immobilization

88% (1547/1759)

4.
Betadine soaks at home three times daily with intermittent alumafoam splint placement
and immobilization
0% (8/1759)

5.

Alumafoam splint placement and immobilization

0% (8/1759)

Preferred Respone
The clinical presentation is consistent with a physeal separation and a nail bed injury. This is
also called a Seymour fracture which is a juxta-epiphyseal fracture of the distal phalanx.
Treatment of a nail bed avulsion and physeal separation is irrigation and debridement,
physeal reduction, nail bed repair and immobilization. The primary goals are to achieve a
stable, viable nail and good cosmetic results.
Inglefield at al retrospectively reviewed 19 children with 22 nail bed injuries. Early operative
repair led to good to excellent results in 91% of patients. They concluded that repair of the
nail bed at the time of injury is superior to secondary correction.
Fassler reviewed fingertip injuries, providing recommendations for treatment based on degree
of soft tissue loss, bone exposure, feasibility for flap coverage and the presence or absence of
mitigating systemic conditions. He also concluded that the outcome of nail bed injuries is
dependent on the severity of injury to the germinal matrix.
Illustration A shows the makeup of the terminal phalanx. Illustration B and C show a
Seymour Fracture before and after irrigation and debridement and reduction.
Illustrations :

A.

B.

C.

Frostbite

Introduction
Characterized by extensive soft tissue damage associated with exposure to temperatures
below freezing point
Pathophysiology
cell biology
o leads to movement of water from intracellular location to extracellular
location
o cellular dehydration leads to cell death
biochemistry
o ice crystal formation occurs within the extracellular fluid at -2 to -15 degrees
Celsius
o sensory nerve dysfunction occurs at -10 degrees Celsius
Prognosis
the severity is increased with
o alcohol consumption/intoxication
o contact of skin with metal or ice

elevated wind chill factor

Presentation
Physical exam
o inspection
blisters form 6-24 hours after rewarming
superficial lesions present as clear blisters

deeper lesions form hemorrhagic blisters which may be


painless

Imaging
Bone scan
o can be used to evaluate the severity of the soft-tissue damage
3rd day after initial injury

Treatment
Nonoperative
o initial resuscitation with warm IV fluids, rewarming of the affected
extremity, wound care and topical antibiotics
indications
first line of treatment
technique
rewarming of the affected extremity or body part
perform in waterbath at a temperature of 40-44 degrees
Celsius for 30 minutes

this may require IV analgesia or even conscious


sedation
repetitive freeze-thaw cycles should be avoided
wound care with topical aloe vera, extremity elevation and
splinting
white/clear blisters
require debridement
hemorrhagic blisters
should be drained, but left intact
topical antibiotics
apply topical antibiotics to prevent the development of
superinfections
o intravenous TPA
indications
no evidence of digital blood flow on bone scan
Operative
o immediate surgical escharotomy
indications
circumferentially constrictive lesion of digit
o surgical debridement with or without amputation
indications
after zone of injury has been completely demarcated
this may take 1-3 months

Complications
Adults
cold Intolerance
vasospastic disease
treatment
calcium channel blockers
indications
late, persistent vasospastic disease
surgical sympathetectomy
indications
late, persistent vasospastic disease
Children
o premature growth plate closure
secondary to chondrocytic injury
o joint laxity, short digits, degenerative joint changes
seen after age 10 in patients with prior frost bite injuries
o
o

Questions
A 22-year-old college student presents with significant finger pain after coming into contact
with liquid nitrogen in his chemistry lab. A clinical photo of the affected finger in shown in
Figure A. What is the most appropriate next step in treatment?

1.

Blister debridement and hyperbaric oxygen therapy

2.

Drainage of the blister with the overlying skin left intact

3.

Full thickness blister and skin debridement with local flap coverage

4.

MRI scan of the digit to assess degree of soft tissue damage

5.

Wet to dry twice-daily dressing changes to the digit

1.

Blister debridement and hyperbaric oxygen therapy

7% (176/2560)

2.

Drainage of the blister with the overlying skin left intact

50% (1273/2560)

3.

Full thickness blister and skin debridement with local flap coverage

17% (427/2560)

4.

MRI scan of the digit to assess degree of soft tissue damage

13% (338/2560)

5.

Wet to dry twice-daily dressing changes to the digit

13% (327/2560)

Preferred Respone
The clinical presentation is consistent with a hemorrhagic blister due to acute frostbite injury.
Of the options presented, the most appropriate treatment is drainage of the blister with the
overlying skin left intact. Hemorrhagic blisters represent deeper injuries, and dbriding them
could lead to desiccation of the underlying dermis. Alternatively, intact blisters can be left in
place and wrapped in dry gauze dressings until they resolve.
Golant et al completed a review article discussing cold exposure injuries to the extremities.
They state that frostbite, the most serious peripheral injury, results in tissue necrosis from
direct cellular damage and indirect damage secondary to vasospasm and arterial thromboses.
With regards to treatment, the authors conclude that dbridement of necrotic tissues is
generally delayed until there is a clear demarcation from viable tissues, a process that usually
takes from 1 to 3 months from the time of initial exposure. They advocate drainage of
hemorrhagic blisters, leaving the overlying skin intact.
Bruen et al review the treatment of digital frostbite in their current concepts review. They
state that physical examination results that are concerning for severe upper-limb injury
include the absence of Doppler pulse signals, absent capillary refill, dark purple discoloration
of the digits, and hemorrhagic blisters. They state that intact blisters should left in place and
wrapped in dry gauze dressings until they resolve. These findings should lead to further

perfusion evaluation with technetium-99m triple-phase bone scanning or angiography.


Incorrect Answers:
Answer 1: Hyperbaric oxygen therapy is not indicated as initial treatment of frostbite injury.
Answer 3: Full thickness skin dbridement should be delayed until full demarcation of the
injury is determined. This can take as long as three months in some cases.
Answer
4:
MRI
scan
is
not
indicated
initially.
Answer 5: Wet to dry dressing changes are most beneficial in the presence of an open wound,
as removal of the dressing when it dries removes drainage and debris. This treatment would
not be beneficial in the presence of a blister with the overlying skin intact.

High-Pressure Injection Injuries

Introduction
Characterized by extensive soft tissue damage associated with a benign high-pressure
entry wound
Epidemiology
demographics
o most common in laborers in industry using paint, automotive grease, solvents
and diesel oil
location
o the non-dominant index finger is the most commonly affected
Pathophysiology
vascular occlusion may lead to local soft tissue necrosis
Prognosis
severity of the injury is dependent on
o time from injury to treatment
o force of injection

o
o

volume injected
composition of material
grease, latex, chloroflourocarbon & water based paints are less
destructive
industrial solvents & oil based paints cause more soft tissue necrosis

Treatment
Nonoperative
o parenteral antibiotics, elevation and early mobilization
indications
there is a limited role for this
less severe injuries
Operative
o irrigation & debridement, foreign body removal and broad-spectrum
antibiotics
indications
most cases require immediate surgical debridement

technique
it is important to remove as much of the foreign material as
possible
broad spectrum antibiotic coverage is important to reduce risk
of post operative infection
outcomes
higher rates of amputation are seen when surgery is delayed
greater than 10 hours after injury

Complications
Amputation

rates of amputation approach 50% with oil-based paint injection injuries


Infection
o necrotic tissue is a good culture medium for bacterial growth
o

Questions
A 36-year-male was using a high-pressure paint gun when he suffered the injury shown in
Figure A. Which of the following variables would have the worst impact on his prognosis?

1.

Delay in surgical treatment

2.

Injected solvent was grease

3.

Injected solvent was water-based paint

4.

An entry wound of greater than 3 cm

5.

Injected solvent was at room temperature

1.

Delay in surgical treatment

81% (2243/2786)

2.

Injected solvent was grease

16% (446/2786)

3.

Injected solvent was water-based paint

2% (49/2786)

4.

An entry wound of greater than 3 cm

1% (31/2786)

5.

Injected solvent was at room temperature

0% (4/2786)

Preferred Respone
The clinical presentation is consistent for a high-pressure injection injury. Delays in surgical
treatment are associated with serious sequelae.
High-pressure injection injuries are characterized by extensive soft tissue damage associated
with a benign high-pressure entry wound. They should be treated with irrigation &
debridement, foreign body removal and broad-spectrum antibiotics. There is a higher rates of
amputation when surgery is delayed.

Bekler et al. looked at the results of 14 surgically treated high-pressure injection injuries of
the hand with a minimum of two years follow-up. Ten of the injuries required formal
operative debridement and foreign body removal. Six required reconstructive microsurgical
procedures and one underwent digital tip amputation. They concluded that high-pressure
injection injury to the hand is a significant problem, which can easily lead to serious sequelae
and, even, amputation.
Rosenwaser et al. report wide dbridement of all involved tissues, decompression of tissue
compartments, exploration and incision of tendon sheaths, removal of injected material, and
saline irrigation are critical in the management of high-pressure injection injuries to the hand.
They emphasize delayed surgery has been associated with increased incidence of morbidity
and amputation.
Figure A shows a typical high-pressure injection injury. Notice the benign looking entry
wound.
Incorrect Answers:
Answer 2: Grease as an injected solvent has a more favorable prognosis when compared to
industrial solvents & oil based paints.
Answer 3: Water-based paint as an injected solvent has a more favorable prognosis when
compared to industrial solvents & oil based paints.
Answer 4: The size of the entry wound does not have a strong correlation with the severity of
injury. Often times these injuries have a benign looking entry wound.
Answer 5: Injected solvent at high temperatures are associated with a worse prognosis.

Base of Thumb Fractures

Introduction
Base of the thumb metacarpal fractures include
Bennett fracture (intra-articular)
Rolando fracture (intra-articular)
extra-articular fractures
Epidemiology
incidence
o 80% of thumb fractures involve the metacarpal base
o most common variant is the Bennet fracture
Pathophysiology
mechanism of injury
o most fractures caused by axial force applied to the thumb
pathoanatomy
o three muscles provide deforming forces at base of thumb
abductor pollicis longus (PIN)
extensor pollicis longus (PIN)
adductor pollicis (Ulnar n.)
o the thumb has extensive CMC motion in sagittal plane
allows for angulation up to 30 degrees in this plane

Bennett Fracture

Intra-articular fracture/dislocation of base of 1st metacarpal characterized by


volar lip of metacarpal based attached to volar oblique ligament
o ligament holds this fragment in place
o small fragment of 1st metacarpal continues to articulate with trapezium
Pathoanatomy
lateral retraction of distal 1st metacarpal shaft by APL and adductor pollicis
o shaft pulled into adduction
o metacarpal base supinated
Prognosis
better than Rolando fx
Imaging
radiographs
o recommended views
fracture best seen with hyper-pronated thumb view
o findings
minimal joint step-off considered best
Treatment
nonoperative
o closed reduction & cast immobilization
indications
nondisplaced fractures
technique
reduction maneuver with traction, extension, pronation, and
abduction
operative
o closed reduction and percutaneous pinning
indications
volar fragment is too small to hold a screw
anatomic reduction unstable
technique
can attempt reduction of shaft to trapezium to hold reduction
o ORIF
indications
large fragment
2mm+ joint displacement
Complications
post-traumatic arthritis

there is no agreement regarding the relationship of post-fixation joint


incongruity and post-traumatic arthritis

Rolando Fracture

Intra-articular fracture of base of 1st metacarpal characterized by


intra-articular comminution
Epidemiology
less common than Bennett's fracture
Pathoanatomy
deforming forces are the same as Bennett's fracture
o volar fragment should have volar oblique ligament attached
o shaft pulled dorsally
typically the base is split into a volar and dorsal fragment
o commonly called a 'Y' fracture
often have more than two proximal fragments
Prognosis
worse than Bennett fx
Treatment
nonoperative
o immobilization
indications
for severe comminution, stable
start early range of motion
operative
o external fixation, CRPP
indications
for severe comminution, unstable
technique
can approximate large fragments with k-wires
o ORIF
indications
most common fixation method
technique
use t-plate or blade plate
can use k-wires of fragments are too small for screw purchase
Complications
commonly results in post-traumatic osteoarthritis

Extra articular Fracture

Extra-articular fracture of base of 1st metacarpal


can be transverse or oblique in nature
Treatment
nonoperative
o spica casting
indications
if joint is reduced and there is less than 30 degrees of
angulation
operative
o CRPP
indications
if reduction cannot be held to result in less than 30 degrees of
angulation
outcome
these fractures typically have the best outcome

Questions
Which of the following muscles provide the primary deforming forces to Bennett and
Rolando fractures (base of the 1st metacarpal fractures)?
1.

Pronator quadratus

2.

Flexor pollicis longus

3.

Extensor pollicis longus

4.

Adductor pollicis longus and abductor pollicis

5.

Abductor pollicis longus and adductor pollicis

1.

Pronator quadratus

1% (13/1748)

2.

Flexor pollicis longus

2% (30/1748)

3.

Extensor pollicis longus

3% (47/1748)

4.

Adductor pollicis longus and abductor pollicis

10% (167/1748)

5.

Abductor pollicis longus and adductor pollicis

85% (1482/1748)

Preferred Respone
The primary deforming forces in Bennett and Rolando fractures are the Abductor pollicis
longus and adductor pollicis.
In a Bennet's or Rolando fracture-dislocation the volar-ulnar fracture fragment is held
reduced by the anterior oblique ligament while strong deforming forces pull the remaining
metacarpal shaft proximally and dorsally, angulate the shaft ulnarly and supinate the shaft.
Most important in these deforming forces are the abductor pollicis longus (APL) inserting on
the base of the metacarpal which pulls the metacarpal shaft proximally and dorsally and the
adductor pollicis (AP) which inserts on the ulnar base of the proximal phalanx and angulates
the metacarpal shaft ulnarly and supinates the shaft. Less important is the extensor pollicis
longus (EPL) which inserts on the base of the distal phalanx and also adds to the ulnar
angulation
of
the
distal
fragment
Soyer reviews the diagnosis, pathoanatomy, and treatment for fractures at the base of the 1st
metacarpal. Understanding the biomechanics, anatomical deforming forces, and the exact
fracture pattern aids the treating surgeon in determining the most appropriate method of
fixation. The most essential factor for obtaining a good functional result is anatomic
restoration of the articular surface.
Elgafy et al. examined the terminal anatomy of the posterior interosseous nerve in their
cadaver study - identifing six terminal branches and describing methods to avoid injury. They
describe how treating surgeons can maximize function and recovery after base of the 1st
metacarpal fractures by understanding these nervous branches and specific fracture pattern
treatment to avoid iatrogenic injury to the PIN.

A.

Thumb Collateral Ligament Injury

Introduction
Thumb collateral ligament injuries include
radial collateral ligament
o rare
ulnar collateral ligament
o most common
o eponyms for ulnar collateral ligament (UCL) injury are
Gamekeeper's thumb for chronic injury
skiers thumb for acute injury
Stener lesion
avulsed ligament with or without bony attachment is displaced
above the adductor aponeurosis
will not heal without surgical repair
Epidemiology
UCL more common than radial collateral ligament
Mechanism
hyper abduction or extension at the MCP joint

Anatomy

UCL is composed of
proper collateral ligament
o resists valgus load with thumb in flexion
accessory collateral ligament and volar plate
o resists valgus load with thumb in extension
o valgus laxity in both flexion and extension is indicative of a complete UCL
rupture

Presentation
History
o hyperabduction injury
Symptoms
o pain at ulnar aspect of thumb MCP joint
Physical exam
o inspection and palpation
mass from torn ligament and possible bony avulsion may be present
o stress joint with radial deviation both at neutral and 30 of flexion
instability in 30 of flexion indicates injury to proper UCL
instability in neutral indicates injury to accessory UCL and/or volar
plate
compare to uninjured thumb MCP joint

Imaging
Radiographs

recommended views
AP, lateral and oblique of thumb
valgus stress view may aid in diagnosis if a bony avulsion has already
been ruled out

MRI
o

can aid in diagnosis if exam equivocal

Treatment
Nonoperative
o immobilization for 4 to 6 weeks
indications

partial tears with < 20 side to side variation of varus/valgus


instability
Operative
o ligament repair
indications
acute injuries with
> 20 side to side variation of varus/valgus instability
>35 of opening
Stener lesion

avulsed ligament with or without bony attachment is


displaced above the adductor aponeurosis
will not heal without surgical repair

technique
can use suture, suture anchors, or small screw to repair
ligament
reconstruction of ligament with tendon graft, MCP fusion, or adductor
advancement
indications
chronic injury

Radial Collateral Ligament Injury


Rare
Treatment
o nonoperative
immobilization
indicated in most cases
Stener's lesion does not occur

Question
Creation of a Stener lesion, as found in Gamekeeper's thumb, requires combined tears of the
proper and accessory ulnar collateral ligaments in order for the ligament to be displaced by
the adductor aponeurosis. Which of the following most accurately describes the role these
ulnar collateral ligaments (PCL/ACL) play in thumb MCP joint stability?
1.

PCL is primary restraint to radial deviation with MCPJ in flexion, ACL provides
restraint to radial deviation with MCPJ in extension

2.

PCL is primary restraint to radial deviation with MCPJ in extension, ACL provides
restraint to radial deviation with MCPJ in extension

3.

ACL is primary restraint to ulnar deviation with MCPJ in flexion, PCL provides
restraint to ulnar deviation with MCPJ in extension

4.

ACL is primary restraint to radial deviation with MCPJ in flexion, PCL provides
restraint to radial deviation with MCPJ in extension

5.

PCL is primary restraint to ulnar deviation with MCPJ in flexion, ACL provides
restraint to radial deviation with MCPJ in extension

1.
PCL is primary restraint to radial deviation with MCPJ in flexion, ACL provides
restraint to radial deviation with MCPJ in extension
59% (903/1540)

2.
PCL is primary restraint to radial deviation with MCPJ in extension, ACL provides
restraint to radial deviation with MCPJ in extension
9% (131/1540)

3.
ACL is primary restraint to ulnar deviation with MCPJ in flexion, PCL provides
restraint to ulnar deviation with MCPJ in extension
9% (139/1540)

4.
ACL is primary restraint to radial deviation with MCPJ in flexion, PCL provides
restraint to radial deviation with MCPJ in extension
17% (259/1540)

5.
PCL is primary restraint to ulnar deviation with MCPJ in flexion, ACL provides
restraint to radial deviation with MCPJ in extension
6% (90/1540)

Preferred Respone
The proper ulnar collateral ligament(PCL) runs from the metacarpal head to the volar aspect
of proximal phalanx and resists ulnar stress with the thumb MCPJ in flexion. The accessory
ulnar collateral ligament(ACL) lies palmar to the proper ligament, and insets inserts onto the
volar plate. The volar plate and ACL function as the principle restraints to ulnar stress with
the thumb MCPJ in extension.
The function of the ulnar collateral ligaments is shown in Illustration A.(Please note the distal
phalanx of the thumb has been removed in Illustration A.) A Stener lesion is described by
displacement of the distal end of the completely ruptured UCL such that it comes to lie
superficial and proximal to the adductor aponeurosis. This is shown in Illustration B.
Thrikannad and Wolff report a case of distal pull-off of the ulnar collateral ligament (UCL)
of the thumb MCPJ with two fracture fragments. They identify the need to look for a second
fragment of bone in these injuries, where an apparently undisplaced fracture is noted at the
base of the proximal phalanx. They suggest that this second fragment probably indicates the
location of the distal end of the UCL and may identify a Stener lesion. A radiographic
example from their paper is shown in Illustration C.
Newland, in his review article on Gamekeeper's Thumb, states that criteria for judging what
constitutes a complete tear vary from 15 deg to 45 deg difference with respect to the opposite

side. He goes on to state, however, that many authors choose an absolute value of >35
degrees of joint laxity compared to the contralateral side when judging a tear to be complete
or incomplete. When an acute tear is identified, surgical repair is recommended.
Illustration :

A.

B.

C.

Paronychia

Introduction
A soft tissue infection of the proximal or lateral nail fold
Epidemiology
incidence
o most common hand infection (one third of all hand infections)
demographics
o usually in children
o more common in women (3:1)
location
o most commonly involve the thumb
Pathophysiology
organism
o acute infection
adults - usually caused by Staphylococcus aureus
children - usually mixed oropharyngeal flora

diabetics - mixed bacterial infection


chronic infection
Candida albicans (more common in diabetics)
often unresponsive to antibiotics

Classification
Acute paronychia
o minor trauma from nail biting, thumb sucking, manicure
Chronic paronychia
o occupations with prolonged exposure to water and irritant acid/alkali
chemicalse.g. dishwashers, florists, gardeners, housekeepers, swimmers,
bartenders
o risk factors for chronic paronychia
diabetes
psoriasis
steroids
retroviral drugs (indinavir and lamivudine)
indinavir is most common cause of paronychia in HIV positive
patients
resolves when medication is discontinued

Anatomy
Nail organ

adds to stability of finger tip by acting as counterforce to finger pulp


thermoregulation (glomus bodies of nail bed and nail matrix)
allows "extended precision grip" (using opposed thumbnail and index
fingernail to pluck out a splinter)
Nail plate
o made of keratin, grows at 3mm/month, faster in summer
o fingernails grow faster than toenails (fingernails take 3-6 months to regrow,
and toenails take 12-18 months)
o growing part is under proximal eponychium
Perionychium
o
o
o

comprises hyponychium, eponychium and paronychium

Presentation
Symptoms
o acute paronychia
pain and
nail fold tenderness
erythema
swelling
o chronic paronychia
recurrent bouts of low-grade inflammation (less severe than acute
paronychia)
Physical exam
o acute paronychia
fluctuance
nail plate discoloration (green discoloration suggests Pseudomonas)

chronic paronychia
nail plate hypertrophy (fungal infection)
nail fold blunting and retraction after repeated bouts of inflammation
prominent transverse ridges on nail plate

Differential
Herpetic whitlow
Felon
Onychomycosis
Psoriasis
Glomus tumor
Mucous cyst

Treatment
Acute paronychia

nonoperative
warm soaks, oral antibiotics and avoidance of nail biting
indications
swelling only, but no fluctuance
medications
augmentin or clindamycin
o operative
I&D with partial or total nail bed removal followed by oral abx
indications
fluctuance (indicates abscess collection)
nail bed mobility (indicates tracking under the nail)
follow with oral antibiotics and routine dressing change
Chronic paronychia
o nonoperative
warm soaks, avoidance of finger sucking, topical antifungals
indications
first line of treatment
medications
miconazole is commonly used
o operative
marsupialization (excision of dorsal eponychium down to level of
germinal matrix)
indications
severe cases that fail nonoperative treatment
technique
combine with nail plate removal
leave to heal by secondary intention
o

Techniques
I&D with partial or total nail bed removal
approach
o may be done in emergency room
o incision into sulcus between lateral nail plate and lateral nail fold
technique
o preserve eponychial fold by placing materials (removed nail) between skin and
nail bed
o if abscess extends proximally over eponychium (eponychia), a separate
counterincision is needed over the eponychium
o obtain gram stain and culture

Complications
Eponychia
o spread into eponychium
Runaround infection
o involvement of both lateral nail folds
Felon
o spread volarward to pulp space

I&D of finger pulp is necessary


Flexor tenosynovitis
o volar spread into flexor sheath
Subungual abscess ("floating nail")
o nail plate removal is necessary
o

Felon

Introduction

Infection of finger tip pulp


usually thumb and index finger
Pathophysiology
mechanism
o penetrating injury including
blood glucose needle stick
splinters
o local spread

may spread from paronychia


o no history of injury in 50% of patients
pathoanatomy
o swelling and pressure within micro-compartments, leading to "compartment
syndromes" of the pulp
organism
o Staphylococcus aureus
most common organism
o gram negative organisms
found in immunosuppressed patients
o Eikenella corrodens
found in diabetics who bite their nails

Anatomy
Fingertip micro-compartments
o pulp fat is separated by fibrous vertical septae running from distal phalanx bone to
dermi

Presentation
Symptoms
o pain, swelling

Physical exam
o tenderness on distal finger

Treatment
Operative
o I&D in emergency room followed by IV antibiotics
indications
most cases due to risk of finger tip compartment syndrome

Techniques
Fingertip irrigation & debridement
o approach
keep incision distal to DIP crease
to prevent DIP flexion crease contracture and prevent extension
into flexor sheath
mid-lateral approach

indicated for deep felons with no foreign body and not


discharging
incision on ulnar side for digits 2,3 and 4 and radial side for
thumb and digit 5 (non-pressure bearing side of digit)
volar longitudinal approach

most direct access


indicated for superficial felons, foreign body penetration or
visible drainage
incisions to avoid

fishmouth incisions - leads to unstable finger pulp


double longitudinal or transverse incision - injury to digital
nerve and artery
o

debridement
avoid violating flexor sheath or DIP joint to avoid spread into these
spaces
break up septa to decompress infection and prevent compartment
syndrome of fingertip

obtain gram stain and culture


hold antibiotics until culture obtained
postoperative
routine dressing changes

Complications
Finger tip compartment syndrome
Flexor tenosynovitis
Osteomyelitis
Digital tip necrosis

Pyogenic Flexor Tenosynovitis

Introduction
Infection of the synovial sheath that surrounds the flexor tendon
incidence 2.5 to 9.4% of all hand infections
risk factors
o diabetes

o
o

IV drug use
immunocompromised patients

mechanism
o penetrating trauma to the tendon sheath
o direct spread from
felon
septic joint
deep space infection
pathoanatomy
o infection travels in the synovial sheath that surrounds the flexor tendon
microbiology
o Staph aureus (40-75%)
most common
o MRSA (29%)
intravenous drug abusers
o other common skin flora
staph epidermidis
beta-hemolytic streptococcus
pseudomonas aeruginosa
o mixed flora and gram negative organsims
in immunocompromised patients
o Eikenella
in human bites
o Pasteurella multocida
in animal bites
"horseshoe abscess"
o may develop from spread pyogenic flexor tenosynovitis
of many individuals have a connection between the sheaths of the
thumb and little fingers at the level of the wrist

infection in one finger can lead to direct infection of the sheath on the
opposite side of the hand resulting a "horseshoe abscess"

Anatomy
Tendon sheaths
function
o to protect and nourish the tendons
anatomy
o variations common

sheaths extends from

index, middle, and ring fingers


from DIP to just proximal to A1 pulley
thumb (flexor pollicus longus sheath)
from IP joint to as proximal as radial bursa (in wrist)
little finger
from DIP joint to as proximal as ulnar bursa (in wrist)

Presentation
Symptoms
o pain and swelling
typically present in delayed fashion (over last 24-48 hours)
usually localized to palmar aspect of one digit
Physical exam
o
Kanavel signs (4 total)

flexed posturing of the involved digit


tenderness to palpation over the tendon sheath
marked pain with passive extension of the digit
fusiform swelling of the digit
increased warmth and erythema of the involved digit

Imaging
Radiographs
o recommended views
radiographs usually not required, but may be useful to rule out foreign
object
MRI
o cannot distinguish infectious flexor tenosynovitis from inflammatory but may
help determine the extent of the ongoing process

Treatment
Nonoperative (rare)
o hospital admission, IV antibiotics, hand immobilization, observation
indications
early presentation
modalities
splinting
outcomes
if signs of improvement within 24 hours, no surgery is required
Operative
o I&D followed by culture-specific IV antibiotics
indications
low threshold to operative once suspected (orthopaedic
emergency)
late presentation
no improvement after 24 hours of non-operative treatment
(confirmed diagnosis)
technique (see below)

Techniques
I&D of flexor tendon
o approach
full open exposure using long midaxial or Bruner incision
two small incisions placed distally at A5 pulley and proximally at A1
pulley and using an angiocatheter

Complications
Stiffness

Tendon or pulley rupture


Spread of infection
Loss of soft tissue
Osteomyelitis
Question
A 46-year-old homeless IV drug abuser presents with the hand infection shown in Figure A,
which developed after sustaining a superficial laceration. Cultures are taken during operative
irrigation and debridement, and he is started on antibiotic therapy. Based on the patients
history, what is the most common pathogen in this setting?

1.

Herpes simplex virus

2.

Candida albicans

3.

Escherichia coli

4.

Eikenella corrodens

5.

Methicillin-resistant staphylococcus aureus

1.

Herpes simplex virus

1% (7/1320)

2.

Candida albicans

1% (7/1320)

3.

Escherichia coli

1% (7/1320)

4.

Eikenella corrodens

3% (34/1320)

5.

Methicillin-resistant staphylococcus aureus

96% (1262/1320)

Preferred Respone
Figure A shows an abscess over the metacarpophalangeal joint of the thumb. Infections with
these characteristics in IV drug abusers are most commonly caused by MRSA, and can affect
any portion of the hand.
Imahara et al retrospectively reviewed 159 hand infections treated in the operating room over
an 11-year period. The examined data included known risk factors for MRSA, including
human immunodeficiency virus infection, diabetes mellitus, intravenous drug use,
incarceration, and homelessness. Intravenous drug use was the only independent risk factor
for CA-MRSA infections.

Boucher et al examined the trends in both nosocomial and community-associated MRSA


infections and explored recent studies of the mechanisms that allow S. aureus to become
resistant to currently available drugs.
Incorrect Answers:
1-Herpes simplex virus can cause Herpetic whitlow, as shown in Illustration A, typically
presents on the fingers health care workers exposed to a carriers mouth. Usually, this
infection appears as small ulcers or vesicles, and operative debridement is contraindicated.
2-Candida albicans is a more rare hand infection typically associated with chronic
paronychia, as shown in Illustration B.
3-Escherichia coli is a less common cause of abscess formation in the hand.
4-Eikenella is usually associated with "fight-bite" infections on the dorsal aspect of the MCP
joint, and does not commonly occur after superficial lacerations. It can also rarely occur in IV
drug users who clean their needles with saliva, as Eikenella is part of the normal oral flora.
An example of an Eikenella infection is shown in Illustration C.

A.

B.

C.

Deep Space & Collar Button Infections

Introduction
Deep space infections
defined as infections of the
o thenar space
most commonly infected
o hypothenar space
o midpalmar space
rare
Collar button abscess
an abscess that occurs in the web space between fingers

Anatomy

a bursa (potential space) just palmar to adductor pollicis and dorsal to flexor tendons
separated from midpalmar potential space by a fascial septum

located dorsal and radial to hypothenar space

located palmar to fifth metacarpal, dorsal and radial to hypothenar fascia, ulnar to
hypothenar septum

Presentation
History
o may or may not have penetrating trauma
Symptoms
o pain
o swelling
Physical exam

pain with flexion of fingers


thenar
pain with thumb flexion
hypothenar
pain with small finger flexion
midpalmar
pain with small, ring, and small finger flexion
thenar and midpalmar spaces
often have loss of palmar concavity secondary to swelling

Imaging
Radiographs
o indicated if there is suspicion for a foreign body
MRI
o indications
help define extent of infection

Treatment
Operative
incision and drainage in conjunction with IV antibiotics

o
o

indications
standard of care for deep space infections and collar button abscesses
technique
use volar and dorsal incisions for collar button abscesses
avoid skin in actual web space

Dog and Cat Bites

Introduction

Bites by domestic animals are common


delayed presentation (>1week) is common
required care is often underestimated
Epidemiology
incidence
o dog bites are most common animal bites in the US (90%)
dog is known to victim in 90% of cases
o cat bites are 2nd most common (10%)

human bites are 3rd most common


demographics
o more common in males
o more frequent at ages 2-19 years
location
o upper extremity bites more common than lower extremity
o

Pathophysiology
mechanism
o dog bites
cause crush, puncture, avulsion, tears and abrasions

large dogs' jaws exert >450lbs/ square inch


more likely to cause structural damage to nerves, vessels, joint
cat bites
penetrate bones and joints, and cause septic arthritis and osteomyelitis
small, sharp teeth cause puncture wounds that seal immediately

penetrate joints and flexor tendons


higher risk for infection than dog bites
Associated conditions
secondary bacterial infection
o most bites do not become infected
o risk factors for infection
bite to hand, foot, or major joint
puncture wounds or crush injuries
treatment delay >12h
age >50y
preexisting host disease
immune suppression (steroids, asplenism)
chronic alcoholism

diabetes mellitus
vascular disease
existing edema of extremity
o

microbiology
most infections are polymicrobial, with at least 1 anerobe
dog bites
Staphylococcus aureus
Streptococcus alpha-hemolytic
Pasteurella multocida (25-50% of dog bite infections)
Corynebacterium
anerobes (e.g. Bacteroides)
Capnocytophaga canimorsus
rare, potentially fatal (in splenectomy patients)
causes septicemia, endocarditis, meningitis
cat bites
Pasteurella multocida (most common, 70-80% of cat bite
infections)
causes intense pain, swelling in 48h
other organisms similar to dog bites

rabies
o
o
o

caused by a rhabdovirus
common animal carriers include dogs, raccoons, bats, foxes
increased risk with open wounds, scratches/abrasions, mucous membranes

Prognosis
serious and fatal bites include
o large, aggressive dogs
o small children
o head and neck bites

Presentation
History
o important to determine
type of animal
time since injury
presence of comorbidities
Symptoms
o pain and swelling
o bleeding
o signs of local or systemic sepsis
Physical Exam
o evaluate depth of puncture wound and presence of crush injury
o check for neurovascular status
o look for joint penetration
o important to photograph wounds

Imaging

Radiographs
o indications to obtain
crush injuries
suspected fracture
suspected foreign body

Studies
Culture
o indications
if signs of infection are present
routine culture not indicated
o technique
deep aerobic and anaerobic culture

Treatment
Noperative
o copious irrigation, prophylactic antibiotics, tetanus toxoid, +/- rabies
prophylaxis
copious irrigation
usually performed in emergency room
saline (>150ml) irrigation with 18-19G needle or plastic
catheter
use povidone-iodine solution if high risk of rabies
antibiotics
indications for antibiotics
cat bites
presentation >8h
immune compromised or diabetic
hand bite
deep bites
choice of antibiotics
amoxicillin/clavulanic acid effective against Pasteurella
multocida
cefuroxime
ceftriaxone
rabies prophylaxis
indicated when any suspicion for rapid animal
suspect if unprovoked attack by animal with bizarre
behavior
human diploid cell vaccine and human rabies immunoglobulin
immobilization
immobilize and elevate extremity
Operative
o formal surgical debridement
indications
crush or devitalized tissue
foreign body

bites to digital pulp space, nail bed, flexor tendon sheath, deep
spaces of the palm, joint spaces
tenosynovitis
septic arthritis
abscess formation

Human Bite

Introduction
Epidemiology
incidence
o third most common bite behind dog and cat
demographics
o more common in males
location
o typically dorsal aspect of 3rd or 4th MCP joint
"fight bite"
Pathophyiology
mechanism

most often result of direct clenched-fist trauma (from tooth) after punching
another individual in the mouth
o can also result from direct bite (i.e. child biting another child)
pathoanatomy
o tooth penetrates capsule of MCP joint
flora (bacteria) from mouth enter joint
bacteria become trapped within joint as fist is released from clenched
position
bacteria now caught under extensor tendon and/or capsule
microbiology
o typically polymicrobial
o most common organisms
alpha-hemolytic streptococcus (S. viridans) and staphylococcus aureus
eikonella corrodens in 7-29%
other gram negative organisms
Associated conditions
extensor tendon lacerations
o can be missed due to proximal tendon retraction
o

Presentation
History
o direct clenched-fist trauma to another individual's mouth
often overlooked
must have high index of suspicion as patients often unwilling to reveal
history
consider the injury a "fight-bite" until proven otherwise
o possible delay in presentation until symptoms become intolerable
Symptoms
o progressive development of pain, swelling, erythema, and drainage over
wound
Physical exam
o fight bite
small wound over dorsal aspect of MCP joint
wound often transverse, irregular
typically 3rd and/or 4th MCPs, but can involve any digit

erythema, warmth, and/or edema overlying wound and joint


purulent drainage
must assess for integrity of extensor tendon function

possible pain with passive ROM of MCP joint


typically no involvement of volar/flexor surface of digit
neurovascular status typically preserved

Imaging
Radiographs
o indicated to assess for foreign body (i.e. tooth fragment) and for fracture

Studies
Culture
o not routinely obtained in ED due to contamination
o deep culture obtained in OR
aerobic and anaerobic

Treatment
Operative
I&D, IV antibiotics
o indications
fight bite
joints or tendon shealths are involved
o antibiotics
IV antibiotics directed at Staph, Strep, and gram-negative organisms
ampicillin/sulbactam (unasyn)
PO antibiotics upon discharge for 5 to 7 days
amoxicillin/clavulanic acid (augmentin)
o debridement
debridement of wound and joint capsule
wound left open for drainage
obtain gram stain and culture

Herpetic Whitlow

Introduction
A viral infection of the hand caused by herpes simplex virus (HSV-1)
Epidemiology
o demographics
occurs with increased frequency in medical and dental personnel
most common infection occurring in a toddlers and preschoolers hand
Pathophysiology
o viral shedding occurs while vesicles are forming bullae

Presentation
Symptoms
o intense burning pain followed by erythema
o malaise
Physical exam
o erythema followed by small, vesicular rash
over the course of 2 weeks, the vesicles may come together to form
bullae
the bullae will crust over and ultimately lead to superficial ulceration
o fever and lymphadenitis may be found

Studies
Tzank smear
o diagnosis confirmed by culture, antibody titers or Tzank smear

Treatment
Nonoperative
o observation +/- acyclovir
indications
standard of treatment
outcomes
self limiting, with resolution of symptoms in 7-10 days
acyclovir may shorten the duration of symptoms
recurrence may precipitated by fever, stress and sun exposure
Operative
o surgical debridement
indications
none
surgical treatment associated with superinfections,
encephalitis, and death and should be avoided

Complications
Superinfections
o often the result of surgical intervention
in pediatric patients, an infection of the digits may occur and require
treatment with an oral antibiotic (penicillinase resistant) ifor 10 days

Atypical Mycobacterium Infections

Introduction
Nontuberculous mycobacterial infections
Epidemiology
demographics
o often found in marine workers
location
o hand and wrist are involved in 50% of cases
risk factors
o immunocompromised host
Pathophysiology

incubation
o average incubation period is two weeks, but can be up to six months
o average time to diagnosis and appropriate treatment is more than 1 year
organisms
o widely encountered in the environment, but rarely cause human pathology
M. marinum
most common atypical mycobacterium infection
more common in stagnant fresh or salt water (aquariums)
M kansasii
found in soil
M terrae
found in soil
M. avium intracellulare
most common in terminal AIDS patients, but can occur in nonHIV patients
Prognosis
natural history
o early presentation includes papules, nodules, and ulcers
o late presentation may have progressed to tenosynovitis, septic arthritis, or
osteomyelitis
morbidity & mortality
o mortality rate is 32%

Presentation
Symptoms
o cutaneous rash with discomfort
Physical exam
o papules, ulcers, and nodules are common, especially on the hands
many times presents with a single nodule that may ultimately spread to
the lymph nodes
indistinguishable from tuberculous mycobacterial infection

Studies
Histology
o granulomas may or may not demonstrate acid-fast bacilli on AFB stain
Cultures and sensitivities are key to diagnosis
o Lowenstein-Jensen culture agar
M. marinum incubated specifically at 30 to 32 C

M. avium intracellulare incubated at room temperature

Treatment
Nonoperative
o oral antibiotics
indications
if diagnosed at early stage
medications
ethambutol,
tetraycline,
trimethoprim-sulfamethoxazole,
clarithromycin, azithromycin
add rifampin if osteomyelitis present
Operative
o surgical debridement + oral antibiotics in combination for 3 to 6 months
indications
later stage disease
use a combination of above medications

Fungal Infections

Introduction
Cutaneous fungal infections of the hand are rare and usually mild
o more common to have fungal infection in macerated skin areas (skin folds)
Prognosis
o usually resolve spontaneously
o May have serious infection in immunocompromised host

Classifications
Infections divided into three categories
cutaneous
o includes nail bed infections (onychomycosis)
subcutaneous
o includes sporothrix schenckii from rose thorn prick
deep
o orthopaedic manifestation
tenosynovial
septic arthritis
osteomyelitis
o organisms include
endemic
coccidiomycosis
histoplasmosis
blastomycosis
opportunistic include
candidiasis
mucormycosis
cryptococcocis
asperfillosisi
o requires surgical debridement

Onychomycosis
Introduction
o defined as fungal infection in vicinity of nail bed (cutaneous)
o most common organisms are
trichophyton rubrum
a destructive nail plate infection
candida
chronic infection of nail fold
Treatment
o topic antifungal treatment & nail plate removal
indications
first line of treatment
o systemic griseofulvin or ketoconazole
indications
recalcitrant cases

Sporothrix schenskii
Introduction
o Sporothrix schenckii a common soil organism
o a subcutaneous infection
o rose thorn in classic mechanism of subcutaneous transmission
Presentation
o physical exam

will show local ulceration (papule) at site of penetration

with time additional lesions form in region on lymphatic vessels

may show proximal lymph node involvement


Evaluation
o S schenckii isolated at room temperature on Sabouraud dextrose agar
Treatment
o oral itraconazole for 3 to 6 months
indications
mainstay of treatment
has replaced potassium iodide due to side effects which included
thyroid dysfunction
rash
GI symptoms

Coccidiomycosis
Introduction
o found in southwest arid regions (e.g., new mexico)
o often a deep infection
Presentation
o manifestations include
subclinical pulmonary involvement
orthopaedic manifestations
synovitis
arthritis
periarticular osteomyelitis
Treatment
o amphotericin B & surgical debridement

Histoplasmosis

Introduction
o histoplasma capsulatum infection
o found in Mississippi River Valleys and Ohio
Presentation
o usually subclinical
o often found incidentally on CXR
o may present with tenosynovial infection
Evaluation
o diagnosed by skin testing
Treatment
o amphotericin B & surgical debridement / tenosynovectomy

Fingertip Amputations & Finger Flaps


Author: Mark Karadsheh
Topic updated on 08/29/15 1:18pm

Introduction

Injury to the finger with variable involvement of soft tissue, bone, and tendon
Goals of treatment
o sensate tip
o durable tip
o bone support for nail growth
Prognosis
o improper treatment may result in stiffness and long-term functional loss
Anatomy
Fingertip anatomy
o eponychium
soft tissue on the dorsal surface just proximal to the nail

o
o

paronychium
lateral nail folds
hyponychium
plug of keratinous material situated beneath the distal edge of the
nail where the nail bed meets the skin
lunula
white portion of the proximal nail
demarcates the sterile from germinal matrix beneath
nail bed
sterile matrix
where the nail adheres to the nail bed
germinal matrix
proximal to the sterile matrix
responsible for 90% of nail growth

Presentation
History
o mechanism
avulsion
laceration
crush
Physical exam
o inspection
often, characteristics of laceration will guide management
presence or absence of exposed bone
o range of motion
flexor and extensor tendon involvement
Imaging
Radiographs
o required imaging
AP/lateral radiographs to assess for bony involvement

Treatment
Nonoperative
o healing by secondary intention
indications
adults and children with no bone or tendon exposed
with < 2cm of skin loss
children with exposed bone
Operative
o primary closure (revision amputation)
indications
finger amputation with exposed bone and the ability to
rongeur bone proximallywithout compromising bony
support to nail bed
o full thickness skin grafting from hypothenar region

indications
fingertip amputation with no exposed bone and > 2cm
of tissue loss
flap reconstruction
indications
exposed bone or tendon where rongeuring bone
proximally is not an option

Surgical Techniques
Secondary intention
o technique
initial treatment with irrigation and soft dressing
after 7-10 days, soaks in water-peroxide solution daily followed by
application of soft dressing and fingertip protector
complete healing takes 3-5 weeks
Full thickness skin grafting from hypothenar region
o technique
split thickness grafts not used because they are
contractile
tender
less durable
donor site is closed primarily
graft is sutured over defect
cotton ball secured over graft helps maintain coaptation with
underlying tissue
o post-operative care
cotton ball removed after 7 days
range of motion encouraged after 7 days
Primary closure with removal of exposed bone (revision amputation)
o technique
must ablate remaining nail matrix
prevents formation of irritating nail remnants
if flexor or extensor tendon insertions cannot be
preserve, disarticulate DIP joint
transect digital nerves and remaining tendons as proximal as
possible
palmar skin is brought over bone and sutured to dorsal skin
Flap reconstruction (see below)
Flap Techniques By Region
Flap treatment options determined by location of lesion

Flap Reconstruction Techniques

V-Y advancement flap


o indications
straight or dorsal oblique finger tip lacerations
Digital island artery
o indications
straight or dorsal oblique finger tip lacerations
volar oblique finger tip lacerations
o advantages

best axial pattern flap


Cross finger flap
o indications
volar oblique finger tip lacerations in patients > 30 years
o advantages
leads to less stiffness
Reverse cross finger flap
o indications
dorsal finger & MCP lacerations
Thenar flap
o indications
volar oblique finger tip lacerations to index or middle finger in patients < 30
years
o advantages
improved cosmesis
Axial flag flap from long finger
o indications
volar proximal finger
dorsal proximal finger & MCP lacerations
Moberg advancement volar flap
o indications
volar thumb if < 2 cm
Neurovascular island flap
o indications
volar thumb up to 4 cm
First dorsal metacarpal artery flap
o indications
dorsal thumb lacerations
volar thumb lacerations if > 2 cm
o technique
based on 1st dorsal metacarpal artery
Z-plasty with 60 degrees flaps
o indications
first web space lacerations
o technique
can lead up to 75% increase in length
Posterior interosseous fasciocutaneous flap
o indications
first web space lacerations

Groin flap
o indications
lesions to dorsal hand
Complications
Flap failure
o cause
inadequate arterial flow
vasospasm often leads to thombosis at anastamosis
inadequate venous outflow
Hook nail deformity
o cause
tight tip closure
insufficient bony support
o treatment
variety of reconstructive procedures have been described

The clinical
vignette is consistent with an oblique amputation of the distal phalanx of a thumb with a defect measuring >2.5 cm
Island volar advancement flaps are a safe and effective procedure for single-stage closure of considerably large
thumb defects measuring up to 3.5 cm in length.

The operative technique chosen for reconstruction of distal volar thumb defects depends largely on the size of the
defect. Island volar advancement flaps used for defects up to 3.5 cm are pure island flaps in which all of the
proximal attachments, with the exception of the neurovascular bundles, are divided to provide maximal
advancement. Mobility up to 4 cm can be achieved with elevation of the entire volar skin of the thumb from the
underlying tendon sheath providing a considerable advantage in thumb reconstruction. The island volar
advancement flap is useful for coverage of the entire distal phalanx from the IP joint crease to the nail bed.

Foucher et al. reviewed long-term clinical results of 13 neurovascular palmar advancement flaps for thumb tip
coverage. Specifically, they reported on Moberg and OBrien flaps. The Moberg flap is a pedicled advancement fla
proximally-based on an intact skin pedicle of the thumb including both neurovascular bundles. The OBrien flap is

modification of the Moberg technique which advances a volar flap based on a subcutaneous pedicle including bot
neurovascular bundles by incising the proximal skin and skin grafting the donor site. The study found that both fla
preserved near-normal pulp sensibility, MP and IP joint motion, and grasp and pinch strength. They suggested tha
Moberg and OBrien flaps remain the first choice for coverage of 1-2 cm thumb pulp defects.

Baumeister et al. reported on the functional outcomes of 25 patients that underwent thumb pulp reconstructions
utilizing Moberg volar advancement flaps. They found that 72% of patients had no or only minor subjective
complaints, 74% had normal sensitivity, DASH scores showed only minor impairments, no flaps resulted in
decreased grip strength, and only minor restrictions were identified in active IP joint motion. All defects with a leng
less than or equal to 2 cm were successfully reconstructed, whereas, patients presenting with defects >2 cm
developed complications.

Mutaf et al. reviewed outcomes of 12 patients that underwent thumb reconstruction utilizing an island volar
advancement flap for traumatic distal thumb injuries measuring 3 to 3.5 cm in length. Their results showed that
none of the flaps failed, no patients had limited mobility or scar contractures, near-normal sensation was achieved
excellent recovery of pinch strength occurred, and maximal preservation of thumb length was possible in all
patients.

Figure A and Illustrations A through C represent a case example presented by Mutaf et al. Figure A depicts an
oblique amputation of the distal phalanx of a right thumb. Illustration A reveals elevation of an island volar
advancement flap on both sides of the digital neurovascular bundles in the same thumb. Illustration B reveals flap
advancement and Illustration C reveals a postoperative image of the same thumb 4 months after surgery.

Incorrect Answers:
Answers 1 & 2: Small or superficial defects may be amenable to conservative treatment or local flaps depending o
the location of the defect.
Answers 3 & 4: The Moberg flap with modifications to lengthen distal advancement as necessary is considered a
standard option for medium-sized defects of the thumb pulp less than or equal to 2 cm.

PREFERRE
RESPONSE 5

Distal fingertip amputations can be successfully managed with local wound care and healing by secondary intenti
if no bone is exposed and the soft tissue defects are minimal. This is especially true in the pediatric population.

Distal fingertip amputations are common injuries seen in the emergency department. If bone is not exposed, the
wounds can be successfully treated with local wound care and dressing changes, followed by soaks in a hydroge
peroxide solution after 7-10 days. Some controversy exists in the pediatric population if the soft tissue loss is > 1
cm, with options for management including a V-Y advancement flap or conservative management with dressing
changes.

Quell et al. review the results of 82 patients with fingertip amputations treated conservatively; 31 of the digits were
treated with primary closure with or without shortening of bone and 54 digits were treated with semiocclusive
dressings. No complications were observed, and all healed fingertips were well padded and painless.

Tupper et al. review sixteen patients with twenty fingertip injuries who underwent V-Y plasty for transverse fingerti
amputations. Sensitivity was 73% of normal, with eight patients reporting hypersensitivity. Contrary to popular
belief, they believe normal sensation following a V-Y plasty is not a reasonable expectation.

Illustration A shows the three levels of fingertip amputations. Zone I is distal to the phalanx; Zone II is distal to the
lunula; and Zone III is proximal to the lunula.
Incorrect Answers:
Answer 1: Emergent replantation is not indicated in distal fingertip amputations.
Answer 2: Revision amputation through the DIP joint could be considered for Zone III injuries
Answer 3: Moberg advancement flaps are considered for volar thumb soft tissue loss.
Answer 4: Z-plasty is considered for soft tissue loss in the webspaces.

Figure C shows a dorsal thumb laceration with exposed tendon that would be most appropriately
treated with a first dorsal metacarpal artery (FDMA) flap.
The first dorsal metacarpal artery is a branch of the radial artery that supplies the dorsal hand
skin from the thumb metacarpal to the long metacarpal, as well as the skin on the dorsal surfaces
of the thumb and index to the proximal interphalangeal joint. The flap is raised distal to proximal
as an island flap containing the FDMA, branches of the radial nerve, fascia of the underlying
interosseous muscle of the first web space, and skin overlying the MP joint and proximal phalanx
of the finger. It is an excellent option for large soft tissue defects on either side of the thumb. In
this case, skin grafting is contraindicated because of exposed tendon without paratenon.
Sherif et al. detail the anatomy of the first dorsal metacarpal artery. They found three consistent
branches, including the radial, ulnar, and intermediate branch. In part II of their study, they review
the results of 23 patients where the FDMA flap was used as a fasciocutaneous or fascial flap for
the coverage of soft tissue hand defects.
Illustration A shows a FDMA flap being raised for coverage of a thumb defect.
Incorrect Answers:
Answer 1: Fingertip amputations with minimal soft tissue loss and no exposed bone can be
allowed to heal through secondary intention.
Answer 2: The posterior interosseous fasciocutaneous flap is an excellent option for lacerations
to the first web space.
Answer 4: This large soft tissue defect on the dorsum of the hand may be treated with a groin
flap.
Answer 5: Fingertip amputations with exposed bone are best treated with local advancement
flaps such as a VY advancement flap.

Figure D shows a volar thumb defect which can be best covered with a Moberg advancement
volar flap (if < 2 cm). FDMA (1st dorsal metacarpal artery) and neurovascular island flaps are
typically used to cover larger soft tissue defects of volar aspect of the thumb. FDMA (1st dorsal
metacarpal artery) flaps can also be used for dorsal thumb wounds as shown in Figure B. The
cross-finger flap is a useful heterodigital flap for digital wounds with primarily volar tissue loss
(Figure A). Additionally, several articles have advocated secondary intention healing even if bone
is exposed as discussed in the 2009 OITE question #48. The thenar flap is useful for volar
defects of the index and middle fingers (Figure C). Figure E represents a ring avulsion injury and
it is treated with vessel repair if there is inadequate circulation and the bone, tendon, and nerve
components are intact. Amputation of the digit is chosen if there is inadequate circulation
concomitant with bone, tendon, or nerve injury.
The referenced articles by Martin and Hynes are review articles discussing the treatment options
available for digit injuries. Illustration A shows the planned incisions for a moberg advancement
flap on a volar thumb defect and Illustration B shows the completed Moberg.

PREFERRED RESPONSE 5

In young children with a fingertip amputation, ointment and dressing changes is the most
appropriate treatment even if bone is exposed.
When deciding on a treatment, consideration of a "reconstruction ladder" is helpful in determining
the least invasive procedure to obtain the optimal outcome. The ladder includes primary closure,
healing by secondary intention, split-thickness skin grafts, full-thickness skin grafts, random
pattern local flaps, axial pattern local flaps, island pattern local flaps, distant random pattern
flaps, distant axial pattern flaps, and free tissue transfer.
Lamon et al reviewed 25 patients, with an average age of 30 years old, with fingertip injuries
treated with dressings and warm soaks started 2 days after injury and noted no healing
complications. Only one patient in this cohort had bone exposed.
Soderberg et al performed a Level 3 study of 36 operative and 34 conservatively treated fingertip
amputations with bone exposure and found no benefit to surgery.
Farrell et al conducted a Level 4 review of 21 fingertip amputations with 6 having exposed bone
and concluded that they healed with excellent results in regards to contour, sensation, and finger
length.
Illustration A shows a homodigital island flap. Illustration B shows a thenar flap. Illustration C
shows a volar flap advancement. Illustration D shows a volar V-Y flap advancement.

PREFERRED RESPONSE 2
One of the most commonly used techniques for lengthening scar contracture in hand surgery is
the Z-plasty. When the two 60 degree triangular flaps are transposed and closed, the original
direction of the scar is rotated and the scar length is increased by approximately 75% Because of
its history the 60 degree Z-plasty is the technique to which other methods of contracture
lengthening are compared.
Hove et al describe the technique, various applications, and different types of Z-plasty used
today. Neither the cross-finger flap nor island pedical flap are useful for this amount of scar
release. Two flap Z-plasty with 25 degree limbs does not offer enough lengthening. Split-thicknes
skin grafts are not useful for either lengthening or the volar aspect of the hand due to the
significant contracture they experience.
Illustration A and B depict the Z-plasty technique.

PREFERRED RESPONSE 5
Based on the location of the lesion, a cross-finger flap would be most appropriate.
Cross finger flaps are indicated in patients > 30 years of age when the lesion is a volar oblique
finger tip lacerations or a volar proximal finger lesions. The advantage is it leads to less stiffness.
Martin et al review the treatment options available for digit injuries. They report treatment of
fingertip injuries is a continuous focus of controversy among hand and orthopaedic surgeons.
Different treatment options have been described, depending on the affected segment and finger,
type of lesion, gender and age of the patient, location, size, and depth of the defect.
Fassler et al reviews the proper management of fingertip injuries discussing variables such as
the severity of soft tissue loss and whether bone is exposed.
Incorrect Answers:
Answer 1: Secondary intention healing of this wound is inappropriate due to size and exposed
tendon.
Answer 2: V-Y advancement flaps are for dorsal injuries.
Answer 3: Thenar flaps are good for getting more bulk for distal fingertip injuries.
Answer 4: A Moberg flap is performed on the thumb. A cross-finger flap is a full-thickness flap
useful for volar soft tissue loss distal to PIP.

The image shows a thenar flap. The digit is flexed at the PIPJ and extended at the DIPJ during
the period prior to flap division, leading to PIPJ stiffness and flexion contracture.
Thenar flaps can be used for coverage of digital tip injuries where there is exposed bone or
extensive pulp loss. Advantages include more subcutaneous fat than a cross finger flap, good
color and texture match, and primary closure of the donor site. Other disadvantages include
limited flap size and donor site tenderness. Contraindications include RA, Dupuytrens
contracture and advanced age with degenerative disease as these predispose to joint stiffness.
Fassler et al. reviewed fingertip injuries. The thenar flap can be used for any finger, although the
small finger can be difficult to position comfortably. The flap can be as wide as 2 cm and should
be 1.5 times as wide as the
defect so as to restore the normal rounded contour to the tip. To decrease the amount of PIPJ
flexion required, the MCPJ and DIPJ should be flexed as much as possible.
Figure A demonstrates a thenar flap of the middle finger of the right hand. Illustration A shows a
preoperative image with surgical planning marks on the right hand.
Incorrect Answers:
Answer 1: The flap is attached by a pedicle prior to flap division. After division, there is generally
good flap take because of adequate neovascularization of the flap.
Answer 2: Thenar skin is a good cosmetic match for digital pulp skin (both volar skin).
Answer 3: Injury to the recurrent motor branch of the median nerve is distinctly uncommon with
this flap.
Answer 4: Donor site sensitivity (not recipient site) is a known complication of this flap.

PREFERRED RESPONSE 4
The clinical scenario is consistent with a dorsal thumb avulsion with missing extensor tendon and
exposed bone necessitating soft tissue coverage. The first dorsal metacarpal artery (Kite) flap is
the most appropriate flap for defects of the dorsal aspect of the thumb.
Fassler et al in a Level 5 review state that the first dorsal metacarpal artery (Kite) flap is
appropriate for defects of the dorsal aspect of thumb. The flap is performed in one stage with the
skin over the dorsum of the proximal index finger elevated with incisions on all four sides. An
incision is extended proximally over the dorsum of the first web space, and a pedicle containing
the first dorsal metacarpal artery, the subcutaneous veins, and branches of the dorsal sensory
branch of the radial nerve is isolated. The skin island with the attached pedicle is transferred to
the thumb defect and sutured in place.
Illustration A shows the technical steps of the first dorsal metacarpal artery (Kite) flap. Illustration
B shows the final functional results of the first dorsal metacarpal artery (Kite) flap are shown in
Illustration B.
Incorrect Answers:
Answer 1: Moberg advancement flaps are indicated for volar thumb defects.
Answer 2 & 3: Wet to dry dressings or vaccuum-assisted wound closure would be inappropriate
in this situation.
Answer 5: V-Y advancement flaps are most appropriate for transverse or dorsal oblique fingertip
amputations.

Ring Avulsion Injuries


Authors: Colin Woon, Mark Vitale
Topic updated on 07/19/15 9:59pm

Introductions
Definition
o sudden pull on a finger ring
results in severe soft tissue
injury ranging from
circumferential soft tissue
laceration to complete
amputation
o skin, nerves, vessels are often
damaged
Epidemiology
o incidence
150,000 incidents of amputations and degloving in the US
per year
5% of upper limb injuries
o location
usually only involves 1 digit (with ring)
o risk factors
working with machinery
wearing a ring
Mechanism
o patients catch their wedding band or other finger ring on moving
machinery or protruding object
o long segment of macro- and microscopic vascular injury from
crushing, shearing and avulsion
Prognosis
o outcomes of injury
extent of injury is greater than what it appears to be
poor prognosis because of long segment vascular injury

treatment outcomes
advances in interposition graft techniques have improved
results with ring avulsion replantation

Anatomy
Muscles
o avulsed digits are devoid of muscles and will survive >12h if cooled
Skin
o skin is the finger's strongest soft tissue
once the skin tears, the remaining tissue quickly degloves
Biomechanics
o Urbaniak Class I injuries at 80N of traction force
o Urbaniak Class III injuries at 154N of traction force
o Standard wedding band (3mm wide, regardless of alloy) will not open at
1000N

Presentation
History
o may have history of working with machinery, getting caught in door
Symptoms
o pain
o bleeding
o lack of sensation at tip
Physical exam
o inspection
irrigate wound and inspect for visible avulsed vessel, nerve, tendon, damaged
edges
staggered injury pattern
proximal skin avulsion (from PIPJ to base of digit)
distal bone fracture or dislocation (distal to PIPJ, often at DIPJ level)
Imaging
Radiographs
o recommended views
Xray both segments (the amputated part, if present, and the remaining digit)
Treatment
Initial

place amputated part, if present, in bag with saline-moistened gauze, followed by bag
ice water
o antibiotics and tetanus prophylaxis
Operative
o

replantation +/- vein graft, DIPJ fusion


indications
disruption of venous drainage only
disruption of venous and arterial flow (requires revascularization)
requires intact PIPJ and FDS insertion
contraindication
complete amputation (especially distal to PIPJ and FDS insertion) is rela
contraindication to replantation
outcomes
survival
lower overall survival for avulsed digits replantation (60%) than fin
replantation in general (90%)
lower survival for complete (66%) vs incomplete avulsion replanta
(78%)
lower survival for avulsed thumb (68%) than finger (78%) replanta
surgeons more likely to attempt technically difficult avulsed
thumb replantation where conditions not favorable because
importance of thumb to hand function (unlike other digits,
where revision amputation would be performed instead)
sensibility
most achieve protective sensibility (2PD 9mm)
better sensibility with incomplete avulsion replantation (8mm) tha
complete (10mm)
range of motion
average total arc of motion (TAM) of 170-200 degrees
better TAM with incomplete avulsion replantation (199 degrees) t
complete (174 degrees)
revision amputation
indications
complete degloving
bony injury with nerve and vessel injury
bony amputation proximal to FDS insertion or proximal to PIPJ
replantation likely to leave poor hand function
consider revision amputation or ray amputation

Surgical Technique
Replantation/revascularization
o approach
under operating microscope mid-lateral approach to digit
o technique
arteries
thorough debridement of nonviable tissue
thorough arterial debridement (inadequate debridement leads to failure)
repair using vein grafts because of significant vascular damage
may need another step-down vein graft because of difficulty in arterial si
matching (small artery, large vein graft)
may reroute arterial pedicle from adjacent digit
disadvantage is this sacrifices major artery from adjacent digit
veins
repair at least 2 veins

important factor in revascularization failure


bone
if amputation occurs at DIPJ, perform primary arthrodesis of DIPJ
skin

perform full-thickness skin grafts or venous flaps to prevent tight closure


may utilize commercially available synthetic acellular dermal matrix.
Complications

Complications of replantation
o cold intolerance (70%)
o revascularization/replantation failure
factors include
most significant factor is repair of <2 veins
vascular damage up to digital pulp
smoking and level of bone injury have not been found to affect surviva
o flexion contracture
o malunion
o revision surgery
Complications of revision amputation
o hyperaesthesia

Replantation
Author: Evan Watts
Topic updated on 01/06/16 1:20am

ntroduction
Trauma is the most common etiology for
upper extremity replantation
Epidemiology
o incidence
90% of upper extremity
amputation occurred after
trauma
o demographics
4:1 male-to-female ratio
o location
most amputations occur at the level of the digits
Pathophysiology
o mechanism of traumatic amputation
sharp dissection
blunt dissection
avulsion
crush
Presentation

History
o timing of injury
o type and location of amputation
number of digits involved
o preservation of amputated tissue
o associated injury
o past medical history
Examination
o stump examined for
zone of injury
tissue viability
supporting tissue structures
contamination
o amputated portion inspected
segmental injury
bone and soft tissue envelope
contamination
Indications
Indications for replantation after trauma
o primary indications
thumb at any level
multiple digits
through the palm
wrist level or proximal to wrist
almost all parts in children
o relative indications
individual digits distal to the insertion of flexor digitorum
superficialis [FDS] (Zone I)
ring avulsion
through or above elbow
Contraindications to replantation
o primary contraindications
severe vascular disorder

mangled limb or crush injury


segmental amputation
prolonged ischemia time with large muscle content (>6 hours)
relative contraindications
single digit proximal to FDS insertion (Zone II)
medically unstable patient
disabling psychiatric illness
tissue contamination

prolonged ischemia time with no muscle content (>12 hours)


Treatment
Transport of amputated tissue
o indications
any salvageable tissue should be transported with the
patient to hospital
o modality
keep amputated tissue wrapped in moist gauze in lactate
ringers solution
place in sealed plastic bag and place in ice water (avoid
direct ice or dry ice)
wrap, cover and compress stump with moistened gauze
Operative
o time to replantation
proximal to carpus
warm ischemia time < 6 hours
cold ischemia time < 12 hours
distal to carpus (digit)
warm ischemia time < 12 hours
cold ischemia time < 24 hours
o general operative sequence of replantation (beavfns)
1. bone fixation +/- shortening (after irrigation and debridement
of soft-tissue and bone)
2. extensor tendon repair
3. artery repair(repair second after bone if ischemic time is >34 hours)
4. venous anastomoses
5. flexor tendon repair
6. nerve repair
7. skin +/- fasciotomy
o finger order
thumb, long, ring, small, index
o for multiple amputations structure-by-structure sequence is most
efficient
digit-by-digit sequence takes the most time
Postoperative Care
Environment
o keep patient in warm room (80F)
o avoid caffeine, chocolate, and nicotine
Replant monitoring
o skin temperature most reliable

concerning changes include a > 2 drop in skin temp in less


than one hour or a temperature below 30 celsius
o pulse oximetry
< 94% indicates potential vascular compromise
Anticoagulation
o adequate hydration
o medications (aspirin, dipyridamile, low-molecular weight dextram,
heparin)
Arterial Insufficiency
o treat with
release constricting bandages
place extremity in dependent position
consider heparinization
consider stellate ganglion blockade
early surgical exploration if previous measures unsuccessful
o thrombosis secondary to vasospasm is most common cause of
early replant failure
Venous congestion
o treatment
elevate extremity
leech application
releases Hirudin (powerful anticoagulant)
Aeromonos hydrophila infection can occur (prophylax
with Bactrim or ciprofloxacin)
heparin soaked pledgets if leeches not available
Complications
Replantation failure
o most frequently cause within 12 hours is arterial thrombosis from
persistent vasospasm
Stiffness
o replanted digits have 50% of total motion
o tenolysis is most common secondary surgery
Myonecrosis
o greater concern in major limb replantation than in digit replantation
Myoglobinuria
o caused by muscle necrosis in larger replants (forearm and arm)
o can lead to renal failure and be fatal
Reperfusion injury
o mechanism thought to be related to ischemia-induced
hypoxanthine conversion to xanthine
o allopurinol is the best adjunctive therapy agent to decrease
xanthine production

Infection
Cold intolerance

PREFERRED RESPONSE 4
The unique functional role of the thumb in opposition and pinch dictates that it be replanted
whenever possible in a healthy patient, regardless of the level of amputation. The remainder of
the answer choices are relative contraindications for digit replantation.
Boulas et al outline indications and contraindications for digit replantation after traumatic
amputation. Contraindications to replantation include multilevel or segmental injury, a single digit
proximal to the FDS insertion, a severe crush or mangling injury, extreme contamination, prior
impaired function, concomitant life-threatening injury, severe medical problems, anesthetic risk,
and major psychiatric disorder.
Waikakul et al determined the influencing factors of the immediate and late outcome of
replantation and revascularization of amputated digits. They found that the type of injury was the
most important factor influencing immediate and late outcomes. They also determined that
connecting the profundus tendon stump of the proximal part to the superficialis tendon of the
amputated part gave a better result than two tendon repair and repairing only the profundus
tendon.
Illustration A shows a clinical photo of an isolated amputation of the thumb proximal to the FPL
insertion.
Incorrect Answers:
Answer 1: Replantation of a single digit proximal to the FDS insertion is associatedwith poor
results related to the loss of PIP joint motion due to flexor sheath adhesion formation.
Answer 2: Crush or mangling injury is associated with serious damage to tissues, which are at
risk for infection, problematic healing, and scarring, thereby contributing to a poor outcome.
Answer 3: Segmental injury to the index finger is a contraindication to replantation due to poor
function post-operatively.
Answer 5: Prolonged warm ischemia time, defined as more than 12 hours, is associated with
replantation failure.

PREFERRED RESPONSE 5
Wrist-proximal amputations should be performed before 12 hours of cold ischemia time or 6
hours of warm ischemia time have elapsed.
Wrist-proximal replantation should be strongly considered for patients in whom the mechanism
allows adequate debridement, the cold ischemia time is less than 12 hours, and whose general
health and comorbidities allow the patient to tolerate an extended surgical procedure. In general,
amputation at the distal forearm and wrist have excellent functional results with replantation
when compared to amputations at other levels. Similar to other major amputations, replantation
should proceed in the following sequence: 1) bone, 2) extensor tendons, 3) flexor tendons, 4)
arteries, 5) nerves, 6) veins (can be done prior to nerve repair) , 7) skin.
Sabapathy et al. review replantation surgery in the upper extremity. They discuss that a
functional extremity could be reconstructed at the upper-arm level in 22% to 34%, at the

proximal forearm level in 30% to 41%, and at the distal forearm level in 56% to 80% of cases.
Hanel et al. review wrist level and proximal amputations in the upper extremity. Among other
things, they state that wrist-proximal amputations should be performed before 12 hours of cold
ischemia time or 6 hours of warm ischemia time have elapsed.
Figure A shows a sharp transcarpal amputation.
Incorrect Answers:
Answer 1: Replantation at the distal forearm and wrist have better functional outcomes than
above the elbow replantations.
Answer 2: Arteries should be reconstructed prior to veins.
Answer 3: Bony stabilization should be performed at the beginning of the procedure.
Answer 4: Vein grafting should be used generously.

PREFERRED RESPONSE 5
The clinical scenario and images depict a through the palm amputation with the digits intact. The
injury is a clean amputation with minimal avulsion. Thus, anatomic replantation of the entire hand

is indicated. Digit transposition refers to using the salvageable digits and replanting them on the
functionally important positions in the hand. Transpositional replantation is not indicated in the
above scenario and would be more appropriate for a multidigit amputation as shown in
Illustration A.
Soucacos et al reviewed their results of transpositional digital microsurgery in 34 patients. They
found that transposition of a digit to the most functional part of the hand lead to a 2-point
discrimination of 10-14mm in transposed digits and equivalent functionality of transposed digits
with anatomically replanted digits.
Schwabegger et al presented a case series of 13 patients with multiple digit amputations. The
primary goal of surgery was function and secondly, cosmesis. They found the results of
transpositional replantation similar to conventional replantation.

PREFERRED RESPONSE 4
As outlined by Pederson, the contraindications to replantation are more relative than the
indications, but they include the following: Single-finger replantations at the level of zone II (from
the A1 pulley to the distal sublimis tendon insertion) are rarely indicated, with the notable
exception of the thumb. Amputated parts that are severely crushed and those with multiple level
injuries have poor function even if they survive replantation. While ring avulsion injuries with a
vascular injury and no bone, tendon or nerve injury (Urbaniak type 2A ring avulsion injuries)
should be repaired, ring avulsion injuries with bone, tendon or nerve injury (Urbaniak type 2B) or
with complete degloving (Urbaniak type 3) have poor outcomes and Urbaniak and colleagues

recommend amputation for such injuries. Very distal amputations at the level of the nail bed are
marginally indicated as there needs to be approximately 4 mm of intact skin proximal to the
nailfold for adequate veins to be present. Indications for replantation that rule out the other 4
choices of this question include the following: Overall, thumb replantation probably offers the best
functional return. Even with poor motion and sensation, the thumb is useful to the patient as a
post for opposition. A replanted thumb offers the best reconstruction available, toe transfers
notwithstanding. Replantation beyond the level of the sublimis tendon insertion (zone I) usually
results in good function. Multiple finger amputations present reconstructive difficulties that may
be difficult to correct without replantation of one or all of the amputated digits. Any hand
amputation from zone III (distally) to zone V (proximally) offers the chance of reasonable function
after replantation, usually superior to available prostheses. Although usually indicated, the
replantation of any hand or arm proximal to the level of the mid-forearm must be carefully
considered.

PREFERRED RESPONSE 4
As reviewed by Soucacos, there are several major indications for single digit replantation: 1)
Level of the amputation is distal to the insertion of FDS. 2) Amputations at the level of the distal
phalanx. 3) Ring avulsion injuries involving both the dorsal and palmar skin and blood supply in
an isolated finger, as long as FDS is intact. 4) Any amputation in a child. 5) Thumb amputation.
Replantation of a single digit, which is amputated at the level of the proximal phalanx or at the
PIP joint, particularly in avulsion or crush injury is contra-indicated. Soucacos also discusses
appropriate surgical teams, transport, and other related issues surrounding a "transplant team."

PREFERRED RESPONSE 5
The Level 2 study by Waikakul et al is a cohort of 552 patients that underwent 1018 digit
replantation. Mechanism of injury was the most important factor influencing the survival rate with
an odds ratio of 46.3. Specifically, avulsion, degloving and extensive crushed amputation
resulted in a low survival rate and poor functional outcome. Cigarette smoking and male gender
were associated with worse results but not to the degree of the mechanism of injury. Utilization of
composite skin and subcutaneous vein grafts as well as connecting the profundus to the
superficialis at the anastomoses correlated to better outcomes. After the operation, 329 of 946
survived digits (34.7%) in 180 patients (35.4%) needed further reconstructive surgery to improve
their function. Tenolysis was the most common procedure followed by staged tendon grafting
and capsulotomy.
The review article by Wang found that tendon procedures, specifically tenolysis, accounted for
47.2% secondary surgeries following digit replantation.

PREFERRED RESPONSE 4
Allopurinol is a xanthine oxidase inhibitor and may have a beneficial role in replantation.
Inhibition of xanthine oxidase also decreases uric acid in patients with gout.

Waikakul et al. published a randomized control trial with a 2-year follow-up comparing thumb
replantation with and without adjunctive allopurinol. There were 60 patients in the trial group, and
38 patients in the control group. All were young, healthy laborers who had sharp or locally
crushed amputations of the thumb at the proximal phalanx with a total ischemic time >10 hours.
The standard management for thumb replantation was used in these patients, except that 300mg
allopurinol was given orally in the trial group on admission and a further 300mg for another 5
days. After the operation, the trial group had a lower infection rate, and less postoperative pain
and chronic swelling than the control group. Recovery of sensation was also better in the trial
group.

PREFERRED RESPONSE 1
Replantation of a single finger amputated proximal to the insertion of the flexor superficialis
tendon is a relative contraindication because of the severe stiffness and poor function
encountered after repairs in this location.
The FDS insertion is in the middle of the middle phalanx and is also what defines the distal
border of Zone II in flexor tendon injuries. If the finger is cut proximal to the insertion of FDS, that
means that FDS is also cut and needs to be repaired leading to severe stiffness and worse PIP
function. The exceptions are when there are multiple digits injured or in young children.
Urbaniak et al found replantation of a single finger amputated distal to the insertion of the flexor
superficialis tendon is justified, but that replantation of a single finger that was amputated
proximal to this insertion is not indicated.
Tamai et al found that an amputation of the hand, forearm, arm, and thumb, as well as multiple
digits are all criteria for replantation.

PREFERRED RESPONSE 2
Waikakul et al. in a study of 1018 replantations found that type of injury was the most important
factor influencing immediate and late outcome. Extensively crushed injuries had the worst
outcome, followed by degloving and avulsion injuries. Sharp cut injuries fared the best. Regular
cigarette smoking resulted in poor immediate survival rate and prolonged ischemia had a
significant influence in final functional outcome, but neither were as important as mechanism of
injury. Alcohol consumption was also a negative predictor. Favorable factors for replantation
survival were female gender, age under 13 years old, and nonsmokers. Regarding ischemia
time, Miller recommends <12 hours of warm ischemia or <24 hours of cold ischemia for a digit to
obtain optimal outcomes.

PREFERRED RESPONSE 3
Arterial thrombosis after digit replantation typically occurs within the first 12 hours postoperatively
whereas venous thrombosis/congestion occurs after the first 12 hours postoperatively. Leeches
excrete Hirudin, which is 100 times more potent than heparin, but are typically used for the
treatment of venous thrombosis/congestion and not arterial thrombosis.
Miller's review states a drop in temperature >2 C in 1 hour or temperature below 30 C indicates
decreased digital perfusion. If arterial insufficiency develops: release constrictive bandages,

place the extremity in a dependent position, consider heparinization, consider stellate ganglion
blockade, or explore early if these maneuvers do not work.

PREFERRED RESPONSE 2
Surgical time in multiple digit replantation is increased by digit-by-digit repair techniques and
decreased by structure-by-structure repair techniques.
The Level 5 article by Morrison and McCombe reviews the indications and results of finger
replantation. Results of replantation from the DIP to PIP joint typically have good outcomes
whereas replantations at the proximal interphalangeal (PIP) joint to MCP joint have poor
outcomes due to flexion contractures. The review article by Wang cites that tenolysis and tendon
procedures were needed in 47.2% of the published cases of digit replantation and is the leading
type of secondary operation.
The classic article by Waikakul et al reviewed 1018 digital replantations in 552 patients. They had
a 92% rate of successful outcome and found that type of injury was the most important factor
influencing immediate and late outcome.

Thumb Reconstruction
Author: Amiethab Aiyer
Topic updated on 03/05/16 8:47am

Introduction

Reconstruction of the thumb requires an intact carpometarcarpal joint


that not only is stable, but is appropriately functional.

Treatment
Toe to thumb procedure
o great toe receives blood supply from the first dorsal metatarsal
artery and dorsalis pedis
The Morrison/wrap around flap allow for maintenance of
length of the hallux. Size and appearance are best
replicated.
o second toe is not as stable for transfer
Vascular pedicle can be based on
dorsalis pedis /1st dorsal metatarsal artery
2nd dorsal metatarsal artery
Web deepening
o Z plasty (2 or 4 flap)
2 flaps provide greater depth
if completed at 45 degrees, relative length is increased by
50%; 60 degrees leads to an increase in length of 75%
o Brand flap
index finger is used to provide a full thickness

(dermoepidermal flap)
can close the donor site primarily
o Dorsal rectangular flaps
Take from dorsum of metacarpals
May require skin grafting
o Arterialized palmar flap
o May use axial or island flaps (locally vs distally)
Osteoplastic reconstruction
o Iliac crest is used to establish mechanical length to the thumb
o an island flap from the radial aspect of the 4th ray is combined
with a reverse radial forearm flap to aid in coverage

Peripheral Nerves Injury & Repair


Author: Derek Moore
Topic updated on 01/03/16 4:28am

Introduction
Mechanism
o

stretching injury
8% elongation will diminish nerve's microcirculation
15% elongation will disrupt axons
examples
"stingers" refer to neurapraxia from brachial plexus
stretch injury
suprascapular nerve stretching injuries in volley ball
players
correction of valgus in TKA leading to peroneal nerve
palsy
compression/crush
fibers are deformed
local ischemia
increased vascular permeability
endoneurial edema leads to poor axonal transport and nerve
dysfunction
fibroblasts invade if compression persists
scar impairs fascicular gliding
30mm Hg can cause paresthesias
increased latencies
60 mm Hg can cause complete block of conduction
laceration
sharp transections have better prognosis than crush injuries
continuity of nerve disrupted
ends retract

nerve stops producing neurotransmitters


nerve starts producing proteins for axonal regeneration
Pathophysiology
o regeneration process after transection
distal segment undergoes Wallerian degeneration (axoplasm
and myelin are degraded distally by phagocytes)
existing Schwann cells proliferate and line up on basement
membrane
proximal budding (occurs after 1 month delay) leads to
sprouting axons that migrate at 1mm/day to connect to the
distal tube
o variables affecting regeneration
contact guidance with attraction to the basal lamina of
the Schwann cell
neurotropism
neurotrophism
neurotrophic factors (factors enhancing growth and
preferential attraction to other nerves rather than other
tissues)
Prognosis
o factors affecting success of recovery following repair
age
is single most important factor influencing success of
nerve recovery
level of injury

is second most important (the more distal the injury the


better the chance of recovery)
sharp transections
have better prognosis than crush injuries
repair delay
worsen prognosis of recovery (time limit for repair is 18
months)
return of function
pain is first modality to return

Anatomy
Highly organized structure consisting of nerve fibers, blood vessels, and
connective tissue
Functional structures
o epineural sheath
surrounds peripheral nerve
o epineurium
surrounds a group of fascicles to form peripheral nerve
functions to cushion fascicles against external pressure

perineurium
connective tissue covering individual fascicles
primary source of tensile strength and elasticity of a
peripheral nerve
provides extension of the blood-brain barrier
provides a connective tissue sheath around each nerve
fascicle
o fascicles
a group of axons and surrounding endoneurium
o endoneurium
fibrous tissue covering axons
participates in the formation of Schwann cell tube
o myelin
made by Schwann cells
functions to increase conduction velocity
o neuron cell
cell body - the metabolic center that makes up < 10% of cell
mass
axon - primary conducting vehicle
dendrites - thin branching processes that receive input from
surrounding nerve cells
Blood supply
o extrinsic vessels
run in loose connective tissue surrounding nerve trunk
o intrinsic vessels
plexus lies in epineurium, perineurium, and endoneurium
Physiology
o presynaptic terminal & depolarization
electrical impulse transmitted to other neurons or effector
organs at presynaptic terminal
resting potential established from unequal distribution of ions
on either side of the neuron membrane (lipid bilayer)
action potential transmitted by depolarization of resting
potential
caused by influx of Na across membrane through three types
of Na channels
voltage gate channels
mechanical gated channels
chemical-transmitter gated channels
o nerve fiber types
o

Classification
Seddon Classification
o neurapraxia
same as Sunderland 1st degree, "focal nerve
compression"
nerve contusion leading to reversible conduction block
without Wallerian degeneration
histology
histopathology shows focal demyelination of the
axon sheath (all structures remain intact)
usually caused by local ischemia
electrophysiologic studies
nerve conduction velocity slowing or a complete
conduction block
no fibrillation potentials
prognosis
recovery prognosis is excellent
o axonotmesis
same as Sunderland 2nd degree
axon and myelin sheath disruption leads to conduction
block with Wallerian degeneration
endoneurium remains intact
fibrillations and positive sharp waves on EMG
o neurotmesis
complete nerve division with disruption of endoneurium
no recovery unless surgical repair performed
fibrillations and positive sharp waves on EMG

Sunderland Classification
o 1st degree
same as neurapraxia
o 2nd degree
same as axonotmesis
o 3rd degree

injury with endoneurial scarring


most variable degree of ultimate recovery
4th degree
nerve in continuity but at the level of injury there is complete
scarring across the nerve)
5th degree
same as neurotmesis

Evaluation

EMG
o
o

often the only objective evidence of a compressive neuropathy (valuable


in workcomp patients with secondary gain issues)
characteristic findings
denervation of muscle
fibrillations
positive sharp waves (PSW)
fasiculations
neurogenic lesions
fasiculations
myokymic potentials
myopathies
complex repetitive discharges
myotonic discharges

NCV
o

focal compression / demyelination leads to


increase latencies (slowing) of NCV
distal sensory latency of > 3.2 ms are abnormal for CTS
motor latencies > 4.3 ms are abnormal for CTS
decreased conduction velocities less specific that latencies
velocity of < 52 m/sec is abnormal
motor action potential (MAP) decreases in amplitude
sensory nerve action potential (SNAP) decreases in amplitude

Treatment
Nonoperative
o observation with sequential EMG
indications
neuropraxia (1st degree)

axonotmesis (2nd degree)


Operative
o surgical repair
indications
neurotomesis (3rd degree)
o nerve grafting
indications

defects > 2.5 cm


type of autograft (sural, saphenous, lateral antebrachial, etc)
no effect on functional recovery
Surgical Techniques
Direct muscular neurotization
o insert proximal nerve stump into affected muscle belly
o results in less than normal function but is indicated in certain cases
Epineural Repair
o primary repair of the epineurium in a tension free fashion
o first resect proximal neuroma and distal glioma
o it is critical to properly align nerve ends during repair to maximize
potential of recovery
Fasicular repair
o indications
three indications exist for grouped fascicular repair
median nerve in distal third of forearm
ulnar nerve in distal third of forearm
sciatic nerve in thigh
o technique
similar to epineural repair, but in addition repair the perineural
sheaths (individual fascicles are approximated under a
microscope)
o outcomes
no improved results have been demonstrated over epineural repair
Nerve grafting
o autologous graft
remains the gold standard of repair for segmental defects > 5cm is
autologous nerve grafting
o allograft
the only synthetic graft which shows equal results to autologous
nerve grafting is a collagen conduit when the gap is 5mm or less
collagen conduits allow for nutrient exchange and accessibility of
neurotrophic factors to the axonal growth zone during regeneration

PREFERRED RESPONSE 4
Axonomesis is a disruption of the nerve axon following injury. Repair/regeneration of the nerve
occurs via proximal budding, followed by antegrade (or distal) axon migration.
The peripheral nerve regeneration process begins with the distal segment undergoing Wallerian
degeneration (axoplasm and myelin are degraded distally by phagocytes). Existing Schwann
cells proliferate and line-up along the basement membrane. Proximal budding occurs after a onemonth delay. This is followed by sprouting axons that migrate in an antegrade fashion to connect
to the distal tube. Repair of the nerve can take months, and often have poor outcomes.
Lee et al. reviewed peripheral never injury and repair. They commented that Wallerian
degeneration (i.e., breakdown of the axon distal to the site of injury) is initiated 48 to 96 hours
after transection. The Schwann cells then align themselves longitudinally, creating columns of
cells called Bngner bands. At the tip of the regenerating axon is the growth cone.
Illustration A shows a chart of peripheral nerve injury. The two main classification systems are
Seddon and Sunderland. Video V is a lecture discussing peripheral nerve injury and
management.
Incorrect Answers:
Answer 1: The distal nerve segment undergoes Wallerian degeneration.
Answer 2: Axon growth occurs from the proximal to distal segment.
Answer 3: Neurotrophic factors do not direct phagocytic activity.
Answer 5: Schwann cells do not degrade axoplasm and myelin.

PREFERRED RESPONSE 5
The clinical scenario describes an ulnar nerve laceration. Studies have shown that the ulnar
nerve does not typically have good outcomes after nerve repair. (worse recovery than repairs of
the tibial, radial, femoral, and musculocutaneous nerves)

Nerve injuries from gunshot injuries (GSWs) can cause both a direct injury to the nerve as well
as surrounding structures (zone of injury). Many factors including age of patient, time to repair,
repair level, and length of repair have been shown to be important determinants in nerve
recovery following repair. The type of nerve graft (sural, saphenous, etc) used has not shown to
be statistically significant in terms of functional recovery after nerve repair.
Secer et al.(J. Neurosurg) reviewed 2210 peripheral nerve lesions in 2106 patients which were
injured by a GSW and who were treated surgically. Of the peripheral nerves repaired surgically,
the tibial, median, and femoral nerve lesions showed the best recovery rate. The deep peroneal
nerve, ulnar nerve, and brachial plexus lesions had the worst recovery.
Secer et al.(Surg. Neur.) found that of 455 patients with 462 ulnar nerve lesions only a good
outcome was noted in 15.06% of patients who underwent high-level repair, 29.60% of patients
who underwent intermediate-level repair, and 49.68% of patients after low-level repair. The
authors also noted that a better functional recovery was noted in patients who were treated
earlier.
Figure A shows a distal humerus fracture caused by a GSW.
Incorrect Answers
Answer 1: Earlier nerve repairs typically have better functional results.
Answer 2: The lower level of nerve repair (more distal), the better functional results.
Answer 3: Shorter length of the nerve repair typically leads to better functional results.
Answer 4: Pre and post operative physical rehabilitation after nerve repairs has been shown to
have better results.

PREFERRED RESPONSE 3
The history and clinical presentation are consistent with ulnar entrapment neuropathy at the level
of the cubital tunnel. This would be classified as a neuropraxia with ischemia origin.
Compression injuries to the peripheral nerves are often the result of micro-vascular dysfunction
as the nerves traverse a high to low pressure gradient. Peripheral nerve injury can be classified

as neuropraxia, axonotmesis and neurotmesis. Compressive neuropathies are typically


neuropraxias, with local myelin damage but not compromise of the major components of the
nerve. In axonotmesis, there is Wallerian degeneration and myelin loss distal to the site of injury.
The most severe type is that of neurotmesis. Neurotmesis is composed of a spectrum of injury, in
which all components are affected except for the perineurium or the endoneurium may be intact.
The worst form of neurotmesis is that of nerve transection.
Elhassan et al. review the pathophysiology of cubital tunnel syndrome. They report nerve
dysfunction results from ischemic changes secondary to compression. Compressive effects on
the nerves can last greater than 24 hours, even after the source of compression has been
removed.
Rempel et al. review the pathophysiology of peripheral nerve compression syndromes. The
authors indicate that deforming pressures to nerves are often the result of stenotic soft tissue
canal boundaries. This leads to interference with local microvasculature of the nerve itself.
Incorrect Answers:
Answer 1, 2: Axonotmesis is considered a second degree nerve injury, characterized by
Wallerian degeneration of axons distal to site of injury. Compression neuropathies are more often
neuropraxias (1st degree nerve injury)
Answer 4: Compression neuropathies result from ischemic insult to the nerve
Answer 5: Neurotmesis may be characterized by complete disruption of all components of nerve
(as in transection) or with disruption of all components except for the perineurium or the
endoneurium. This is not characteristic of a compression neuropathy such as cubital tunnel
syndrome.
Illustration A demonstrates the Wartenberg sign, where the patient has persistent small finger
abduction/extension resulting from weakness of the 3rd palmar interosseous/small finger
lumbrical.
Illustration B reveals clawing which results from over powering of the intrinsic muscles by the
extensors; a tenodesis effect results in flexion of the PIP/DIP joints. This is more severe in ulnar
nerve compression at Guyons canal.
Illustration C shows the Froment sign, where the FPL attempts to compensate for a deficient
pinch, because of weakness of the adductor pollicis. Illustration D demonstrates atrophy of the
1st dorsal webspace from chronic compressive changes.
Illustration E demonstrates atrophy of the thenar compartment which is consistent with carpal
tunnel syndrome

PREFERRED RESPONSE 2
Repair of segmental nerve loss in the hand using collagen conduits allows for nutrient exchange
and accessibility of neurotrophic factors to the axonal growth zone during regeneration. While the
other listed answers have been used, none has shown the efficacy of collagen conduits or
autograft.
Li et al. describe the repair of peripheral nerves with a tubular collagen conduit and review
supporting data from in vitro and in vivo primate studies to this regard.
Bertleff et al. describe the recovery of sensory nerve function after treatment of traumatic
peripheral nerve lesions with a biodegradable poly(DL-lactide-epsilon-caprolactone) neurolac
nerve guide, compared to their control of end-to-end repair, no autologous grafting. They show
equal results between primary end-to-end repair and their synthetic graft.
Waitayawinyu et al. compared 2 synthetic polyglycolic acid conduits to autogenous nerve grafting
using histopathologic and neurophysiologic analyses in a segmental defect rat model. They
found that collagen conduits and autografts produced comparable results, which were
significantly better than polyglycolic acid conduits.

PREFERRED RESPONSE 3
Of the choices listed, the radial nerve has the best opportunity for recovery.
Roganovic performed a prospective study of 393 graft repairs of the median, ulnar, radial, tibial,
peroneal, femoral, and musculocutaneous nerves which showed that peripheral nerves differ
significantly regarding the motor recovery potential, and the difference depends on the level of
nerve repair. The following nerves had excellent recovery potential: the radial,
musculocutaneous, and femoral nerves. The following nerves had moderate recovery potential:
the median, ulnar, and tibial nerves. The following nerve had poor recovery potential: the
peroneal nerve.
Mohler et al, recommends testing nerve action potentials at the time of nerve exploration to guide
surgical treatment.

PREFERRED RESPONSE 2
Following a Sunderland second-degree injury, axon regeneration is possible because the
endoneurium is intact.
There are two classification schemes for peripheral nerve injuries, which include the Seddon and
the Sunderland systems. Under the Sunderland classification, a second-degree injury is
considered a part of the axonotmesis spectrum. The endoneurium, perineurium and epineurium

are still intact. This enables complete functional recovery.


Lee et al. review the pathophysiology and evaluation of peripheral nerve injuries. They note that
in Sunderland type two injuries, there is physiologic disruption of the axons. Because the
endoneurium is still intact, axons are able to regenerate. This process takes months.
Illustration A is a schematic of the various stages of peripheral nerve injury.
Incorrect Answers
Answers 1, 3: Although the epineurium and perineurium are intact in a Sunderland type 2 injury,
axon regeneration is possible because of an intact endoneurium.
Answers 4, 5: The myelin sheath and Schwann cells are disrupted in axonotmesis.

PREFERRED RESPONSE 2
Vitamin B12 deficiency is a known cause of peripheral sensory neuropathy and B12 levels
should be evaluated in patients presenting with peripheral sensory neuropathy. It is associated

with decreased deep tendon reflexes, pathologic reflexes like Babinski's sign, and
fatigue/depression. The inability to whistle is associated with fascioscapular dystrophy.
Hydrophobia is associated with rabies infection.
Smith and Singleton evaluated 138 patients referred with predominantly sensory symptoms to
identify a standardized approach to diagnosis. They recommend that patients be tested for
glucose tolerance and vitamin B(12) concentration in all cases, but that other tests should be
performed only when the clinical scenario is suggestive.
Steiner et al. describe a case report of a patient with vitamin B12 sensory peripheral neuropathy
and associated EMG evidence of nerve demyelination as the potential cause for the observed
clinical symptoms.

PREFERRED RESPONSE 3
Merkel's skin receptors are slowly adapting skin receptors that detect pressure, texture, and low
frequency vibration and can be appropriately evaluated by static two-point discrimination.
Merkel's disk receptors adapt slowly and sense sustained pressure, texture, and low-frequency
vibrations.
Szabo et al state in their review that static and moving two point discrimination are best to initially
evaluate innervation density for both quickly and slowly adapting fibers. Vibratory moving 2 point
discrimination is best for evaluation of quickly adapting fibers.
Meissner corpuscle, a rapidly adapting sensory receptor, is very sensitive to touch. Pacinian
corpuscles are ovoid in shape, measuring approximately 1 mm in length. They respond to highfrequency vibration and rapid indentations of the skin. Ruffini corpuscles are slowly adapting
receptors that detect stretching of the skin.
Illustration A demonstrates Meissner's corpuscles (A), Pacinian corpuscles (B), Merkel's receptor
(C), free nerve ending (D), and Ruffini corpuscles (E). Illustration B displays the function and
location of the receptors.

PREFERRED RESPONSE 4
The epineurium is a supportive sheath surrounding peripheral nerves that cushions fascicles
against external pressure. It is comprised of a loose meshwork of collagen and elastin fibers that
are aligned parallel with the nerve fibers.
Illustration A & B depicts the contents of a nerve including epineurium, perineurium, and
endoneurium.
Incorrect Answers:
Answer 1: Endoneurium is a fibrous tissue that covers the axon, Schwann cell, and myelin of
each nerve fiber.
Answer 2: Fibronectin and laminin are extracellular matrix glycoproteins that facilitate directional
nerve fiber branching.
Answer 3: N-cadherin is an adhesive membrane glycoproteins on neural ectoderm and facilitate
growing axons.
Answer 4: Perineurium is a dense connective tissue which surrounds nerve fascicles. It provides
high tensile strength. The perineurium also limits diffusion within the intraneural environment and
subsequently prevents injury from edema.

Extremity Flap Reconstruction


Author: Mark Karadsheh
Topic updated on 08/29/15 1:15pm

Introduction
Definition of flap
unit of tissue transferred from a donor site to a recipient site while
maintaining its own vascular supply
Definition of pedicle
o vascular portion of the transferred tissue
o usually contains one artery and one or more veins
Indications for flap coverage
o soft tissue injury with exposed
bone
tendons
cartilage
orthopaedic implants
Prognosis
o free tissue transfer within 72 hours for severe trauma in the upper
extremity has been shown to decrease complication rates
o

Classification

Blood supply classification


o axial pattern local flaps
contain single arteriovenous pedicle (a "named vessel")
indications
primary/secondary closure not advisable and
area cannot support STSG or FTSG and
length-width ratio needed > 2:1
o random pattern flaps
supported by numerous microcirculation with no single
arteriovenous pedicle
indications
primary/secondary closure not advisable andarea
cannot support STSG or FTSG and
length-width ratio needed < 2:1
o venous flap
uses veins as inflow and outflow of arterial blood
Tissue type classification
o cutaneous
include skin and subcutaneous tissue
o fascial flap
include fascia with no overlying skin
example
temporoparietal flap
o muscle flaps
usually requires additional transfer of a skin graft to cover
muscle
alternatively, muscle can be transposed as part of a
musculocutaneous flap (composite flap)
if motor nerve is not preserved the flap will atrophy to 50%
of its original size
o bone flaps
free fibula
based on peroneal artery pedicle
useful for diaphyseal reconstruction
free iliac crest
based on deep circumflex iliac vessels
useful for metaphyseal reconstuction
o composite flaps
consists of multiple tissue types
examples
radial forearm flap (fasciocutaneous)
Mobilization type classification
o local flap
tissue transferred from an area adjacent to defect

distal randon pattern flap


transfer of tissue to a noncontiguous anatomic site
indications
surrounding tissue will not support a local flap
length-width ratio needed < 2:1
distal axial pattern flap
indications
surrounding tissue will not support a local flap
length-width ratio needed > 2:1
free tissue transfer
indicatoins
local or distant tissue not sufficient for distal axial and
random pattern flaps

Technique
Ladder of reconstruction
o in order of increasing complexity
primary closure
secondary closure
healing by secondary intention
skin graft
local flap
regional flap
free tissue transfer

Complications
Flap Failure
o inadequate arterial flow
treatment
immediate return to operating room
o inadequate venous outflow
treatment
loosen dressings, removal of selected sutures
return to operating room if not relieved by above
measure
Donor site morbidity
o may be cosmetically unacceptable
o pain related to grafting
o seroma
treatment
aspiration
excision if encapsulated
Nonunion for vascularized bone transfer
o incidence
may be as high as 32% if no additional bone graft is used

PREFERRED RESPONSE 1
This patient has a large thumb pulp defect measuring 3.5 cm in length, extending proximal to the
interphalangeal joint (IPJ) crease. Inset of a Moberg flap large enough to cover the defect would
necessitate IPJ flexion >45 degrees, increasing the risk of IPJ stiffness.
Thumb pulp defects may be resurfaced by different means, depending on size. The Moberg flap
is suited for medium (1.8-3 cm) defects. For defects >1.5 cm, there is increased risk of wound
dehiscence, parrot beak nail deformity, and decreased soft tissue padding. Modifications such as
V-Y flaps, bilateral Z-plasties, Burrow triangles, 2 lateral triangular flaps at the proximal edge of
the flap, or advancement of an island flap with skin grafting of the secondary defect (OBrien
modification), are recommended.
Baumeister et al. reviewed the functional outcome of Moberg flaps. These flaps do not cause
marked impairment of active ROM and any reduction in the AROM of the IP joint is because of a
loss of hyperextension.
Horta et al. reviewed the use of multiple flaps (Moberg, radial innervated cross-finger,
Venkataswami-Subramanian, Foucher, Tezcan, and Littler). They recommended the Foucher
flap because of good sensibility, single-stage surgery, and no need for cortical reintegration
(unlike the Littler flap)
Figure A shows a large thumb pulp defect. Illustration A shows the options for resurfacing thumb
pulp defects of different sizes. Illustration B is a diagram of these options. Illustrations C and D
depict the Holevich dorsal metacarpal artery flap (with overlying skin strip). Illustrations E and F
depict the Foucher dorsal metacarpal artery flap (islanded).
Incorrect Answers:
Answers 2, 3, 4, 5: These flaps are all possible options for large thumb pulp defects.

PREFERRED RESPONSE 4
The dominant arterial blood supply to a medial gastrocnemius muscle flap is the sural artery.
Rotational gastrocnemius flaps are useful for coverage of the proximal third of the tibia and some
wounds/defects about the knee. Medial and lateral gastrocnemius arterial supply is from the
medial and lateral sural arteries respectively. Coverage of the middle third of the tibia requires
use of a rotational soleus muscle flap, supplied by the peroneal artery proximally and the
posterior tibial artery distally. Coverage of the distal third of the tibia requires a free muscle flap
transfer, based on a specific vascular pedicle.
Illustration A depicts the medial gastrocnemius flap with its sural artery pedicle.

The sural artery supplies the both heads of the gastrocnemius and is the pedicle for rotational
flaps. Eighty-five percent of the time there is a single vascular source.

Skin Grafting
Author: Daniel Hatch
Topic updated on 03/15/16 1:22am

Introduction
A skin graft is an avascular graft and consists of
o partial-thickness dermal tissue
o full-thickness dermal tissue
Donor site
o most commonly autologous
Goals of treatment
o cover deep structures
o create a barrier to bacteria,
o restore dynamic function of the limb
o prevent joint contractures
Indications
o well-perfused wound beds over muscle or subcutaneous tissue
Contraindications
o wounds with exposed bone, tendon, nerves, or blood vessels

Full-Thickness Skin Grafts


Indications
o volar hand wounds and fingertips
Donor sites
o proximal forearm
o hypothenar eminence of hand
Graft elements

contain full thickness of dermis and epidermis, containing hair


follicles and sweat glands
o subcutaneous fat is not included because it decreases vascular
ingrowth and survival
o nutrition is obtained by diffusion from the wound bed
Technique
o apply under gentle tension over a well-perfused wound bed
o place multiple tie-over sutures to decrease shear forces
o dressing should include a medicated gauze and moist cotton
o leave dressing in place for 5 to 7 days
Outcomes
o pros
better reinnveration and sensation
less scar contracture
more durable and wear resistant to shear stresses
o cons
hematomas and seromas can still cause failure
revascularization takes 2 to 3 days
o

Other Skin Grafts


Allograft
o indications
used as a temporary measure to prepare the wound bed for
autograft
Xenograft
o indications
used occasionally as biologic dressings

PREFERRED RESPONSE 2
Soft tissue defects of the palm are most appropriately treated with flap coverage followed with
full-thickness grafts. A flap is a unit of tissue supported by blood vessels and moved from a donor
site to a recipient site to cover a defect in tissue.

This patient's full-thickness coverage was created from a posterior interosseous artery island flap
as shown in Illustrations A-C. The skin of the dorsal hand is similar to that of the rest of the body
and thus may be adequately replaced by split-thickness skin grafts from the skin of most donor
sites. In contrast, palmar hand skin differs from that of the dorsal hand in that it (1) lacks both hair
and sebaceous glands and (2) has specialized encapsulated nerve endings (Meissners
corpuscles and Vater-Pacini corpuscles) that confer enhanced sensation via mechanoreception.
Full thickness skin grafts (FTSG) transfer all of the skin appendages and nerve endings except
those sweat glands located in the subcutaneous tissue and some of the Vater-Pacini corpuscles
of palmar and plantar skin. It is necessary to remove all fat and subcutaneous tissue from the
undersurface of a full-thickness skin graft, as this will otherwise act as a barrier preventing
vascularization and graft survival.

Tendon Transfer Principles


Author: Deborah Allen
Topic updated on 11/11/15 4:42pm

Introduction
Principles of tendon transfers
o

o
o

match muscle strength


force proportional to cross-sectional area
greatest force of contraction exerted when muscle is at
resting length
amplitude proportional to length of muscle
work capacity = (force) x (amplitude)
motor strength will decrease one grade after transfer
should transfer motor grade 5
appropriate tensioning
appropriate excursion
can adjust with pulley or tenodesis effect
Smith 3-5-7 rule
3 cm excursion - wrist flexors, wrist extensors
5 cm excursion - EDC, FPL, EPL

7 cm excursion - FDS, FDP


o surgical priorities
1. elbow flexion (musculocutaneous n.)
2. shoulder stabilization (suprascapular n.)
3. brachiothoracic pinch (pectoral n.)
4. sensation C6-7 (lateral cord)
5. wrist extension and finger flexion (lateral and posterior cords)
o selection
determine what function is missing
determine what muscle-tendon units are available
evaluate the options for transfer
o basic principles
donor must be expendable and of similar excursion and
power
one tendon transfer performs one function
synergistic transfers rehabilitate more easily
it is optimal to have a straight line of pull
one grade of motor strength is lost following transfer
Prognosis
o age
leading prognostic factor
worse after age 30
o location
distal is better than proximal
Presentation
Physical exam
o brachial plexus injury
Horner's sign
correlates with C8-T1 avulsion
often appears 2-3 days following injury
severe pain in anesthetic limb
indication of root avulsion
loss of rhomboid function
indication of root avulsion
o radial nerve palsy
classified according to location of lesion proximal or distal to
the origin of PIN
low radial nerve palsy
PIN syndrome
high radial nerve palsy
loss of radial nerve proper function (triceps,
brachioradialis, ECRL plus muscles innervated
by PIN)

median nerve palsy


classified according to location of lesion proximal or distal to
the origin of AIN
low median nerve palsy
loss of thumb opposition (APB function)
high median nerve palsy
loss of thumb opposition
loss of thumb, index finger, and middle finger
flexion
ulnar nerve palsy
low ulnar nerve palsy
loss of power pinch
abduction of the small finger (Wartenberg sign)
clawing
results from imbalance between intrinsic and
extrinsic muscles
high ulnar nerve palsy
loss of ring and small finger FDP function
primary distinguishing deficit
clawing less pronounced because extrinsic flexors are
not functioning

Studies
Sensory and motor evoked potentials
o better than standard EMG/NCS
Treatment

Nonoperative
o physical therapy, splinting, and antispasticity medications
indications
decreased passive range of motion
spasticity
Operative
o early surgical intervention (3 weeks to 3 months)
indications
total or near-total brachial plexus injury
high energy injury
o late surgical intervention (3 to 6 months)
indications
partial upper-level brachial plexus palsy
low energy injury
postoperative care
protect for 3-4 weeks then begin ROM
continue with protective splint for 3-6 weeks

synergistic transfers are easier to rehabilitate


(synergistic actions occur together in normal function,
e.g., finger flexion and wrist extension)

Complications
Adhesions
o necessitate aggressive therapy and possible secondary tenolysis

PREFERRED RESPONSE 3
Figures A and B show a pre and post-operative radiograph of a both bone forearm fracture. The
posterior interosseus nerve is at risk during surgical approaches to this fracture pattern and care
should be taken.
Ropars et al retrospectively reviewed 15 patients who underwent treatment for radial nerve and
PIN palsy. For PIN palsy, they concluded the most beneficial transfers included transferring the
flexor carpi radialis to the finger extensors (to restore finger extension) and palmaris longus to
the extensor pollicis longus (to restore extension of the thumb). In contrast with a radial nerve
palsy, with a PIN palsy the patient has adequate wrist extension due to intact ECRL (providing
radial wrist extension) supplied by the radial nerve proximal to the PIN.
Ustn et al in their cadaveric studies were able to show that it is possible to perform posterior
interosseous nerve neurotization via the median nerve.
Hirachi et al reviewed the results of 17 traumatic PIN palsies that were treated either with nerve
repair, tendon transfers, or nonoperatively. They noted that associated muscle damage resulted
in poorer results.
The muscles involved in the suggested transfer (FCR, ED, PL, EPL) are shown in illustration AD.

Carpal Tunnel Syndrome


Author: Jason McKean
Topic updated on 03/05/15 9:30am

Introduction
Most common compressive neuropathy
o pathologic (inflamed) synovium most common cause of idiopathic
CTS
Epidemiology
o affects 0.1-10% of general population
o risk factors
female sex
obesity
pregnancy
hypothyroidism
rheumatoid arthritis
advanced age
chronic renal failure
smoking
alcoholism
repetitive motion activities
mucopolysaccharidosis
mucolipidosis
Pathophysiology
o mechanism
precipitated by
exposure to repetitive motions and vibrations
certain athletic activities
cycling
tennis
throwing
o pathoantomy
compression may be due to

repetitive motions in a patient with normal anatomy


space occupying lesions (e.g., gout)
Associated conditions
o diabetes mellitus
o hypothyroidism
o rheumatoid arthritis
o pregnancy
o amyloidosis
Prognosis
o good prognostic indicators include
night symptoms
short incisions
relief of symptoms with steroid injections
not improved when incomplete release of transverse carpal
ligament is discovered
Anatomy

Carpal tunnel defined by


o scaphoid tubercle and trapezium radially
o hook of hamate and pisiform ulnarly
o transverse carpal ligament palmarly (roof)
o proximal carpal row dorsally (floor)
Carpal tunnel consists of
o nine flexor tendons
o one nerve (median nerve)
o FPL is the most radial structure
Branches of median nerve
o palmar cutaneous branch of median nerve
lies between PL and FCR at level of the wrist flexion crease
o recurrent motor branch of median nerve
50% are extraligamentous with recurrent innervation
30% are subligamentous with recurrent innervation
20% are transligamentous with recurrent innervation
cut transverse ligament far ulnar to avoid cutting if
nerve is transligamentous
Carpal tunnel is narrowest at the level of the hook of the hamate
Presentation
Symptoms
o numbness and tingling in radial 3-1/2 digits
o clumsiness
o pain and paresthesias that awaken patient at night
o self administered hand diagram

the most specific test (76%) for carpal tunnel syndrome


Physical exam
o inspection may show thenar atrophy
o carpal tunnel compression test (Durkan's test)
is the most sensitive test to diagnose carpal tunnels
syndrome
performed by pressing thumbs over the carpal tunnel and
holding pressure for 30 seconds.
onset of pain or paresthesia in the median nerve
distribution within 30 seconds is a positive result.
o Phalen test
wrist volar flexion for ~60 sec produces symptoms
less sensitive than Durkin compression test
o Tinel's test
provocative tests performed by tapping the median nerve
over the volar carpal tunnel
o Semmes-Weinstein testing
most sensitive sensory test for detecting early carpal tunnel
syndrome
measures a single nerve fiber innervating a receptor or
group of receptors
o innervation density test
static and moving two-point discrimination
measures multiple overlapping of different sensory units
and complex cortical integration
the test is a good measure for assessing functional nerve
regeneration after nerve repair
Imaging
Radiographs
o not necessary for diagnosis

Studies
Diagnostic criteria
o numbness and tingling in the median nerve distribution
o nocturnal numbness
o weakness and/or atrophy of the thenar musculature
o positive Tinel sign
o positive Phalen test
o loss of two point discrimination
EMG and NCV
o overview
often the only objective evidence of a compressive
neuropathy (valuable in work comp patients with secondary

gain issues)
not needed to establish diagnosis (diagnosis is clinical) but
recommended if surgical management is being considered
o demyelination leads to
NCV
increase latencies (slowing) of NCV
distal sensory latency of > 3.2 ms
motor latencies > 4.3 ms
decreased conduction velocities less specific than
latencies
velocity of < 52 m/sec is abnormal
EMG
test the electrical activity of individual muscle fibers
and motor units
detail insertional and spontaneous activity
potential pathologic findings
increased insertional activity
sharp waves
fibrillations
fasciculations
complex repetitive discharges
Histology
o nerve histology characterized by
edema, fibrosis, and vascular sclerosis are most common
findings
scattered lymphocytes
amyloid deposits shown with special stains in some cases
Treatment
Nonoperative
o NSAIDS, night splints, activity modifications
indications
first line of treatment
modalities
night splints (good for patients with nocturnal

symptoms only)
activity modification (avoid aggravating activity)
steroid injections
indications
adjunctive conservative treatment
outcomes
80% have transient improvement of symptoms (of
these 22% remain symptoms free at one year)

failure to improve after injection is poor prognostic


factor
surgery is less effective in these patients
Operative
o carpal tunnel release
indications
failure of nonoperative treatment (including steroid
injections)
temporary improvement with steroid injections
is a good prognostic factor that the patient will
have a good result with surgery)
acute CTS following ORIF of a distal radius fx
outcomes
pinch strength return in 6 week
grip strength is expected to return to 100%
preoperative levels by 12 weeks postop
o revision CTR for incomplete release
indications
failure to improve following primary surgery
incomplete release most common reason
outcomes
only 25% will have complete relief after revision CTR
50% some relief
25% will have no relief
Technique
Open carpal tunnel release
o

antibiotics
prophylactic antibiotics, systemic or local, are not indicated
for patients undergoing a clean, elective carpal tunnel
release
technique
internal neurolysis, tenosynovectomy, and antebrachial
fascia release do not improve outcomes
Guyon's canal does not need to be released as it is
decompressed by carpal tunnel release
lengthened repair of transverse carpal
ligament only required if flexor tendon repair performed
(allows wrist immobilization in flexion postoperatively)
complications
correlate most closely with experience of surgeon
incomplete release
progressive thenar atrophy due to injury to an unrecognized
transligamentous motor branch of the median nerve

Endoscopic carpal tunnel release

o
o
o

advantage is accelerated rehabilitation


long term results same as open CTR
most common complication is incomplete division of transverse
carpal ligament

PREFERRED RESPONSE 4
The clinical presentation is consistent with carpal tunnel syndrome caused by an atypical space
occupying lesion - in his case, gout. The most appropriate next step in the management of his
symptoms would be referral to a rheumatologist where medical therapy, such as prophylaxis with
colchicine, could be initiated.
Carpal tunnel syndrome is the most common compressive neuropathy, affecting up to 10% of the
general population. Risk factors include female sex, advanced age, obesity, and repetitive motion
activities. Typically, patients will develop symptoms of median nerve compression including
thenar muscle atrophy, numbness in the radial 3.5 digits, night pain, and positive Tinel's and

Phalen tests. First line management is non-operative, including NSAIDs, night splints, and
activitiy modification. Carpal tunnel release surgery is indicated for those who have failed
conservative management.
Chen et al. described 23 unusual cases of CTS in which space-occupying lesions were
responsible for the symptoms and signs of median nerve compression. In patients with an
atypical presentation, such as male gender, non-middle-aged, or unilateral involvement, spaceoccupying lesions such as gout, synovial sarcoma, lipoma, and ganglions should be investigated
as a cause.
Fitzgerald et al. discussed gout affecting the hand and wrist. The medical treatment of gout
includes NSAIDs such as indomethacin or ibuprofen for acute flares, and colchicine and
allopurinol for chronic prophylaxis.
Figures A and B represent axial CT and MRI images showing calcification and gouty tophi
deposition in the carpal tunnel floor.
Incorrect Answers:
Answer 1: Aspiration is not a first line treatment for tophaceous gout.
Answer 2: The clinical picture is not suspicious for a malignancy, therefore a biopsy would not be
indicated.
Answer 3: Night splints would not help diminish the space occupying lesion, in this case,
tophaceous gout.
Answer 5: Chronic tophaceous gout that has failed medical therapy may require surgical
excision.

PREFERRED RESPONSE 4
The patient is undergoing a clean, elective hand surgery. Prophylactic antibiotics, systemic or
local, are not indicated for these procedures.
Carpal tunnel syndrome is the most common compressive neuropathy. Individuals who fail
medical management (night splints, NSAIDs, activity modification) are candidates for carpal
tunnel release surgery (CTS). The surgery may be performed open or endoscopically. The
reported incidence of post-operative infections following CTS varies between studies from 0% to
8%.

Whittaker et al. performed a prospective, randomized, double-blinded, placebo controlled trial


investigating the use of antibiotic prophylaxis in clean, incised hand injuries. They found no
significant difference in infection rates between patients who received IV flucloxacillin, IV followed
by oral flucloxacillin, and an oral placebo (13% vs. 4% vs. 15%, p=0.19). They did not support
the use of routine antibiotic prophylaxis prior to clean hand surgery.
Bykowski et al. retrospectively reviewed 8,850 outpatient elective hand surgeries and found no
significant difference in the rate of surgical site infection, including patients with diabetes or
history of smoking. They concluded that antibiotics should not be routinely administered prior to
clean, elective hand surgeries.
Harness et al. found no statistical difference in the incidence of surgical site infection following
CTS without prophylactic antibiotic compared with patients who received prophylactic antibiotics
(0.7% vs. 0.4%, p=0.354). They did not recommend routine antibiotic prophylaxis.
Illustration A reviews the anatomic components of the carpal tunnel.
Incorrect Answers:
Answers 1, 2, 3, 5: Antibiotics do not decrease the risk of post-operative infection in clean,
elective hand surgery. Surgeons should consider the potential risks of antibiotics prior to
administration, including Clostridium difficile colitis, antibiotic allergies, bacterial resistance, and
so on.

PREFERRED RESPONSE 3
The patients history, examination, and nerve conduction velocity tests (normal distal sensory
latency is <3.5 ms) are consistent with carpal tunnel syndrome. There is Level 1 and 2 evidence
supporting local steroid injection or splinting for the nonoperative treatment of carpal tunnel
syndrome. Phonophoresis, Vitamin B6 (pyridoxine), heat therapy, bumetanide, and physical
therapy are not considered the most appropriate options for carpal tunnel syndrome
management.
The AAOS clinical guidelines for carpal tunnel syndrome consist of 9 clinical recommendations
supported with a grading of the recommendation and levels of evidence for the literature
contributing to the recommendation.
The use of neutral wrist splints for carpal tunnel syndrome is most useful for improving night-time
symptoms. However wrist splinting is most functional at 30 degrees of extension, and the neutral
splints can be functionally limiting when used during productive daytime hours.

PREFERRED RESPONSE 4
The only neurovascular structure that runs in the carpal tunnel is the median nerve. Flexor carpi
radialis is (FCR) is not a tendon within the carpal tunnel. In summary, the carpal tunnel contains
the median nerve, FPL and 4 tendons each of the FDP and FDS. Of note, with respect to the
FDS tendons, the 3rd and 4th FDS tendons are volar to the 2nd and 5th FDS tendons.

PREFERRED RESPONSE 5
EMG's detect the electrical potential generated by muscle cells when these cells are electrically
activated. They give information about the muscle motor unit and can display the presence of
fibrillations, sharp waves, motor recruitment, and insertional activity of the muscle. The nerve
conduction (NCV) portion of the electrodiagnostic study measures the speed at which the nerve
impulse travels down the axon. Large, myelinated nerve fibers conduct impulses the fastest and
thus only these fibers are evaluated in the nerve conduction portion of the electrodiagnostic
study. Distal latencies and conduction velocities are measured with NCV's. General parameters
for NCV diagnosis of carpal tunnel syndrome include a distal motor latency of >4.5 msec, a distal
sensory latency of >3.5msec, or a conduction velocity of < 52 m/sec.
The articles by Brumback et al and Gooch et al is a review of electrodiagnostic studies for
compression neuropathies.

PREFERRED RESPONSE 2
This question is based on the fact that carpal tunnel canal pressure varies with wrist position.
Use of neutral wrist splints for carpal tunnel syndrome is most useful for improving noctural
symptoms. The reason for this is the functional position of the wrist is approximately 30 degrees
of extension, and the neutral splints can be functionally limiting when used during productive
daytime hours.
The reference by Gerritsen et al is a randomized controlled study of splinting versus surgery for
carpal tunnel. They found a 80% success rate for surgery at final follow-up versus 54% for
splinting at 3 months, which increased to 90% at 18 months for surgery and 75% for splinting.
The reference by Omer is a review of carpal tunnel, and it covers the diagnosis, treatment, and
follow-up care of these patients. They note the need for careful diagnosis to avoid unnecessary
or inappropriate surgery.
Weiss et al showed that carpal tunnel pressures are elevated when the wrist is in extension, and
are lowest at near neutral. If one couples this with the inherent tunnel pressure increase from the
disease itself, its easy to see that extension splinting is a double hit and can lead to increased
symptoms.

PREFERRED RESPONSE 5
All of the listed physical exam findings, except for loss of small digit adduction (Wartenberg sign),
has been found to be predictive for diagnosing carpal tunnel syndrome.

Szabo et al in a Level 3 study used a regression model to analyze the most predictive factors for
correctly diagnosing carpal tunnel syndrome (CTS). Their analysis found that with an abnormal
hand diagram, abnormal sensibility by Semmes-Weinstein testing in wrist-neutral position, a
positive Durkan's test, and night pain, the probability that carpal tunnel syndrome will be correctly
diagnosed is 0.86. They found the tests with the highest sensitivity were Durkan's compression
test (89%), Semmes-Weinstein testing after Phalen's maneuver (83%), and hand diagram scores
(76%). Night pain was a sensitive symptom predictor (96%). The most specific tests were the
hand diagram (76%) and Tinel's sign (71%). The authors concluded that the addition of
electrodiagnostic tests did not increase the diagnostic power of the combination of these 4
clinical tests, and proceeding with surgical release is appropriate even if the EMG is normal.
Wartenberg sign is persistent abduction and extension of the small digit when a patient is asked
to adduct the digits and is seen in cubital tunnel syndrome, but not carpal tunnel syndrome.
Illustration V demonstrate the Durkan's Compression test for carpal tunnel syndrome.

PREFERRED RESPONSE 4

Gellman et al quantified grip and pinch strength post-operatively after carpal tunnel release. They
found grip strength was 28% of preoperative level at 3 weeks; 73% by 6 weeks, returned to the
preoperative level by 3 months, and 116% at 6 months. Pinch strength was 74% of preoperative
level at 3 weeks, 96% at 6 weeks, 108% at 3 months, and 126% at 6 months.

AIN Compressive Neuropathy


Authors: Ujash Sheth, Mark Vitale
Topic updated on 12/01/15 5:50pm

Introduction
A compressive neuropathy of the AIN that results in
o motor deficits only
o no cutaneous sensory changes
Pathoanatomy
o potential sites of entrapment
tendinous edge of deep head of pronator teres
most common cause
FDS arcade
edge of lacertus fibrosus
accessory head of FPL (Gantzer's muscle)
accessory muscle from FDS to FDP
abberant muscles (FCRB, palmaris profundus)
thrombosed ulnar radial or ulnar artery
o patient with complete AIN palsy should have no motor function to
all muscles innervated by AIN
patients with incompletes palsies or with Martin-Gruber
anastamoses (anomalous anatomy in 15% of population
where axons of AIN may cross over and connect to ulnar
nerve and innervate other muscle groups)
present with intrinsic weakness
Associated conditions
o Parsonage-Turner Syndrome
bilateral AIN signs caused by viral brachial neuritis
be suspicious if motor loss is preceded by intense shoulder
pain and viral prodrome
Anatomy
AIN is terminal motor branch of median nerve
o AIN arises from the median nerve approximately 4-6 cm distal to
the medial epicondyle
o Travels between FDS and FDP initially, then between FPL and
FDP, then it lies on the anterior surface of the interosseous
membrane traveling with the anterior interoseous artery to pronator
quadratus
o Terminal branches innervate the joint capsule and the intercarpal,

radiocarpal and distal radioulnar joints.


AIN has principally motor innervation (no cutaneous sensory) and
innervates 3 muscles
o FDP (index and middle finger)
o FPL
o pronator quadratus
Presentation
Symptoms
o motor deficits only
o no complaints of pain, unlike other median compression
neuropathies (carpal tunnel syndrome and pronator syndrome)
Physical exam
o weakness of grip and pinch, specifically thumb, index and middle
finger flexion
o patient unable to make OK sign (test FDP and FPL)
o pronator quadratus weakness shown with weak resisted pronation
with elbow maximally flexed
o distinguish from FPL attritional rupture (seen in rheumatoids) by
passively flexing and extending wrist to confirm tenodesis effect in
intact tendon
if tendons intact, passive wrist extension brings thumb IP
joint and index finger DIP joint into relatively flexed position
Evaluation
NCV / EMG
o helpful to make diagnosis
o may reveal abnormalities in the FPL, FDP index and middle finger
and pronator quadratus muscles
o assess severity of neuropathy
o may rule out more proximal lesions
Treatment
Nonoperative
o observation, rest and splinting in 90 flexion
indications
in vast majority of patients, unless clear space
occupying mass
majority will improve with nonoperative management
technique
elbow splinting in 90 degrees of flexion (8-12 weeks)
Operative
o surgical decompression of AIN
indications

if nonoperative treatment fails after several months


approximately 75% success rate of surgical
decompression
Techniques

Surgical decompression of AIN


o technique
release of superficial arch of FDS and lacertus fibrosus
detachment of superficial head of pronator teres
ligation of any crossing vessels
removal of any space occupying lesion
Complications
Recurrence

PREFERRED RESPONSE 1
This patient presents with anterior interosseous nerve (AIN) syndrome and is often seen in
conjunction with brachial neuritis (Parsonage-Turner Syndrome).
AIN syndrome leads to motor palsies of the flexor pollicis longus and the two radial profundus
tendons leading to the clincal image shown in Figure A. The pronator quadratus is also involved
and can be tested with the elbow held in a flexed position to neutralize the humeral head of the
pronator teres muscle. No sensory changes occur and electromyographic (EMG) and

nerveconduction (NCV)studies are often helpful in establishing the diagnosis.


The Level 5 review article by Rodner et al. discusses that anterior interosseous nerve syndrome
usually resolves with time, particularly if the lesion is secondary to neuritis. Observation for 3 to 6
months with splinting at 90 degrees is favored before surgical treatment.
Parsonage and Turner reported in their 1948 landmark article, on several cases of isolated AIN
palsy caused by neuralgic amyotrophy (ie, Parsonage-Turner syndrome [PTS] or brachial plexus
neuritis). PTS symptoms may include pain and motor and/or sensory dysfunction in one or in
multiple peripheral nerves of the upper extremity.

Pronator Syndrome
Author: David Abbasi
Topic updated on 11/03/14 8:27pm

Introduction
A compressive neuropathy of the median nerve at the level of the elbow
Epidemiology
o more common in women
o common in 5th decade
o has been associated with well-developed forearm muscles (e.g.
weight lifters)
Pathoanatomy
o 5 potential sites of entrapment include
supracondylar process
residual osseous structure on distal humerus present
in 1% of population
ligament of Struthers
travels from tip of supracondylar process to medial
epicondyle
not to be confused with arcade of Struthers which is a
site of ulnar compression neuropathy in cubital tunnel
syndrome
bicipital aponeurosis (a.k.a. lacertus fibrosus)
between ulnar and humeral heads of pronator teres
FDS aponeurotic arch
Associated conditions
o commonly associated with medial epicondylitis
Presentation
Symptoms
o paresthesias in thumb, index, middle finger and radial half of ring
finger as seen in carpal tunnel syndrome
in pronator syndrome paresthesias often made worse with
repetitive pronosupination
o should have characteristics differentiating from carpal tunnel

syndrome (CTS)
aching pain over proximal volar forearm
sensory disturbances over the distribution of palmar
cutaneous branch of the median nerve (palm of hand)
which arises 4 to 5 cm proximal to carpal tunnel
lack of night symptoms
Physical exam
o provocative tests are specific for different sites of entrapment
positive Tinel sign in the proximal anterior forearm but no
Tinel sign at wrist nor provocative symptoms with wrist
flexion as would be seen in CTS
resisted elbow flexion with forearm supination (compression
at bicipital aponeurosis)
resisted forearm pronation with elbow
extended (compression at two heads of pronator teres)
resisted contraction of FDS to middle finger (compression
at FDS fibrous arch)
o possible coexisting medial epicondylitis
Imaging
Radiographs
o recommended views
elbow films are mandatory
o findings
may see supracondylar process
Studies
EMG and NCV
o may be helpful if positive but are usually inconclusive
o may exclude other sites of nerve compression or identify doublecrush syndrome
Treatment
Nonoperative
o rest, splinting, and NSAIDS for 3-6 months
indications
mild to moderate symptoms
technique
splint should avoid forearm rotation
Operative
o surgical decompression of median nerve
indications
only when nonoperative management fails for 3-6
months

technique
decompression of the median nerve at all 5 possible
sites of compression
outcomes
of surgical decompression are variable
80% of patients having relief of symptoms

Cubital Tunnel Syndrome


Author: Deborah Allen
Topic updated on 04/13/15 6:56am

Introduction
A compressive neuropathy of the ulnar nerve
o 2nd most common compression neuropathy of the upper extremity
Sites of entrapment

most common
between the two heads of FCU/aponeurosis (most common site)
within arcade of Struthers (hiatus in medial intermuscular septum)
between Osborne's ligament and MCL
o less common sites of compression include
medial head of triceps
medial intermuscular septum
medial epicondyle
fascial bands within FCU
anconeus epitrochlearis (anomalous muscle from the medial olecrano
the medial epicondyle)
aponeurosis of FDS proximal edge
o external sources of compression
fractures and medial epicondyle nonunions
osteophytes
heterotopic ossification
tumors and ganglion cysts
Associated conditions
o cubitus varus or valgus deformities
o medial epicondylitis
o burns
o elbow contracture release
o

Anatomy
Ulnar nerve
o pierces intramuscular septum at arcade of Struthers 8 cm proximal to the
medial epicondyle as it passes from the anterior to posterior compartment of
the arm

enters cubital tunnel


Cubital tunnel
o roof
formed by FCU fascia and Osborne's ligament (travels from the media
epicondyle to the olecranon)
o floor
formed by posterior and transverse bands of MCL and elbow joint
capsule
o walls
formed by medial epicondyle and olecranon
o

Presentation
Symptoms
o paresthesias of small finger, ulnar half of ring finger, and ulnar dorsal
hand

exacerbating activities include


cell phone use (excessive flexion)
occupational or athletic activities requiring repetitive elbow flexio
and valgus stress
o night symptoms
caused by sleeping with arm in flexion
Physical exam
o inspection and palpation
interosseous and first web space atrophy
ring and small finger clawing
observe ulnar nerve subluxation over the medial epicondyle as the
o
o

elbow moves through a flexion-extension arc


sensory
decreased sensation in ulnar 1-1/2 digits
motor
loss of the ulnar nerve results in paralysis of intrinsic muscles (adducto
pollicis, deep head FPB, interossei, and lumbricals 4 and 5) which lead
to
weakened grasp
from loss of MP joint flexion power
weak pinch
from loss of thumb adduction (as much as 70% of pinch
strength is lost)
Froment sign
compensatory thumb IP flexion by FPL (AIN) during key
pinch
compensates for the loss of MCP flexion by adducto
pollicis (ulna n.)

adductor pollicis muscle normally acts as a


MCP flexor, first metacarpal adductor, and IP
extensor

Jeanne sign
compensatory thumb MCP hyperextension and thumb
adduction by EPL (radial n.) with key pinch
compensates for loss of IP extension and thumb
adduction by adductor pollicis (ulna n.)
Wartenberg sign
persistent small finger abduction and extension during
attempted adduction secondary to weak 3rd palmar
interosseous and small finger lumbrical
Masse sign
palmar arch flattening and loss of ulnar hand elevation
secondary to weak opponens digiti quinti and decreased
small finger MCP flexion
extrinsic weakness
Pollock's test
shows weakness of two ulnar FDPs
provocative tests
Tinel sign positive over cubital tunnel
elbow flexion test
positive when flexion of the elbow for > 60 seconds reproduces
symptoms
direct cubital tunnel compression exacerbates symptoms

Studies
EMG / NCV
o helpful in establishing diagnosis and prognosis
o threshold for diagnosis
conduction velocity <50 m/sec across elbow
low amplitudes of sensory nerve action potentials and compound
muscle action potentials

Treatment
Nonoperative
o NSAIDs, activity modification, and nighttime elbow extension splinting
indications
first line of treatment with mild symptoms
technique
night bracing in 45 extension with forearm in neutral rotation
outcomes
management is effective in ~50% of cases
Operative

in situ ulnar nerve decompression without transposition


approach
elbow medial approach
indications
when nonoperative management fails
before motor denervation occurs
technique
open release of cubital tunnel retinaculum
endoscopically-assisted cubital tunnel release
favorable early results but lacks long-term data
outcomes
meta-analyses have shown similar clinical results with
significantly fewer complications compared to decompression w
transposition
80-90% good results when symptoms are intermittent and
denervation has not yet occurred
poor prognosis correlates most with intrinsic muscle atrophy
ulnar nerve decompression and anterior transposition
indications
failed in situ release
throwing athlete
patient with poor ulnar nerve bed from tumor, osteophyte, or
heterotopic bone
technique
subcutaneous, submuscular, or intramuscular transposition
outcomes
similar outcomes to in situ release but increased risk of creating
new point of compression
medial epicondylectomy
indications
visible and symptomatic subluxating ulnar nerve
technique
in situ release with medial epicondylectomy
outcomes
risk of destabilizing the medial elbow by damaging the medial
ulnar collateral ligament

Complications
Recurrence
o secondary to inadequate decompression, perineural scarring, or tethering at
the intermuscular septum or FCU fascia
o higher rate of recurrence than after carpal tunnel release
Neuroma formation

iatrogenic injury to a branch of the medial antebrachial cutaneous nerve may


cause persistent posteromedial elbow pain

PREFERRED RESPONSE 4
The video demonstrates Froment's sign on the patient's right hand, which is characterized by
interphalangeal (IP) flexion during attempted key pinch, and is found in patients with ulnar
neuropathy. Therefore it can be found with ulnar nerve compression in the cubital tunnel (Cubital
Tunnel Syndrome) or in Guyon's Canal (Ulnar Tunnel Syndrome).
Froment's sign is performed by having the patient pinch a piece of paper with the thumb IP joint
extended against resistance (pulling paper away). It should be done with both hands side by side
to compare them to each other.
In a hand with a ulnar neuropathy, adductor pollicis (ulnar n.) is deficient, and can not flex the
MCP joint to give pinch strength with an extended IP joint. The thumb compensates by recruiting
the FPL (median n.) to flex the IP joint to give pinch strength. The result is, in a positive
Froment's sign, the IP joint will flex (buckle) to try to give increased strength to the pinch.

PREFERRED RESPONSE 1

The ulnar nerve passes posterior to the medial epicondyle and medial to the olecranon, then
enters the cubital tunnel. The roof of the cubital tunnel is primarily made up of Osborne's
ligament, and the floor consists of the medial collateral ligament.
These soft tissue structures can cause narrowing of the tunnel, especially with elbow flexion,
leading to ulnar nerve compression and cubital tunnel syndrome. This is shown in Illustration A.
The Arcade of Struthers is a band of deep fascia that attaches to the intermuscular septum and
covers the ulnar nerve 8 cm proximal to the medial epicondyle. The intramuscular septum is
continuous from the medial epicondyle to the coracobrachialis muscle. The ulnar nerve travels
through the two heads of the FCU distal to the cubital tunnel. These anatomic landmarks are
shown in Illustration B.
Morrey evaluated 26 patients with post-traumatic contracture of the elbow who were treated with
either operative release alone, or operative release and distraction arthroplasty. Twenty-four
(96%) of the patients had improved elbow function and two had persistent ulnar neuritis treated
with nerve transposition.
Cheung et al discuss the various surgical approaches to the elbow and the indications for each.

PREFERRED RESPONSE 2
There are five sites of potential ulnar nerve entrapment around the elbow: arcade of Struthers,
medial intermuscular septum, medial epicondyle, cubital tunnel, and deep flexor pronator
aponeurosis.
The ulnar nerve emerges from the medial intermuscular septum, under the arcade of Struthers,
and lies on the medial head of the triceps. At the level of the elbow, the ulnar nerve continues
distally toward the posterior aspect of the condylar groove, passing between the medial

epicondyle and olecranon to enter the cubital tunnel. The roof is formed by the arcuate
(Osbornes) ligament. This ligament blends distally with the antebrachial fascia superficial to the
aponeurosis and connects the ulnar and humeral heads of the FCU. The ligament of Struthers is
a fibrous band extending from the supracondylar process of the humerus to the medial
epicondyle which can cause compression of the median nerve.
Elhassan et al discuss the pathogenesis, evaluation, and treatment of entrapment neuropathy of
the ulnar nerve.
Illustration A shows the various site of compression at the elbow

PREFERRED RESPONSE 1
The patient's clinical presentation and physical exam are consistent with cubital tunnel
syndrome. The clinical photograph demonstrates Froment's sign; compensatory IP hyperflexion
of FPL (AIN) to compensate for the loss of adductor pollicis (ulnar nerve) during key pinch.

Simple decompression of the ulnar nerve is less invasive and achieves clinical outcomes
equivalent to decompression with transposition.
Zlowodzki et al conducted a meta-analysis evaluating anterior transposition and simple
decompression of the ulnar nerve. No difference in motor nerve-conduction velocities or clinical
outcome scores was found.
Bartels performed a prospective randomized trial (included in the Zlowodski meta-analysis) on
152 patients comparing simple decompression to transposition. No difference in clinical results at
1 year were reported, but a significantly higher complication rate occurred in the transposition
group (31%) compared to simple decompression (9.6%).
Nabhan et al performed a level 1 study randomizing 66 patients to simple decompression or
subcutaneous ulnar nerve transposition. No differences were found with respect to clinical
outcome or nerve conduction velocities.

Ulnar Tunnel Syndrome


Author: Daniel Hatch
Topic updated on 12/17/15 11:14am

Introduction
Ulnar nerve compression neuropathy caused by direct compression
in Guyon's canal
o also known as handlebar palsy (seen in cyclists)
Pathoanatomy
o causes of compression include
ganglion cyst (80% of nontraumatic causes)
lipoma
repetitive trauma
ulnar artery thrombosis or aneurysm
hook of hamate fracture or nonunion
pisiform dislocation
inflammatory arthritis
fibrous band, muscle or bony anomaly
congenital bands
palmaris brevis hypertrophy
idiopathic
Anatomy
Guyons canal
o course
is approximately 4 cm long
begins at the proximal extent of the transverse carpal
ligament and ends at the aponeurotic arch of the
hypothenar muscles
o contents
ulnar nerve bifurcates into the superficial sensory and deep

motor branches
boundaries and zones (see table below)

Deep branch of the ulnar nerve


o innervates all of the interosseous muscles and the 3rd and 4th
lumbricals.
o Innervates the hypothenar muscles, the adductor pollicis, and
the medial head (deep) of the flexor pollicis brevis (FPB)
Classification

Presentation varies based on location of compression within Guyon's


canal and may be
o
o
o

Motor only
Sensory only
Mixed Motor & Sensory

Presentation
Presentation varies based on location of compression within Guyon's
canal and may be
o pure motor
o pure sensory
o mixed motor and sensory
Symptoms
o pain and paresthesias in ulnar 1-1/2 digits
o weakness to intrinsics, ring and small finger digital flexion or
thumb adduction
Physical exam
o inspection & palpation
clawing of ring and little fingers
caused from loss of intrinsics flexing the MCPs and
extending the IP joints

Allen test
helps diagnose ulnar artery thrombosis
neurovascular exam
ulnar nerve palsy results in paralysis of the intrinsic
muscles (adductor pollicis, deep head FPB, interossei,
and lumbricals 4 and 5)
weakened grasp
from loss of MP joint flexion power
weak pinch
from loss of thumb adduction (as much as 70% of
pinch strength is lost)
Froment sign
IP flexion compensating for loss of thumb adduction
when attempting to hold a piece of paper
loss of MCP flexion and adduction by adductor
pollicis (ulnar n.)
compensatory IP hyperflexion by FPL (AIN)
Jeane's sign
a compensatory thumb MCP
hyperextension and thumb adduction by EPL (radial
n.)
compensates for loss of IP extension and thumb
adduction by adductor pollicis (ulna n.)
Wartenberg sign
abduction posturing of the little finger

Imaging
Radiographs
o useful to evaluate hook of hamate fractures
CT scan
o useful to evaluate hook of hamate fractures
MRI
o useful to evaluate for a ganglion cysts
o a gradient echo MRI will also show an ulnar artery
aneurysm
Doppler US or arteriogram
o useful to diagnosis ulnar artery thrombosis and aneurysm
Studies

NCS and EMG


o helpful in establishing diagnosis and prognosis
o threshold for diagnosis
conduction velocity <50 m/sec across elbow

low amplitudes of sensory nerve action potentials and


compound muscle action potentials
Differential

How to differentiate ulnar tunnel syndrome from cubital tunnel


syndrome
o cubital tunnel demonstrates
less clawing
sensory deficit to dorsum of the hand
motor deficit to ulnar-innervated extrinsic muscles
Tinel sign at the elbow
positive elbow flexion test
Treatment
Nonoperative
o activity modification, NSAIDS and splinting
indications
as a first line of treatment when symptoms are mild
Operative
o local decompression
indications
severe symptoms that have failed nonoperative
treatment
o tendon transfers
indications
correction of clawed fingers
loss of power pinch
Wartenberg sign (abduction of small finger)
o carpal tunnel release
indications
patients diagnosed with both ulnar tunnel syndrome
and CTS
Techniques

Local surgical decompression


o release hypothenar muscle origin
o decompress ganglion cysts
o resect hook of hamate
o vascular treatment of ulnar artery thombosis
o explore and release all three zones in Guyon's canal
Tendon transfers
o correct claw fingers
possible grafts include ECRL, ECRB, palmaris longus
tendons must pass volar to transverse metacarpal

o
o

ligament in order to flex the proximal phalanx


attach with either a two or four-tailed graft to the A2 pulley
of the ring and small fingers
restore power pinch
Smith transfer using ECRB or FDS of ring finger
restore adduction of small finger
transfer ulnar insertion of EDM to A1 pulley or radial
collateral ligament of the small finger

Complications
Recurrance

PREFERRED RESPONSE 4
Compression of the ulnar nerve within Guyon's canal, termed ulnar tunnel syndrome, is most
commonly caused by a ganglion cyst. A lack of dorsal ulnar sensory deficit helps differentiate
entrapment here from at the elbow because the dorsal ulnar cutaneous nerve branches proximal
to Guyon's canal. The clinical photo demonstrates Froment's sign where the FPL is used to
substitute for the weakened adductor pollicis resulting in flexion of the thumb at the
interphalangeal joint, and MCP joint hyperextension. The AIN can be compressed by the
accessory head of the FPL (Gantzer's muscle) which results in loss of FPL, index FDP and PQ

motor function and no sensory deficits. Ulnar nerve compression at Osborne's ligament, the two
heads of the FCU, or by the anconeus epitrochlearis will classically result in volar and dorsal
ulnar sensory loss of the affected hand.

PIN Compression Syndrome


Author: Evan Watts
Topic updated on 12/12/14 4:15pm

Introduction
A compressive neuropathy of the PIN which affects the nerve supply of
the forearm extensor compartment
Epidemiology
o incidence
reported as 3 per 100,000 people yearly
o demographics
more common in manual laborers, males and bodybuilders
Pathophysiology
o mechanism of injury
microtrauma
from repetitive pronosupination movements
trauma
fracture/dislocation (e.g., monteggia fx, radial head fx,
etc)
space filling lesions
e.g. ganglion, lipomas, etc
inflammation
e.g. rheumatoid synovitis of radiocapitellar joint
iatrogenic (surgery)
o pathoanatomy:
five potential sites of compression include
fibrous tissue anterior to the radiocapitellar joint
between the brachialis and brachioradialis
leash of Henry
are recurrent radial vessels that fan out across
the PIN at the level of the radial neck
extensor carpi radialis brevis edge
medio-proximal edge of the extensor
carpi radialis brevis
"arcade of Frhse"
which is the proximal edge of the superficial
portion of the supinator
supinator muscle edge
distal edge of the supinator muscle

Anatomy
PIN
o

origin
PIN is a branch of the radial nerve that provides motor innervation
to the extensor compartment
course
passes between the two heads of origin of the supinator muscle
direct contact with the radial neck osteology
passes over abductor pollicis longus muscle origin to reach
interosseous membrane
transverses along the posterior interosseous membrane
innervation
motor
common extensors
ECRB (often from radial nerve proper, but can be
from PIN)
Extensor digitorum communis (EDC)
Extensor digiti minimi (EDM)
Extensor carpi ulnaris (ECU)
deep extensors
Supinator
Abductor pollicis longus (APL)
Extensor pollicus brevis (EPB)
Extensor pollicus longus (EPL)
Extensor indicis proprius (EIP)
sensory
sensory fibers to dorsal wrist capsule
provided by terminal branch which is located on the
floor of the 4th extensor compartment
no cutaneous innervation

Presentation
Symptoms
o insidious onset, often goes undiagnosed
o defining symptoms
pain in the forearm and wrist
location depends on site of PIN compression
e.g., pain just distal to the lateral epicondyle of
the elbow may be caused by compression at
the arcade of Frohse
weakness with finger, wrist and thumb movements
Physical exam
o inspection
chronic compression may cause forearm extensor
compartment muscle atrophy
o motion
weakness

finger metacarpal extension weakness


wrist extension weakness
inability to extend wrist in neutral or ulnar
deviation
the wrist will extend with radial deviation due to
intact ECRL (radial n.) and absent ECU (PIN).
provocative tests
resisted supination
will increase pain symptoms
normal tenodesis test
tenodesis test is used to differentiate from extensor
tendon rupture from RA

Evaluation
Radiographs
o indications
not commonly needed for the diagnosis of PIN compression
syndrome
MRI
o indications
not commonly needed for the diagnosis of PIN compression
syndrome
may be help to site and delineate the soft tissue mass
responsible for compression
helpful for surgical planning of mass resection
Studies

EMG
o

indications
may help identify the level of nerve compression
may be used to rule out differential diagnoses of
neuropathy

Differential

Cervical spine nerve compression


Brachial plexus compression
Peripheral neuropathy
Treatment
Nonoperative
o rest, activity modification, stretching, splinting, NSAIDS
indications
recommended as first-line treatment for all cases
o lidocaine/corticosteroid injection

indications
a compressive mass, such as lipoma or ganglion, has
been ruled out
isolated tenderness distal to lateral epicondyle
trial of rest, activity modification, anti-inflammatories
were not effective
technique
single injection 3-4 cm distal to lateral epicondyle at
site of compression
surgical decompression
indications
symptoms persist for greater than three months of
nonoperative treatment
compressive mass detected on imaging
outcomes
results are variable
spontaneous recovery of motor function was seen in
75 - 97% of non-traumatic case series
may continue to improve for up to 18 months

Technique

Surgical decompression
o approach

anterolateral approach to elbow


is most common
approach
may also consider posterior approach
decompression
decompression should begin with release of
fibrous bands connecting brachialis and
brachioradialis
leash of Henry
fibrous edge of ECRB
radial tunnel, including arcade of Frosche and distal
supinator

Complications
Neglected PIN compression syndrome
o muscle fibrosis of PIN innervated muscles
o resulting in tendon transfer procedures to re-establish function
Chronic pain

PREFERRED RESPONSE 3
Based on the choices above, fibrillations will be seen in the extensor pollicis longus, supinator
and abductor pollicis longus muscles.
The radial nerve splits into the superficial radial branch and the posterior interosseous nerve
(PIN) at the anterior aspect of the radiocapitellar joint, just proximal to the supinator muscle. The
PIN innervates the EDC, EDM, ECU, EPB, EPL, EIP, APL and sometimes the ECRB.
Compressive neuropathy of the PIN leads to motor dysfunction, namely weakness with wrist and
finger extension.
Lubhan et al. review uncommon compression neuropathies affecting the upper extremity. They
indicate that PIN syndrome may be caused by rheumatoid arthritis and compressive ganglion
cysts. Depending on which nerve branch is affected, partial lesions may develop. They
recommend use of conservative measures (rest, activity modification and splinting) first.
Decompressive procedures may be indicated in symptoms lasting greater than 3 months.
Illustration A shows the course of posterior interosseous nerve from proximal to distal along the
course of the supinator. This proximal edge of the supinator (Arcade of Froshe), the fibrous edge
of the ECRB and the leash of Henry are three main points of compression of the PIN.
Incorrect Answers
Answer 1, 2, 4: The radial nerve proper innervates the ECRL, ECRB and Brachoradialis
Answer 5: The recurrent motor branch of the median nerve innervates the APB

Radial Tunnel Syndrome


Author: Colin Woon
Topic updated on 03/18/16 6:34pm

Introduction
A compressive neuropathy of the posterior interosseous nerve (PIN)
with pain only
o no motor or sensory dysfunction
Pathophysiology
o involves same sites of compression as PIN syndrome, which
include (from proximal to distal)
fibrous bands anterior to radiocapitellar joint
radial recurrent vessels (leash of Henry)
medial edge of ECRB
proximal aponeurotic edge of the supinator (arcade of
Frohse)
most frequent site of entrapment of the PIN
distal edge of the superficial layer of the supinator
Associated conditions
o

lateral epicondylitis
RTS is difficult to distinguish from lateral epicondylitis
and coexists in 5% of patients

Anatomy
Radial Tunnel
o 5cm in length
o from the level of the radiocapitellar joint, extending distally past
the proximal edge of the supinator
o boundaries
lateral
brachioradialis
ECRL
ECRB
medial
biceps tendon
brachialis
floor
capsule of the radiocapitellar joint
PIN
o origin
PIN is a branch of the radial nerve that provides motor
innervation to the extensor compartment
o course
passes between the two heads of origin of the supinator
muscle
direct contact with the radial neck osteology
passes over abductor pollicis longus muscle origin to reach

interosseous membrane
transverses along the posterior interosseous membrane
innervation
motor
common extensors
ECRB (often from radial nerve proper, but
can be from PIN)
Extensor digitorum communis (EDC)
Extensor digiti minimi (EDM)
Extensor carpi ulnaris (ECU)
deep extensors
Supinator
Abductor pollicis longus (APL)
Extensor pollicus brevis (EPB)
Extensor pollicus longus (EPL)
Extensor indicis proprius (EIP)
sensory
sensory fibers to dorsal wrist capsule
provided by terminal branch which is located on
the floor of the 4th extensor compartment
no cutaneous innervation

Presentation
Symptoms
o deep aching pain in dorsoradial proximal forearm
from lateral elbow to wrist
increases during forearm rotation and lifting activities
o muscle weakness
because of pain and not muscle denervation
Physical exam
o tenderness
over mobile wad over the supinator arch
maximal tenderness is 3-5cm distal to lateral epicondyle
more distal than lateral epicondylitis
o provocative tests
resisted long finger extension test
reproduces pain at radial tunnel
resisted supination test (with elbow and wrist in extension)
reproduces pain at radial tunnel
passive pronation with wrist flexion
reproduces pain at radial tunnel
passive stretch of supinator muscle increases
pressure inside radial tunnel
radial tunnel injection test

diagnostic if injection leads to a PIN


palsy and relieves pain
o
o

sensory
may have paresthesias in the first dorsal web space
motor
no motor manifestations

Imaging
MRI
o
o

usually negative
indications
to identify muscle changes in muscles innervated by PIN
denervation edema/atrophy within the
supinator/extensor
to evaluate compression sites
may show thickened edge of ECRB, prominent radial
recurrent vessels (leash of Henry), swelling of
PIN
to identify other causes of entrapment (rare)
tumors, ganglia, radiocapitellar synovitis, bicipital
bursitis, radial head fractures and dislocations

Studies
Electrodiagnostic studies
o EMG/NCV are inconclusive because
PIN carries unmyelinated Group IV fibers (C-fibers,
nociception) and small myelinated Group IIA afferent fibers
(temperature)
pressure on these fibers produces pain
these fibers cannot be evaluated by EMG/NCV
the large myelinated fibers of PIN remain normal, producing
normal EMG/NCV
Diagnostic injection
o injection of local anesthetic (LA) into the area of localized
tenderness
o ensure that LA does not spread to lateral epicondyle
Differential Diagnosis
Lateral epicondylitis
o both conditions coexist in 5% of patients
o in lateral epicondylitis, tenderness is directly over the lateral
epicondyle
o in RTS, tenderness is 3-5cm distal to the lateral epicondyle
Cervical radiculopathy at C6-7

electrodiagnostic studies may show denervation

Treatment
Nonoperative
o activity modification, temporary splinting, NSAIDS
indications
first line of treatment for at least one year
technique
activity modification
avoid prolonged elbow extension with forearm
pronation and wrist flexion)
o corticosteroid injection
indications
both diagnostic and therapeutic
outcomes
70% improvement at 6 weeks
60% pain free at 2 years
Operative
o radial tunnel release
indications
extensive nonoperative treatment fails
outcomes
surgical release has disappointing results
only 50-90% good to excellent results
delayed maximal recovery of up to 9-18 months
lower success rate in the following groups
concomitant multiple entrapment neuropathies
(60%)
concomitant lateral epicondylitis (40%)
workers compensation patients (30%)
Techniques
Radial tunnel release
o approach
dorsal approaches to the PIN
3 planes have been described
between ECRB and EDC
between brachioradialis and ECRL
transmuscular brachioradialis-splitting
anterior approach to the PIN
between brachioradialis and biceps
o technique
release arcade of Frohse
release distal edge of supinator

release fibrous bands superficial to the radiocapitellar joint


outcomes
success rate of surgical decompression is 70-90%

PREFERRED RESPONSE 2
The patient has radial tunnel syndrome, which often presents with insidious onset of pain and
tenderness several centimeters distal to the lateral epicondyle, and pain elicited with active
extension of the long finger against resistance can help differentiate the condition from lateral
epicondylitis.
Radial tunnel syndrome is a compressive neuropathy that can occur between the mobile wad
laterally and the biceps aponeurosis and brachialis insertion medially as the nerve courses over
the radiocapitellar joint into the forearm. Patients usually have diffuse pain over the site of the
radial tunnel, sometimes have radiating pain in the distribution of the superficial radial nerve, and
occasionally have subtle weakness or fatigue of the wrist and extrinsic finger extensors. Initial
treatment should include conservative measures such as stretching, activity modification, and
NSAIDS; Injections can be performed for both diagnostic and therapeutic reasons. Surgical
intervention is indicated if pain persists despite exhausting conservative measures. The most
common anatomic causes of radial tunnel syndrome are fibrous adhesions between the
brachialis and brachioradialis, the Leash of Henry (radial recurrent vessels), the fibrous edge of
the ECRB, the arcade of Frhse (supinator arch), and fibrous bands of the leading edge of the
supinator muscle.
Dang et al. discuss compression neuropathies of the upper extremity in their 2009 review article.
They highlight the importance of the clinical exam in diagnosing radial tunnel syndrome,
especially the location of pain, which is distal to that of lateral epicondylitis. Additionally ruling out
other less common diagnoses on the differential can be assisted by EMG (radiculopathy or
plexopathies), MRI (tumor or other causes of mass effect), and diagnostic injections.
Illustration A shows the anatomy of the five common sites of compressing in the radial tunnel.
Incorrect Answers:
1. Radiocapitellar arthritis would not be antagonized by stretch of the common extensors of the

wrist
3. Carpal tunnel syndrome is diagnosed by evidence of nerve compression of the median nerve
at the wrist and should not be associated with pain near the origin of the wrist extensors
4. Lateral epicondylitis can very much mimic radial tunnel syndrome; however, the location of the
pain and tenderness on exam can be a very helpful
5. Intersection syndrome is a chronic tenosynovitis of the ECRL and ECRB characterized by pain
at the intersection of the 1st and 2nd dorsal compartments of the wrist

Wartenberg's Syndrome
Author: Colin Woon
Topic updated on 05/31/14 6:28am

Introduction

Definition
o compressive neuropathy of thesuperficial sensory radial
nerve(SRN)
o also called "cheiralgia paresthetica"
o sensory manifestation only
o no motor deficits
Epidemiology
o incidence
rare
o demographics

male:female ratio is 1:4, more common in women


age bracket is 20-70 years
Pathoanatomy
o SRN compressed by scissoring action
of brachioradialis and ECRL tendons during forearm pronation
o also by fascial bands at its exit site in the subcutaneous plane
Associated conditions
o associated with De Quervain's disease in 20-50%
Prognosis
o spontaneous resolution of symptoms is common
o treatment outcomes
74% success after surgical decompression
Anatomy
The superficial sensory branch of the radial nerve
o arises from the bifurcation of the radial nerve in the proximal forearm
o travels deep to the brachioradialis in the forearm
o emerges from between brachioradialis and ECRL 9cm proximal to radial
styloid
o bifurcates proximal to the wrist
dorsal branch lies 1-3cm radial to Lister's tubercle
supplies 1st and 2nd web space
palmar branch passes within 2cm of 1st dorsal compartment,
directly over EPL
supplies dorsolateral thumb

Presentation
History
o may have history of trauma
forearm fracture
handcuffs
tight wrist band, wristwatch band, bracelet or plaster cast
Symptoms
o ill-defined pain over dorsoradial hand (does not like to wear
watch)
o paresthesias over dorsoradial hand
o numbness
o symptom aggravation by motions involving repetitive wrist flexion
and ulnar deviation
o no motor weakness
Physical exam
o provocative tests
Tinel's sign over the superficial sensory radial nerve (most
common exam finding)
wrist flexion, ulnar deviation and pronation for one

minute
Finkelstein test increases symptoms in 96% of patients
because of traction on the nerve
Imaging
Radiographs
o of limited value
o may demonstrate old forearm fracture
Studies
Electrodiagnostic tests
o EMG and NCV of limited value
Diagnostic injection
o diagnostic wrist block may temporarily relieve pain
Differential
De Quervain's tenosynovitis
o pain is not aggravated by wrist pronation, unlike Wartenberg Syndrome
Lateral antebrachial cutaneous nerve (LACN) neuritis
o positive Tinel's sign over LACN can be mistaken for positive Tinel's over
superficial sensory radial nerve
Intersection syndrome
o may have dorsoradial forearm swelling
o symptom exacerbation and "wet leather" crepitus on repeated wrist
flexion/extension

Treatment
Nonoperative
o rest, activity modification, NSAIDS, and wrist splints
indications
first line of treatment
techniques
avoid aggravating activities
remove inciting factors (e.g. tight wristwatch band)
o corticosteroid injection
although evidence to support this is limited
Operative
o surgical decompression
indications
symptoms persist after 6 months
Surgical Technique
Surgical Decompression
o approach
longitudinal incision volar to Tinel's sign
to avoid injury to LACN
to avoid tethering of incision scar over SRN

decompression technique
neurolysis and release of fascia between brachioradialis and ECRL

Complications
Failed decompression
Persistent pain and numbness
Wound dehiscence
Infection

Intrinsic Minus Hand (Claw Hand)


Author: Deborah Allen
Topic updated on 11/30/15 2:37pm

Introduction

Caused by imbalance between strong extrinsics and deficient intrinsics


Characterized by
o MCP hyperextension
o PIP & DIP flexion
Causes
o ulnar nerve palsy
cubital tunnel syndrome
ulnar tunnel syndrome
o median nerve palsy
Volkmann's ischemic contracture
leprosy (Hansen's disease)
failure to splint the hand in an intrinsic-plus posture following a crush
injury
o Charcot-Marie-Tooth disease (hereditary motor-sensory neuropathy)
o compartment syndrome of the hand
Pathoanatomy

Pathoanatomic components
o loss of intrinsics
leads to loss of baseline MCP flexion and loss of IP extension
o strong extrinsic EDC
leads to unopposed extension of the MCP joint
remember the EDC is not a significant extensor of the PIP joint
most of the MCP extension forces on the terminal insertion of th
central slip come from the interosseous muscles
o strong FDP and FDS
leads to unopposed flexion of the PIP and DIP
Presentation
Symptoms
o decreased hand function
Physical exam
o

MCP hyperextension and IP joint flexion


with an ulnar nerve palsy, the deformity will be worse in the 4th and 5t
digits (lumbricals innervated by the ulnar nerve)
not as severe in the 2nd and 3rd digits (lumbricals innervated by
median nerve)
functional weakness
unable to perform prehensile grasp
diminished grip and pinch strength
provocative tests
if MCP joints are brought out of hyperextension, the flexion deformity o
the DIP & PIP will correct

Treatment
Operative
o contracture release and passive tenodesis vs. active tendon transfer
indications
progressive deformity that is affecting quality of life

technique
goal is to prevent MCP joint hyperextension
(SBQ11.1) A 40-year-old male sheet metal worker sustained a crush injury to his hand. His hand
was treated in a short arm splint after closed reduction and percutaneous pinning of multiple
metacarpal fractures. The patients fractures healed uneventfully however, he presented six
months later with the deformity shown in Figure A. What pathoanatomic process is responsible
for his deformity?
1.

Volar plate laxity and tethering of the lateral bands at the proximal interphalangeal joints
3% (43/1601)

2.

FDP laceration distal to the origin of the lumbricals


1% (17/1601)

3.

Adhered FDP tendon of the middle finger


1% (16/1601)

4.

Imbalance between spastic intrinsics and weak extrinsics


17% (279/1601)

5.

Imbalance between strong extrinsics and deficient intrinsics


77% (1237/1601)

PREFERRED RESPONSE 5
The clinical presenatation is consistent with a claw hand deformity characterized by MCP
hyperextension and IP joint flexion.
Imbalance between strong extrinsics and deficient intrinsics is the pathoanatomic process of a
claw hand, also called intrinsic minus hand deformity. Intrinsic minus hand posture can result
from a variety of causes including ulnar or median nerve palsy, Volkmanns ischemic contracture,
leprosy, hereditary motor-sensory neuropathy, failure to splint a crush-injured hand using intrinsic
plus posture, or compartment syndrome of the hand, as was the case in this clinical vignette.
Ouellette et al performed a retrospective review of nineteen patients managed with fasciotomy
for compartment syndrome of the hand. They found that the most consistent clinical finding in
making the diagnosis of compartment syndrome was a tense, swollen hand in an intrinsic minus
position.
Dellaero et al, in their review of compartment syndrome of the hand, discuss the etiology,
diagnosis, and treatment of acute hand compartment syndrome. They emphasize that the main
goal in the management of ischemic contracture is restoration of function; however the return of
normal limb functionality is an unlikely result.
Figure A is a clinical photograph showing a classic claw hand deformity. Notice the MCP
hyperextension and IP joint flexion.

(OBQ04.33) Extrinsic imbalance from splinting a crushed hand with metacarpophalangeal joint
extension causes what characteristic hand deformity?
Review Topic
1.

Distal interphalangeal joint extension


4% (15/383)

2.

Ulnar subluxation of the metacarpophalangeal joints


2% (8/383)

3.

Proximal interphalangeal joint extension


13% (49/383)

4.

Proximal interphalangeal joint flexion


73% (279/383)

5.

Swan-neck deformity
8% (31/383)

PREFERRED RESPONSE 4
Failure to splint the hand in an intrinsic positive position leads to increased extrinsic finger flexor
tension, leading the DIP and PIP joints to have an increasing flexion position. Illustration A and B
show a clinical image and illustration of intrinsic minus hand.
von Schroeder et al present a Level 5 review of hand crush injuries. They conclude that early
diagnosis and treatment is critical, but the functional outcome is often poor with associated
Volkmann's contracture.

Intrinsic Plus Hand


Author : Mark karadsheh

Introduction
Caused by muscles imbalance between
spastic intrinsics (interosseoi and lumbricals)
weak extrinsics (FDS, FDP, EDC)
Characterized by
MCP flexion
PIP & DIP extension
Etiology
trauma
o direct trauma
o indirect trauma
o vascular injury
o compartment syndrome
rheumatoid arthritis
o MCP joint dislocations and ulnar deviation lead to spastic intrinsics
neurologic pathology
o traumatic brain injury
o cerebral palsy
o cerebrovascular accident
o Parkinson's syndrome
Pathoanatomy
spastic intrinsics
o leads to flexion of the MCP and extension of the IP joints

EDC weakness
o fails to provide balancing extension force to MCP joint
FDS & FDP weakness
o fail to provide balancing flexion force to PIP and DIP joints
Presentation
Symptoms
o

difficulty gripping large objects

Physical exam
o

inspection
MCP joint flexion and IP joint extension

provocative tests
Bunnell test (intrinsic tightness test)
differentiates intrinsic tightness and extrinsic tightness
positive test when PIP flexion is less with MCP extension than
with MCP flexion

Imaging
Radiographs
no radiographs required in diagnosis or treatment

Treatment
Nonoperative
o

passive stretching
indications
mild cases

Operative
o

proximal muscle slide

indications
less severe deformities when there is some remaining function
of the intrinsics (e.g., spastic intrinsics)
o

distal instrinsic release (distal to MP)


indications
more severe deformity involving both MCP and IP joints
dysfunctional intrinsic muscles (e.g., fibrotic)

Surgical techniques
Proximal muscle slide
o

techinque
subperiosteal elevation of interossei lengthens muscle-tendon unit

Distal intrinsic release


o

technique
resection of intrinsic tendon distal to the transverse fibers responsible
for MCP joint flexion

Boutonniere Deformity
Author : Chad Krueger

Introduction
A Zone III extensor tendon injury characterized by
o

PIP flexion

DIP extension

Mechanism
o

caused by rupture of the central slip over PIP joint from


laceration
traumatic avulsion (jammed finger)
capsular distension in rheumatoid arthritis

Pathoanatomy
o

pathoanatomic sequence includes


rupture of central slip
causes the extrinsic extension mechanism from the EDC
to be lost
prevents extension at the PIP joint
attenuation of triangular ligament
causes intrinsic muscles of the hand (lumbricals) to act as
flexors at the PIP joint
lumbricals also extend the DIP joint without an opposing
or balancing force
palmar migration of collateral bands and lateral bands
the lumbricals' pull becomes unopposed, pulling through
the base of the distal phalanx and volar to the PIP
causes PIP flexion and DIP extension

bone deformity
injury involves all three phalanges

the middle phalanx flexes on the proximal phalanx at the PIP


joint
the distal phalanx is hyperextended relative to the middle
phalanx at the DIP joint
Associated conditions
o

rheumatoid arthritis

pseudo-boutonniere
refers to PIP joint flexion contracture in the absence of DIP
extension

Anatomy
Muscle
o

lumbrical muscles
originate from the FDP and insert on the lateral bands

Ligament anatomy
o

extensor hood and central slip


the extrinsic extensor tendon joins the extensor hood at the
MCP
the central portion of the extensor hood forms the central slip
the central slip inserts onto the middle phalanx and acts to
extend the PIP joint

lateral bands
the lateral bands are formed from the deep head of the dorsal
interossi combining with the volar interossi
the lateral bands insert onto the base of the distal phalanx to
extend the DIP joint

triangular ligament
spans the two lateral bands, preventing them from subluxing
volarly

transverse retinacular ligament


prevents dorsal subluxation of the lateral bands

Blood supply
o

interosseous muscles

receive blood from vessels formed by a combination of the deep


palmer arch and the ulnar artery
Presentation
Physical exam
deformity
o

characterized by PIP flexion DIP extension

Elson test
o

is the most reliable way to diagnose a central slip injury before the
deformity is evident

bend PIP 90 over edge of a table and extend middle phalanx against
resistance.
in presence of central slip injury there will be
weak PIP extension
the DIP will go rigid
in absence of central slip injury DIP remains floppy because the
extension force is now placed entirely on maintaining extension
of the PIP joint; the lateral bands are not activated

Imaging
Radiographs
recommended view
o

radiographs are not required in evaluation and treatment of


Boutonniere deformity

Treatment
Nonoperative
o

splint PIP joint in full extension for 6 weeks


indications
acute closed injuries (< 4 weeks)
technique
encourage active DIP extension and flexion in splint
to avoid contraction of oblique retinacular ligament

complete part-time splinting for an additional 4-6 weeks


Operative
o

primary central band repair


indications
acute displaced avulsion fx (proximal MP avulsion seen
on x-ray)
open wound that needs I&D
lateral band relocation vs. terminal tendon tenotomy vs. tendon

reconstruction
indications
in chronic injuries after FROM is obtained with therapy or
surgical release
technique
terminal tendon tenotomy (modified Fowler or Dolphin
tenotomy)(never central slip tenotomy)
secondary tendon reconstruction (tendon graft, Littler,
Matev)
triangular ligament reconstruction
o

PIP arthrodesis
indications
rheumatoid patients
painful, stiff and arthritic PIP joint

Questions :
1. Chronic injury to what anatomic structure can lead to a boutonnire deformity of the
finger?
1.

terminal extensor tendon


2% (28/1667)

2.

sagittal band
8% (131/1667)

3.

volar plate

3% (50/1667)
4.

flexor digitorum profundis tendon insertion


2% (37/1667)

5.

central slip of the extensor tendon


85% (1413/1667)
PREFERRED RESPONSE
Rupture of the central slip of the extensor tendon and subsequent subluxation of the
lateral bands leads to a boutonnire deformity, which is characterized by PIP flexion and DIP
extension. Central slip injuries can be caused by a laceration or traumatic avulsion. In the
listed reference, Imatami et al treated a series of central slip injuries associated with
attachment fractures successfully with ORIF. As stated by Tuttle et al, rupture of the terminal
extensor tendon leads to a mallet finger. Sagittal band injury can lead to subluxation of the
extensor tendon at the level of the MCP joint. Chronic volar plate injuries can lead to swan
neck deformities. Avulsion of the FDP insertion leads to a jersey finger. Illustrations A and B
are a clinical photograph and anatomic diagram of a boutonneire deformity.
Illustrations:

2. A 54-year-old female presents with a hand deformity. A surgical procedure is being


considered that relocates the lateral bands dorsally to counteract the pathophysiology of the
deformity. Which of the following deformities does this patient most likely have?
Topic

Review

1.

Boutonneire finger deformity


82% (1288/1562)

2.

Lumbrical plus finger deformity


2% (38/1562)

3.

Mallet finger deformity


2% (26/1562)

4.

Jersey finger deformity


0% (6/1562)

5.

Swan neck finger defomity


13% (201/1562)
PREFERRED RESPONSE
Boutonniere deformity is characterized with the PIP in flexion and the DIP in
hyperextension as shown in Illustration A. It is caused by central slip rupture or attenuation
(secondary to capsular distention, e.g., rheumatoid arthritis), laceration, or traumatic
disruption. Volar subluxation of the lateral bands due to incompetence or disruption of the
triangular ligaments leads to increased deformity as the lateral bands become flexors of the
PIP. Relocation of the lateral bands to their original dorsal position to counteract the
pathophysiology of the deformity is an option for patients that have an approximately 40
degree active flexion contracture but full passive extension
Illustrations :

Swan Neck Deformity


Author : Derek Moore

Introduction
Characterized by
o

hyperextension of PIP

flexion of DIP

Caused by
o

lax volar plate

imbalance of muscle forces on PIP (extension force > flexion force)

Injuries include
o

MCP joint volar subluxation (rheumatoid arthritis)

mallet finger

FDS laceration

intrinsic contracture

Seen in rheumatoid arthritis

Pathoanatomy
Primary lesion is lax volar plate that allows hyperextension of PIP. Causes include
o

trauma

generalized ligament laxity

rheumatoid arthritis

Secondary lesion is imbalance of forces on the PIP joint (PIP extension forces that is
greater than the PIP flexion force). Causes of this include
o

mallet injury
leads to transfer of DIP extension force into PIP extension forces

FDS rupture
leads to unopposed PIP extension combined with loss of integrity of
the volar plate

intrinsic contracture
tethering of the lateral (collateral) bands by the transverse retinacular
ligament as a result of PIP hyperextension.
if the lateral (collateral) bands are tethered, excursion is restricted and
the extension force is not transmitted to the terminal tendon, and is
instead transmitted to the PIP joint

MCP joint volar subluxation


caused by rheumatoid arthritis

Presentation
Symptoms
o

snapping and locking of the fingers

Physical exam
o

hyperextension of PIP

flexion of DIP

Imaging
Radiographs
recommended views
o

AP and lateral view of the affected hand

Treatment
Nonoperative
o

double ring splint


indications
can prevent hyperextension of PIP

Operative
o

volar plate advancement and PIP balancing with central slip tenotomy
indications
progressive deformity
technique
address volar plate laxity with volar plate advancement
correct PIP joint muscles imbalances with either
FDS tenodesis indicated with FDS rupture
spiral oblique retinacular ligament reconstruction
central slip tenotomy (Fowler)

Quadrigia Effect
Author : Tracy Jones

Introduction
The quadrigia effect is characterized by an active flexion lag in fingers adjacent to a
digit with a previously injured or repaired flexor digitorum profundus tendon.
Mechanism
o

most commonly caused by a functional shortening of the FDP tendon due to


over-advancement of the FDP during tendon repair
>1 cm advancement associated with quadrigia
adhesions
retraction of the tendon
"over-the-top" FDP repair of the distal phalanx after amputation

Pathoanatomy
o

FDP tendons of long, ring, and little fingers share a common muscle belly
therefore excursion of the combined tendons is equal to the shortest
tendon
improper shortening of a tendon during repair results in
inability to fully flex adjacent fingers

Anatomy
Flexor digitorum profundus
Zones of the flexor tendons
o

most injuries resulting in quadrigia involve Zone I

Presentation
Symptoms
o

inability to fully flex the fingers of the hand adjacent to the injured finger

patient may complain of "weak grip"

Physical exam
o

upon making a fist the fingers adjacent to the injured digit will not reach full
flexion

grip strength decreased

Imaging
Radiographs
usually not required

Treatment
Nonoperative
o

observation
indications
mild symptoms not affecting quality of life

Operative

release FDP of injured digit


indications
severe symptoms limiting function

Questions :
1. A butcher sustains a traumatic amputation of the ring finger through the distal
interphalangeal joint. He is brought to the operating room where the flexor digitorum
tendon is retrieved and advanced to the distal stump. Three months after surgery the
patient notes that when he makes a fist, only his ring finger tip reaches the palm. What is
this patient's clinical problem?

1.

mallet finger
2% (25/1542)

2.

swan neck deformity


1% (10/1542)

3.

boutonniere deformity
1% (15/1542)

4.

lumbrical plus deformity


6% (93/1542)

5.

quadrigia syndrome
90% (1392/1542)
PREFERRED RESPONSE
Quadrigia syndrome occurs when a flexor digitorum profundus (FDP) tendon is
shortened and advanced. Malerich et al found the tolerable degree of FDP advancement was
1 cm. The common muscle belly of the FDP prevents the tendons to the other fingers from
reaching full excursion. Treatment is release of the shortened tendon. Lumbrical plus occurs

when the FDP tendon retracts and causes paradoxical interphalangeal extension when trying
to flex. Mallet finger is an injury to the terminal extensor tendon. Boutonniere deformity
occurs from central slip injury and results in PIP flexion and DIP extension. Swan-neck
deformity consists of hyperextension at the PIP joint with flexion at the DIP joint typically
caused by volar plate attenuation.
2. A 35-year-old butcher inadvertently lacerates his ring finger FDP tendon at the level of the
DIP joint which is subsequently repaired. Following the operation he notes the inability to
fully flex his long and small fingers at the DIP joints with attempted fist clenching as well
as a weak grip. Which of the following intraoperative maneuvers was likely responsible
for this?

1.

FDP reconstruction with a long tendon graft


3% (92/2672)

2.

FDS to FDP transfer at level of the A2 pulley


2% (56/2672)

3.

Inadequate repair of the C3, A4 and A5 pulleys


2% (44/2672)

4.

Distal advancement of lumbricals


3% (81/2672)

5.

Overtensioning of the FDP tendon


89% (2387/2672)
PREFERRED RESPONSE
The clinical presentation is most consistent with the quadrigia effect which is caused by
overtensioning of the FDP tendon during surgical repair. The FDP tendons share a common
muscle belly and have many interconnections. Overtensioning one tendon has a reciprocal

effect on the length-tension curve of the remaining three muscle-tendon units, weakening grip
strength in these digits. Malerich et al performed a cadaveric study looking at FDP
advancement on hand function. They determined advancement >1cm can lead to an
imbalance of muscle function in the profundus system. Kaufmann et al. studied maximal grip
strength and point of contact in the extrinsic system. They determined that FDP grip strength
was optimized when the FDP point of contact was at the DIP. Thus moving the FDP insertion
point distal or advancing a lacerated FDP tendon leads to a decrease in grip strength.

Lumbrical Plus Finger


Author : Colin Woon

Introduction
Characterized by paradoxical extension of the IP joints while attempting to flex the
fingers
Epidemiology
o

location
most common in middle finger (2nd lumbrical)
FDP 3, 4, 5 share a common muscle belly
cannot independently flex 2 digits without pulling on
the third
index finger has independent FDP belly
when making a fist following FDP2 transection, it is
possible to only contract FDS2 (and not FDP2), thus
avoiding paradoxical extension

Pathophysiology
o

mechanism
FDP disruption distal to the origin of the lumbicals (most common)
can be due to
FDP transection
FDP avulsion

DIP amputation
amputation through middle phalanx shaft

"too long" tendon graft


o

pathoanatomy
lumbricals originate from FDP
with FDP laceration, FDP contraction leads to pull on lumbricals
lumbricals pull on lateral bands leading to PIP and DIP extension of
involved digit
with the middle finger, when the FDP is cut distally, the FDP shifts
ulnarly (because of the pull of the 3rd lumbrical origin)(bipennate)
this leads to tightening of the middle finger lumbrical (2nd
lumbrical, unipennate), and amplifies the "lumbrical plus"
effect

Anatomy
Lumbricals
1st and 2nd lumbricals
o

unipennate

median nerve

originate from radial side of FDP2 and FDP3 respectively

3rd and 4th lumbricals


o

bipennate

ulnar nerve

3rd lumbrical originates from FDP 3 & 4

4th lumbrical originates from FDP 4 & 5

all insert on radial side of extensor expansion

Presentation
History
o

recent volar digital laceration (FDP transection) or sudden axial traction on


flexed digit (FDP avulsion)

Symptoms
o

notices that when attempting to grip an object or form a fist, 1 digit sticks out
or gets caught on clothes

Physical exam
o

paradoxical IP extension with grip (fingers extend while holding a beer can)

Treatment
Operative
tenodesis of FDP to terminal tendon or reinsertion to distal phalanx
o

indications
FDP lacerations

lumbrical release
o

indications
if FDP is retracted or segmental loss makes it impossible to fix

contraindications

do not transect lumbricals 1 & 2 if there is concomitant ulnar nerve


palsy
with ulnar nerve paralysis, the interosseous muscles are also lost
(interosseus muscles extend the IP joints)
o

technique
transect at base of flexor sheath (in the palm)

Questions :
What is a potential complication of an amputation at the level of the distal interphalangeal
joint?

1.

Central slip rupture


3% (48/1549)

2.

Swan neck deformity


8% (126/1549)

3.

Boutonniere deformity
9% (132/1549)

4.

Lumbrical plus finger


46% (706/1549)

5.

Quadrigia effect
34% (525/1549)
PREFERRED RESPONSE
A lumbrical plus finger is descibed as paradoxical extension of the IP joints while
attempting to flex the fingers. In the case a lumbrical plus finger secondary to a DIP
amputation, the PIP will extend upon attempted finger flexion.

The review article by Parkes describes how the lumbricals originate from the FDP.
When the FDP is lacerated or amputated, FDP contraction leads to pull on the lumbricals.
This leads to shortening of the lateral bands and paraodoxical PIP and DIP extension.
(Illustrations A-C). There are several causes of lumbrical plus finger including (1) FDP
laceration or rupture distal to the lumbrical origin, (2) amputation of the DIP distal to central
slip insertion, and (3) excessively long flexor tendon graft. Treatment consists of lumbrical
release at the level of the flexor sheath in the palm, which then prevents paradoxical PIP
extension.
Quadrigia may occur when the profundus is advanced of greater than 1 cm in repair.
The FDP tendons share a common muscle belly, and distal advancement of one tendon will
effect the flexion strength of the adjacent digits.
Illustrations:

Ulnar Variance
Author : Colin Woon

Introduction
Definition
o

length of the ulna compared to the radius

measured in shoulder abducted 90deg, elbow flexed 90deg, forearm neutral,


hand aligned with forearm axis

Epidemiology
o

demographic
male:female relationship
UV is lower in males than females
age bracket
UV increases with age

risk factors
positive UV may be present in child gymnasts
distal radial growth plate injury leading to premature closure of
distal radial physis

Pathophysiology
o

congenital
Madelung deformity (positive UV)
reverse Madelung deformity (negative UV)

trauma/mechanical
distal radius/ulnar fracture with shortening
growth arrest (previous Salter-Harris fracture)
DRUJ injuries (Galeazzi and Essex-Lopresti)

iatrogenic
joint leveling procedures (radial or ulnar shortening/lengthening)

radial head resection (positive UV)


Associated conditions
o

positive ulnar variance


ulnar abutment syndrome
SLD
TFCC tears
arthrosis
ulnar head
lunate
triquetrum
lunotriquetral ligament tears

negative ulnar variance


Kienbock's disease
ulnar impingement syndrome
ulna impinges on the radius proximal to the sigmoid notch

Ulnar Variance
Ulnar

Length Difference (ulnar -

Load Passing Through

Load Passing

Variance

radial length)

Radius

Through Ulna

Neutral

0 (<1mm)

80%

20%

Positive

+2.5mm

60%

40%

Negative

-2.5mm

95%

5%

Neutral

Positive

Negative

Anatomy
Neutral ulnar variance (ulnar zero)
o

difference between ulnar and radial length is <1mm

Positive ulnar variance


o

ulnar sided wrist pain from increased impact stress on the lunate and
triquetrum

UV becomes more positive in pronation

UV becomes more positive during grip

Negative ulnar variance


o

UV decreases in supination

Imaging
Radiographs
o

recommended view
PA of the wrist with shoulder abducted 90 deg, elbow flexed 90 deg,
neutral forearm rotation

Method to determine ulnar variance


o

draw 2 lines

1 line tangential to the articular surface of the ulna and perpendicular


to its shaft
1 line tangential to the lunate fossa of the radius and perpendicular to
its shaft.
o

measure the distance between these 2 lines (normal is 0mm)

if the ulnar tangent is distal to the radial tangent = positive UV

if the ulnar tangent is proximal to the radial tangent = negative UV

MRI
o

can estimate but not quantify degree of UV

because specific wrist position cannot be duplicated in MRI

Treatment
Depends on specific condition
ulnar abutment syndrome
TFCC tears
Kienbock's disease

Questions :
In a patient with -2.5mm of ulnar variance, which of the following statements best describes
the distribution of compressive load across the wrist?

1.

Approximately 50% of the wrist load is accepted by distal radius and 50% is accepted by
the distal ulna
1% (3/396)

2.

Approximately 80% of the wrist load is accepted by the distal radius and 20% is accepted
by the distal ulna
27% (107/396)

3.

Approximately 80% of the wrist load is accepted by the distal ulna and 20% is accepted by
the distal radius
4% (14/396)

4.

Approximately 95% of the wrist load is accepted by the distal radius and 5% is accepted by
the distal ulna
64% (253/396)

5.

Approximately 60% of the wrist load is accepted by the distal radius and 40% is accepted
by the distal ulna
3% (13/396)
PREFERRED RESPONSE
Ulnar variance describes the cranio-caudal position of the distal ulna in relation to the
distal radius at the wrist. In neutral ulnar variance, 80% of the compressive load across the
wrist is accepted by the distal radius, and 20% is accepted by the distal ulna. With -2.5mm of
ulnar variance (negative ulnar variance), approximately 5% of the wrist load is accepted by
the distal ulna. With +2.5mm of ulnar variance (positive ulnar variance), approximately 40%
of the wrist load is accepted by the distal ulna.
As discussed in the biomechanical study by Palmer and Werner, the loading
characteristics of the wrist are dependent on the radio/ulnar variance. Specifically, a 2.5 mm
increase in ulnar variance increases load accepted by ulno-carpal joint from 18% to 42%; a
2.5 mm decrease in the ulno-carpal variance will decrease the load accepted by the ulnocarpal joint to 4.3%.
Friedman and Palmer review the clinical diagnosis, pathophysiology, and treatment of ulnar
impaction syndrome.

TFCC Injury
Author : David Abbasi, Mark Vitale

Introduction
Mechanism of TFCC injury
Type 1 traumatic injury
o

mechanism
most common is fall on extended wrist with forearm pronation
traction injury to ulnar side of wrist

traction injury to ulnar wrist

Type 2 degenerative injury


o

associated with positive ulnar variance

associated with ulnocarpal impaction

Anatomy
TFCC made up of
o

dorsal and volar radioulnar ligaments


deep ligaments known as ligamentum subcruentum

central articular disc

meniscus homolog

ulnar collateral ligament

ECU subsheath

origin of ulnolunate and ulnotriquetral ligaments

Blood supply

periphery is well vascularized (10-40% of the periphery)

central portion is avascular

Origin
o

dorsal and volar radioulnar ligaments originate at the sigmoid notch of the
radius

Insertion
o

dorsal and volar radioulnar ligaments converge at the base of the ulnar styloid

Classification
Class 1 - Traumatic TFCC Injuries
1A

Central perforation or tear

1B

Ulnar avulsion (without ulnar styloid fx)

1C

Distal avulsion (origin of UL and UT ligaments)

1D

Radial avulsion

Class 2 - Degenerative TFCC Injuries


2A

TFCC wear and thinning

2B

Lunate and/or ulnar chondromalacia + 2A

2C

TFCC perforation + 2B

2D

Ligament disruption + 2C

2E

Ulnocarpal and DRUJ arthritis + 2D

Presentation
Symptoms
o

wrist pain

turning a door key often painful

Physical exam
o

positive "fovea" sign


tenderness in the soft spot between the ulnar styloid and flexor carpi
ulnaris tendon, between the volar surface of the ulnar head and the
pisiform
95% sensitivity and 87% specificity for foveal disruptions of TFCC or
ulnotriquetral ligament injuries

pain elicited with ulnar deviation (TFCC compression) or radial deviation


(TFCC tension)

Imaging
Radiographs
o

usually negative

zero rotation PA view evaluates ulnar variance

dynamic pronated PA grip view may show pathology

Arthography
o

joint injection shows extravasation

MRI
o

has largely replaced arthrography

tear at ulnar part of lunate indicates ulnocarpal impaction

sensitivity = 74-100%

Arthroscopy
o

most accurate method of diagnosis

indicated in symptomatic patients after failing several months of splinting and


activity modification

Differential
Differential for ulnar sided wrist pain

Treatment
Nonoperative
o

immobilization, NSAIDS, steroid injections


indications
all acute Type I injuries
first line of treatment for Type 2 injuries

Operative
o

arthroscopic debridement
indications
type 1A
diagnostic gold standard

arthroscopic repair
indications
type 1B, 1C, 1D
best for ulnar and dorsal/ulnar tears
generally acute, athletic injuries more amenable to repair than
chronic injuries
outcomes
patient should expect to regain 80% of motion and grip strength
when injuries are classified as acute (<3 months)

ulnar diaphyseal shortening


indications
Type II with ulnar positive variance is > 2mm
advantage of effectively tightening the ulnocarpal ligaments
and is favored when LT instability is present

Wafer procedure
indications
Type II with ulnar positive variance is < 2mm
type 2A-C

limited ulnar head resection


indications
type 2D

Darrach procedure

indications
contraindicated due to problems with ulnar stump instability

Techniques
Arthroscopic debridement
o

approach
arthroscopic approach to the wrist
performed through combination of 3-4 and 6R portal

technique
maintain 2 mm rim peripherally otherwise joint can become unstable

pros & cons


not effective if patient has ulnar positive variance
80% of patients obtain good relief of pain

Arthroscopic repair
o

approach
arthroscopic approach to the wrist

technique
many techniques exist such as outside-in and inside-out
generally suture based repair

pros & cons


only works for peripheral tears where blood supply is present
patient immobilized for 6 weeks

complications
ECU tendonitis from suture knot
dorsal sensory nerve injury

Ulnar diaphyseal shortening


o

approach
dorsal approach to the forearm

technique
osteotomy of the diaphysis or metaphysis followed by plate fixation

pros & cons


can address > 2 mm ulnar variance
requires immobilization and time for fracture healing
can help tension the ulnocarpal ligaments

complications
nonunion
hardware irritation necessitating removal

Wafer procedure
o

approach
dorsal approach to the forearm

technique
ulnar cortex is not disrupted
do not extend bone removal into the DRUJ

pros & cons


intrinsic stability of ECU, TFCC, and ulnar periosteum obviate need
for plate fixation

Limited ulnar head resection


o

approach
arthroscopic approach to the wrist

technique
removal of approximately 2-4 mm of bone under the TFCC
distal ulnar burred through central TFCC defect

pros & cons


can be technically difficult to obtain level shortening through TFCC
window
only applicable when patient has < 2mm of ulnar variance

Darrach procedure
o

approach
dorsal approach to the forearm

technique
resection of the distal 1-2cm of the distal ulna
TFCC should be approximated to the wrist capsule

pros & cons


salvage procedure for pain relief only
distal joint is unstable

complications
ECU tendon can sublux over remaining ulna causing pain

Questions :
1. A 19-year-old football player suffers a fall onto a pronated, extended wrist. He has pain
with resisted ulnar deviation and is tender to palpation just distal to the ulnar styloid. He
has no tenderness over the extensor carpi ulnaris (ECU) tendon. Current radiographs are
shown in in Figures A and B and and MRI of the wrist is shown in FIgure C. Which of the
following is the most likely diagnosis?

Review Topic

FIGURES:

1.

ECU tendon rupture


1% (8/1227)

2.

Triangular fibrocartilaginous complex (TFCC) tear


92% (1130/1227)

3.

Hook of hamate fracture


5% (63/1227)

4.

Scapholunate ligament injury


1% (8/1227)

5.

Perilunate dislocation
1% (16/1227)

PREFERRED RESPONSE
Fall from standing onto an extended and pronated wrist is a risk factor for injuries to
the soft tissues of the wrist. The structures at risk include the triangular fibrocartilaginous
complex (TFCC), the lunotriquetral ligament, ulnolunate ligament, hook of hamate, ulnar
styloid, and the extensor carpi ulnaris (ECU) tendon sheath. Pain with resisted ulnar deviation
and ulnar catching are all concerning for injury to the TFCC. MRI is useful for diagnosing
TFCC tears ( Illustration A shows another example).
Papapetropoulos et al in their review article discuss the evaluation and arthroscopic
treatment of TFCC injuries. Specifically they discuss that most tears in athletes are acute and
amenable to repair by repair of the dorsal tear to the ECU tendon sheath.
Cohen in his review of injuries in athletes discusses scapholunate ligament,
lunotriquetral ligament, and midcarpal injuries. Of note he divides scapholunate and
lunotriquetral ligament injuries into dissociative lesions (abnormal motion within proximal
carpal bones) vs. midcarpal lesions which are generally considered nondissociative (abnormal
motion between proximal and distal carpal bones).
Rettig in his review of sports injuries of the extremities discusses the Palmer
classification of TFCC tears. Specifically he notes that central tears are more associated with
repetitive activities in patients with positive ulnar variance.
Incorrect Answers:
Answer 1: The patient is not tender in the region of the ECU tendon sheath.
Answer 3: The carpal tunnel view radiograph shows no hook of hamate fracture.
Answer 4 and 5: Wrist radiographs shows no scapholunate widening or perilunate
dislocation. Physical exam in this case is more consistent with a TFCC injury.
Illustrations:

2. Which of the following structures is an anatomical component of the triangular


fibrocartilage complex?

1.

Extensor carpi ulnaris tendon sheath


67% (647/967)

2.

Lunotriquetral interosseous ligament


11% (103/967)

3.

Extensor digiti minimi tendon sheath


2% (18/967)

4.

Radioscaphocapitate ligament
6% (59/967)

5.

Flexor carpi ulnaris tendon sheath


14% (133/967)
PREFERRED RESPONSE
The extensor carpi ulnaris tendon sheath is part of the triangular fibrocartilage
complex (TFCC).
Palmer et al studied the anatomy and function of the TFCC through anatomical
dissections and biomechanical testing. The TFCC was found to be composed of the sheath of
the extensor carpi ulnaris (ECU), an articular disc, the dorsal and volar radioulnar ligaments,
the meniscus homologue, and the ulnar collateral ligament. Biomechanically, they
determined that the TFCC functions as a cushion at the ulnocarpal interface, and is a major
stabilizer of the DRUJ.
Nakamura et al histologically examined the origins and insertions of the TFCC in
fresh-frozen cadaver wrists. They found that the floor of the ECU sheath originated from the
dorsal side of the fovea of the ulna, through an arrangement of Sharpey's fibers. Illustration
A shows the anatomy of the TFCC.

Illustrations:

Ulnocarpal Abutment Syndrome


Author : Team Orthobullets

Introduction
Syndrome cause by excessive impact stress between ulna and carpal bones (primarily
lunate)
o

positive ulnar variance

Pathoanatomy
o

in a wrist with +2 mm ulnar variance approximately


40% of the load goes to the ulna
60% to the radius

in a normal neutral wrist approximately


20% of the load goes to the ulna
80% to the radius

Associated conditions
o

positive ulnar variance can be seen in the setting of:


scapholunate dissociation
TFCC tears
lunotriquetral ligament tears
radial shortening from previous Colles fracture

Presentation
Symptoms
o

pain on dorsal side of DRUJ

increased pain with ulnar deviation of wrist

pain with axial loading

ulna sided wrist pain

Physical exam

Ballottement test
dorsal and palmar displacement of ulna with wrist in ulnar deviation
positive test produces pain

Nakamura's ulnar stress test


ulnar deviation of pronated wrist while axially loading, flexing and
extending the wrist
positive test produces pain

fovea test
used to evaluate for TFCC tear or ulnotriquetral ligament tear
performed by palpation of the ulnar wrist between the styloid and FCU
tendon

Imaging
Radiographs
o

recommended views
AP radiograph with wrist in neutral supination/pronation and zero
rotation
required to evaluate ulnar variance
pronated grip view
increases radiographic impaction
arthrography can show TFCC tear and lunotriquetral ligament tear

findings
ulna positive variance
sclerosis of lunate and ulnar head

MRI
o

evaluate for TFCC tears which may be caused by ulnocarpal impingement and
often influences treatment

Differential
Ulnar sided wrist pain
DRUJ instability or arthritis
TFCC tear
LT ligament tear
pisotriquetral arthritis

ECU tendonitis or instability

Treatment
Nonoperative
o

supportive measures
indications
may attempt supportive measures as first line of treatment

Operative
o

ulnar shortening osteotomy


indications
most cases of ulnar positive variance
most cases of DRUJ incongruity

Wafer procedure
technique
2 to 4mm of cartilage and bone removed from under TFCC
arthroscopically

Darrach procedure (ulnar head resection)


indications
reserved for lower demand patients
complications
risk of proximal ulna stump instability

Sauv-Kapandji procedure
indications
good option for manual laborers
technique
creates a distal radioulnar fusion and a ulnar pseudoarthrosis
proximal to the fusion site through which rotation can occur

ulnar hemiresection arthroplasty


indications
usually requires an intact or reconstructed TFCC
appropriate treatment option in the presence of post-traumatic
DRUJ with concomitant distal ulnar degenerative changes

ulnar head replacement


indications
severe ulnocarpal arthrosis
salvage for failed Darrach
outcomes
early results are promising, long-term results pending

Techniques
Ulnar shortening osteotomy
approach
o

subcutaneous to ulna

technique
o

often combined with arthroscopic TFCC repair

Questions :
1. A 32-year-old carpenter has a 6-month history of ulnar-sided wrist pain that is worsened
opening a jar, squeezing a wet towel, typing, or changing a gearshift. Radiograph and MRI
images are detailed in Figures A through C. All of the following concerning ulnar
shortening osteotomy are true EXCEPT:
FIGURES:

1.

Care should be taken to avoid the dorsal sensory branch of the ulnar nerve
1% (24/1954)

2.

Results are encouraging even for those with degenerative changes in the distal radioulnar
joint
42% (821/1954)

3.

Placement of the plate to the dorsal surface of the ulna can cause tendinitis of the extensor
carpi ulnaris
2% (30/1954)

4.

Concomitant arthroscopy may be indicated for patients with concurrent tears of the
triangular fibrocartilage complex
16% (314/1954)

5.

Degenerative cystic changes of the ulnar carpal bones can resolve after the ulnar shortening
osteotomy
39% (759/1954)
PREFERRED RESPONSE
For patient's with ulnar impaction syndrome, concomitant arthrosis in the distal
radioulnar joint (DRUJ) is a contraindication to ulnar shortening osteotomy.
Ulnar impaction syndrome is caused by abutment of the ulnar head into the carpal
bones. It is worsened by activities that have wrist rotation and ulnar deviation. A positive
ulnar variance with or without cystic changes of the carpus is often seen on plain radiographs.
Coexisting central TFCC tears are common and can be addressed by simultaneous
arthroscopic or open dbridement.
Baek et al. describes 31 patients that had improved Gartland and Werley scores
following ulnar shortening osteotomy. They also noted that all patients with degenerative
cystic changes had resolution of the cysts at 1-2 year followup and they include a detailed
outline of their surgical technique.
Chun et al. similarly reviewed 30 wrists of 27 patients with ulnar impaction syndrome
with very good outcomes with minimal complications and no ulnar nonunions following
ulnar shortening osteotomy. Exclusion criteria included any exisiting arthrosis in the DRUJ
Figure A is a plain radiograph noting ulnar positive variance and mild cystic changes
in the lunate. Figures B and C are T1 and T2 MRI images of the wrist noting increased signal
in both the lunate and ulnar head.
Incorrect answers:
Answer 1, 3, 4, and 5 are all important factors to consider when performing ulnar shortening
osteotomy.

Illustrations:

2. A 42-year-old construction worker presents with pain in his right wrist. A current
radiograph of the wrist is shown in Figure A. He reports that rotating activities, such as
turning a screw driver, are bothersome and the pain is preventing him from working. A
current MRI reveals a TFCC tear, and nonsurgical treatment has failed to provide relief.
Treatment should now consist of:
FIGURES:

1.

Repair of the ulnar styloid nonunion


1% (7/1144)

2.

Darrach resection of the distal ulna


17% (195/1144)

3.

Complete ulnar head resection


2% (28/1144)

4.

Ulnar hemiresection arthroplasty and TFCC reconstruction/repair


74% (841/1144)

5.

Isolated arthroscopic TFCC reconstruction


6% (66/1144)
PREFERRED RESPONSE
The clinical presentation is consistent with DRUJ arthritis in a heavy laborer. Of the
options listed, ulnar hemiresection arthroplasty with concurrent TFCC reconstruction or
repair would be the most appropriate treatment.
While there are multiple treatment options, the ulnar hemiresection arthroplasty with
concurrent TFCC reconstruction or repair is considered most appropriate in heavy laborers,
as it would likely resolve the pain and enable them to return to work sooner. The TFCC
should be intact when performing an ulnar hemiresection arthroplasty to prevent distal ulna
instability with forearm rotation. One could also consider performing a Suave-Kapandji
procedure. This procedure creates a distal radioulnar fusion and an ulnar pseudarthrosis
proximal to the fusion site through which rotation can occur. The advantage is that the
ulnocarpal joint is not sacrificed, and a stable wrist is created.
Scheker et al reported on the outcome of ulnar shortening performed on 32 wrists with
early osteoarthritis of the DRUJ. The postoperative wrist ratings were 7/32 excellent, 11/32
good, 9/32 fair, 5/32 poor, with plate irritation being the most frequent postoperative
complication.
Figure A is a radiograph showing significant DRUJ arthritis. Illustration A shows
ulnar hemiresection arthroplasty. Illustration B shows a Darrach procedure. Illustration C
shows a Sauve-Kapandji procedure. Illustration D is a treatment schematic of TFCC
reconstruction.
Incorrect Answers:
Answer 1: There is no obvious ulnar styloid non-union.
Answer 2: As mentioned in Miller's review text, the Darrach procedure is typically reserved
for low-demand, elderly patients and may lead to painful proximal ulna stump instability.

Answer 3: Complete ulnar head resection is not indicated.


Answer 5: TFCC reconstruction will not improve or treat the DRUJ arthritic changes.
Illustrations:

D
3. An ulnar shortening osteotomy would be MOST indicated for which of the following
patients presenting with longstanding ulnar sided wrist pain refractory to conservative
measures?

1.

34-year-old female with an ulnar neutral wrist and distal radioulnar joint incongruity
1% (10/1467)

2.

34-year-old female with an ulnar positive wrist and distal radioulnar joint incongruity
89% (1310/1467)

3.

34-year-old female with an ulnar negative wrist and distal radioulnar joint incongruity
2% (23/1467)

4.

78-year-old female with ulnar positive wrist and distal radioulnar joint arthritis
8% (115/1467)

5.

78-year-old female with ulnar negative wrist and distal radioulnar joint arthritis
0% (4/1467)
PREFERRED RESPONSE
Ulnar shortening osteotomy is the best procedure for young adults with longstanding
ulnar sided wrist pain due to ulnar positive variance and associated distal radioulnar joint
(DRUJ) incongruity. Ulnar positive variance causes an "ulnar impaction syndrome" as the
distal ulnar styloid can cause damage to the triangular fibrocartilage complex (TFCC), and
ulnocarpal joint (illustration A.)
Advantages of an ulnar shortening osteotomy include preservation of ulnar dome
articular cartilage and DRUJ joint, and also tightens the TFCC and ulnocarpal ligaments as
the distal ulna is translated and fixed proximally after the osteotomy.
It is also important to note that ulnar shortening in the setting of preoperative DRUJ
incongruity may simultaneously decrease ulnocarpal abutment and improve congruity at the
distal radioulnar articulation. One specific instance in which to avoid an ulnar shortening in
an ulnar positive wrist with DRUJ incongruity is a joint with a reverse oblique inclination in
the coronal plane. This may create abnormally high radioulnar contact and may lead to joint
degeneration
Illustrations:

Ulnar Styloid Impaction Syndrome


Author : Richard Yoon

Introduction
Epidemiology
o

incidence
common cause of ulnar-sided wrist pain

demographics
more prevalent in Asians than Whites
more positive ulnar variance

Pathophysiology
o

pathoanatomy
impaction between ulnar styloid tip and triquetrum that is seen in
patients with excessively long ulnar styloids or ulna positive wrists

Associated conditions
o

radial malunion

congenitally short radius

premature radial physeal closure

Prognosis
o

little known about natural history

Anatomy
Ulnocarpal joint
transmits about 20% of the load through the wrist
o

increasing ulnar length by 2.5mm relative to the radius increases this load up
to 50%

pronation and hand grasp both increase elative ulnar variance and transmission
forces across the wrist

Classification
Ulnar Variance
Ulnar

Length Difference (ulnar - radial Load Passing Through Load Passing Through

Variance

length)

Radius

Ulna

Neutral

0 (<1mm)

80%

20%

Positive

+2.5mm

60%

40%

Negative

-2.5mm

95%

5%

Neutral

Positive

Negative

Presentation
Symptoms
o

ulnar side wrist pain

pain with pronation or grip

Physical exam
o

inspection
pain and swelling
tenderness along ulnar styloid and/or triangular fibrocartilage complex
(TFCC)

motion
limited range of motion due to pain

ulnar stress test


maximum ulnar deviation, axial loading, rotation from supination to
pronation to reproduce symptoms

Imaging
Radiographs
o

posteroanterior (PA) view to determine ulnar variance


excessive length determined by subtracting ulnar variance from ulnar
styloid length and dividing this by the width of the ulnar head (<.22 is
normal)
may exhibit subchondral sclerosis, cyst formation on ulnar side

pronated grip PA view


evaluate for any dynamic ulnar variance

contralateral comparison views

MRI
o

can help evaluate TFCC and the lunotriquetral interossesous ligament (LTIL)

Treatment
Nonoperative
o

activity modifications, NSAIDS, steroid injections


indications
first line of treatment
technique
rest should be tried for a minimum of 6-12 weeks

Operative
o

ulnar shortening osteotomy


currently, the gold standard

partial ulnar styloidectomy (Wafer procedure)


can be done open or arthroscopically
encouraging early results, but no superiority established

Complications
Non-union
Tendon rupture
Persistent pain/hardware irritation
Infection

Kienbocks disease
Author : Mark Karadsheh

Introduction
Avascular necrosis of the lunate leading to abnormal carpal motion
Epidemiology
o

incidence
most common in males between 20-40 years old

risk factors
history of trauma

Pathophysiology
o

thought to be caused by multiple factors


biomechanical factors
ulnar negative variance
leads to increased radial-lunate contact stress
repetitive trauma

anatomic factors
geometry of lunate
vascular supply to lunate
patterns of arterial blood supply have differential
incidences of AVN
disruption of venous outflow

Prognosis
o

potentially debilitating condition if unrecognized and untreated

Anatomy
Blood supply to lunate
3 variations
o

Y-pattern

X-pattern

I-pattern
31% of patients
postulated to be at the highest risk for avascular necrosis

Classification
Lichtman Classification
Stage

Description

Treatment

Stage I

No visible changes on xray, changes Immobilization and NSAIDS


seen on MRI

Stage II

Sclerosis of lunate

Joint

leveling

procedure (ulnar

negative

patients)
Radial wedge osteotomy or STT fusion (ulnar
neutral patients)
Distal

radius

core

decompression

Revascularization procedures
Stage IIIA

Lunate collapse, no scaphoid rotation Same as Stage II above

Stage IIIB

Lunate

collapse,

fixed

scaphoid Proximal row carpectomy or STT fusion

rotation
Stage IV

Degenerated adjacent
joints

Stage 1

Stage 3a

intercarpal Wrist fusion, proximal row carpectomy, or


limited intercarpal fusion

Stage 2

Stage 3b

Stage 4
Presentation
Symptoms
o

dorsal wrist pain


usually activity related
more often in dominant hand

Physical exam
o

inspection and palpation


+/- wrist swelling
often tender over radiocarpal joint

range of motion
decreased flexion/extension arc
decreased grip strength

Imaging
Radiographs
o

recommended views
AP, lateral, oblique views of wrist

findings (see table above)

most useful once lunate collapse has already occurred

best for showing

CT

extent of necrosis
trabecular destruction

lunate geometry
MRI
o

best for diagnosing early disease

findings
decreased T1 signal intensity

reduced vascularity of lunate


Treatment
Nonoperative
o

immobilization, NSAIDS
indications
initial management for Stage 1 disease
outcomes
a majority of these patients will undergo further degeneration
and require operative management

Operative
o

temporary scaphotrapeziotrapezoidal pinning

indications
adolescent with radiographic evidence of Kienbock's and
progressive wrist pain
o

joint leveling procedure


indications
Stage I, II, IIIA disease with ulnar negative variance
technique
can be radial shortening osteotomy or ulnar lengthening

radial wedge osteotomy


indications
Stage I, II, IIIA disease with ulnar positive or neutral variance

vascularized bone grafts


indications
Stage I, II, IIIA disease
outcomes
early results promising, but long-term data lacking
greatest success in Stage II patients

distal radius core decompression


indications
Stage I, II, IIIA disease
technique
creates a local vascular healing response

STT fusion
indications
Stage II disease with ulnar neutral or positive variance
Stage IIIA or IIIB disease

must address internal collapse pattern (DISI)


o

proximal row carpectomy (PRC)


indications
stage IIIB disease
stage IV disease
outcomes
some studies have shown superior results of STT fusion over
PRC for stage IIIB disease

wrist fusion
indications
stage IV disease
technique
must remove arthritic part of joint

total wrist arthroplasty


indications
Stage IV disease
outcomes
long-term results not available

Techniques
Vascularized bone grafts
o

technique
many options have been described including
transfer of pisiform
transfer of distal radius on a vascularized pedicle of pronator
quadratus
transfers of braches of the first, second, or third dorsal
metacarpal arteries
4 + 5 extensor compartment artery (ECA)
temporary pinning of the STT joint, SC joint or external fixation may
be used to unload lunate after revascularization

Impact of surgical procedure on radiolunate contact stress

Operative Procedure

% decrease on radiolunate contact stress

STT fusion

3%

Scaphocapitate fusion

12%

Capitohamate fusion

0%

Ulnar lengthening of 4mm

45%

Radial shortening of 4mm

45%

Capitate shortening and capitohamate fusion

66%, but 26% increase in radioscaphoid load

Questions :
1. Figures A through E depict various conditions affecting the pediatric hand and wrist. For
which of the depicted conditions is temporary scaphotrapeziotrapezoidal pinning most
indicated?
FIGURES:

1.

A
4% (72/1698)

2.

B
33% (561/1698)

3.

C
4% (76/1698)

4.

D
51% (864/1698)

5.

E
7% (116/1698)
PREFERRED RESPONSE
Temporary scaphotrapeziotrapezoidal (STT) pinning is indicated for treatment of
Kienbocks disease in adolescents as shown in Figure D. The radiograph shows increased
density and slight lunate collapse. The result is a decrease in radiolunate contact stress while
increasing the load on the radioscaphoid articulation. STT pinning is not indicated in any of
the conditions explained below.
Ando et al retrospectively reviewed the results of six adolescents treated with
temporary scaphotrapezoidal (ST) pinning. All patients had an increase in wrist
flexion/extension arc, strength, and lunate intensity on MRI from their preoperative baseline.
Shigematsu et al published a case study on a single 11-year-old patient with wrist pain
at rest and with use who was treated with temporary scaphotrapeziotrapedoidal (STT) pinning
and cast immobilization for 8 weeks. Both wrist ROM and grip strength improved. Lunate
revascularization was also seen on subsequent MRI.
Incorrect Answers:
Answer 1,2,3: Radial clubhand, scaphoid fracture, and hypoplastic thumb are not treated with
temporary scaphotrapeziotrapezoidal pinning.

Answer 5: Gymnasts wrist is a distal radius physeal injury due to repetitive axial loading.
Plain films will show physeal widening and hazy irregularity. The condition is not treated
with temporary scaphotrapeziotrapezoidal pinning.
2. A 39-year-old male presents with longstanding right wrist pain. He has failed conservative
measures including prolonged immobilization. His radiographs and MRI are seen in
figures A and B. Which of the following options is an accepted treatment option?
FIGURES:

A
1.

Ulnar shortening osteotomy


5% (83/1648)

2.

TFCC repair
1% (19/1648)

3.

Radius core decompression


80% (1326/1648)

4.

Arthroscopic lunate chondroplasty and debridement


12% (200/1648)

5.

Scapholunate ligament reconstruction


1% (16/1648)

PREFERRED RESPONSE
The patient in the clinical scenario has Kienbock's disease. Treatment options include
a joint leveling procedure, or radius core decompression, which is thought to incite a local
vascular healing response in the lunate.
Sherman et al did a biomechanical study reviewing distal radius core decompression
for Kienbock's disease. Although the procedure has good clinical outcomes for this disease
process, their findings did not show any biomechanical explanation for these good outcomes.
Illarramendi et al reviewed results of curettage of the distal radius and ulna
metaphyseal bone through small cortical windows for the treatment of Kienbock's disease.
They concluded that the decompression procedure had good results without any
complications. Most patients had improvement in pain and were able to return to work.
Incorrect Answers:
Answer 1: Kienbock's disease is commonly associated with ulnar negative variance which is
thought to lead to increased forces on the lunate leading to this disease. Therefore a ulnar
shortening osteotomy would not be appropriate.
Answer 2,4,5: Are not treatment options for this disease process.
3. A 32-year-old carpenter complains of progressively worsening wrist pain for the last 2
months. He denies any recent history of trauma to the wrist or hand. An MRI is obtained
and a representative image is provided in Figure A. Which of the following surgical
interventions is thought to be effective for this condition by inciting a local vascular
healing response?
FIGURES:

A
1.

Wrist fusion

0% (7/1425)
2.

Ulnar shortening osteotomy


16% (223/1425)

3.

Distal radius core decompression


79% (1122/1425)

4.

Proximal row carpectomy


2% (23/1425)

5.

Scapholunate ligament reconstruction


3% (45/1425)
PREFERRED RESPONSE
This clinical scenario and imaging studies are consistent with Kienbock's disease,
avascular necrosis of the lunate, in the pre-collapse stage. Core decompression of the distal
radius is an accepted treatment for Kienbock's disease. The procedure creates a local vascular
healing response facilitating vascular recovery prior to collapse and degeneration of the
lunate. Other acceptable interventions include revascularization with a pedicled graft and
joint leveling procedures such as a radial shortening osteotomy. The radial shortening
osteotomy is ideal for patients with negative ulnar variance who experience greater loads
through the radiolunate fossa.
Sherman et al performed a cadaveric study demonstrating minimal change in the
distribution of force between the radiocarpal fossa and ulnocarpal fossa following core
decompression of the distal radius.
Illarramendi et al reviewed 22 cases of Kienbock's treated with radial and ulnar
metaphyseal core decompression. No surgical complications occurred, and 20 of 22 reported
satisfactory clinical outcomes while one patient developed intercarpal arthritis.
Incorrect Answers:
1. Proximal row carpectomy and wrist fusion would be options for the collapsed and
degenerative lunate.

2. Ulnar shortening osteotomy and scapholunate ligament reconstruction are incorrect as they
do not address the pathology of Kienbock's.
4. Proximal row carpectomy and wrist fusion would be options for the collapsed and
degenerative lunate.
5. Ulnar shortening osteotomy and scapholunate ligament reconstruction are incorrect as they
do not address the pathology of Kienbock's.
4. A 30-year-old female undergoes arthroscopy for a chronically painful right wrist that
failed to improve with 4 months of immobilization and NSAIDS. Her clinical examination
revealed point tenderness dorsally over the lunate but no tenderness elsewhere in the wrist.
A picture from the procedure is shown in Figure A where 'R' identifies the distal radius, 'L'
the lunate, and '*' represents a chondral flap. The articular surface of the lunate is stable to
probing. A radiograph and MRI image of the patients wrist are shown in Figures B and C
respectively. What is the most appropriate next step in treatment?
FIGURES:

1.

Continue Immobilization and NSAIDS


8% (183/2342)

2.

Radial shortening osteotomy


71% (1657/2342)

3.

Proximal row carpectomy


8% (198/2342)

4.

Scaphotrapeziotrapezoid fusion
9% (216/2342)

5.

Wrist fusion
3% (76/2342)
PREFERRED RESPONSE
The patients clinical presentation and radiographs are consistent with Stage 2
Kienbock's disease in the setting of negative ulnar variance. Radial shortening osteotomy is
the most appropriate treatment option listed for Stage 2 disease which is defined as lunate
sclerosis without significant collapse. Shortening osteotomy can alter DRUJ contact pressures
leading to remodeling, especially in the presence of a Tolat Type II DRUJ, such as that
shown in the radiographs. However, this remodeling has been shown to occur without the
development of arthritis, and therefore is not a contraindication to this procedure.
This patients radiographs shows some slight sclerosis of the lunate and negative ulnar
variance, and the MRI shows diffuse edema and early osteonecrosis of the lunate. The
arthroscopic image shows a cartilage flap with a stable base left on the lunate. Based on these
images, the patient has Stage 2 disease and should be treated with a joint leveling procedure;
or radial shortening osteotomy in this case.
Sltusky et al provide a review article which focuses on the methodology behind a
normal arthroscopic wrist examination and discusses some of the more standard arthroscopic
procedures along with the expected outcomes.
Bain et al review the arthroscopic staging of Kienbock's disease, and state that this
techinique is a valuable assessment tool which allows for not only classification of
Kienbock's disease, but also may guide treatment.
Schuind et al. provide a review of the pathogenesis of Kienbock's. They conclude that
the natural history of the condition is not well known, and the symptoms do not correlate well
with the changes in shape of the lunate and the degree of carpal collapse. They also state that
there is no strong evidence to support any particular form of treatment.

Illustration A shows a table which outlines the Stages of Kienbock's Disease.


Illustration B shows a table which outlines the general treatment options for each stage of
Kienbock's Disease.
Incorrect Answers:
Answer 1: Immobilization and NSAIDS is indicated in Stage I disease or as a first line of
treatment for Stage 2, which this patient has failed.
Answer 3: Proximal row carpectomy is indicated in Stage 3B.
Answer 4: STT Fusion is indicated in Stage 3B.
Answer 5: Wrist fusion is indicated in Stage 4.
Illustrations:

B
5. A 37-year-old man has a 2-year history of increasing right wrist pain that is worse at night
and aggravated by activity. He denies systemic symptoms, history of trauma, or recent
weight loss. On physical exam he has tenderness over the dorsal radiocarpal joint.
Radiographs of the right wrist are shown in Figure A. Which of the following imaging
studies would be most sensitive for determining the stage of this patient's underlying
condition?
FIGURES:

1.

Ultrasound
1% (24/2252)

2.

Angiography
8% (174/2252)

3.

CT scan of the wrist


63% (1425/2252)

4.

Clenched fist AP radiograph of wrist


8% (172/2252)

5.

Bone scan of the wrist


20% (448/2252)
PREFERRED RESPONSE
The clinical presentation of dorsal radiocarpal wrist pain is suggestive of Kienbocks
disease. Figure A shows an AP radiograph of the right wrist with evidence of lunate sclerosis
with no obvious collapse. The imaging study most sensitive for identifying early lunate
collapse in Kienbock's disease is CT scanning of the wrist.
Kienbocks disease is defined by avascular necrosis of the lunate. It is classified into 4
stages under the Lichtman Classification. In stage 1, plain radiographs appear normal and
magnetic resonance imaging is required for diagnosis. MRI is useful for detecting early
disease when sclerosis is not evident on plain film radiographs. In stage 2, plain radiographs
and/or CT scan images will show sclerosis of the lunate but no evidence of collapse. In stage
3, radiographs and/or CT scan images will show lunate collapse. For stage 4, radiographs
show degenerative changes to the adjacent carpus and intercarpal joints.
Imaeda et al. examined the use of MRI for the diagnosis and staging of Kienbock's
disease. They found that MRI was most sensitive in detecting early focal loss of signal

intensity in the lunate on T1-weighted images. This was a key diagnostic feature in early
stages of Kienbck's disease when plain radiographs appear normal.
Cross et al. reviewed the latest concepts for diagnosis, staging, and management of
Keinbock's disease. They suggest that computed tomography (CT) or tomography will better
characterize lunate necrosis and trabecular destruction once collapse or sclerosis has occurred
in late stage disease.
Illustration A is a collection of CT scanning images that show osteonecrosis of the
lunate. The blue arrow shows lunate flattening and sclerosis. The red double arrow shows a
loss of lunate height and the yellow shows fragmentation of the bone.
Incorrect Answers:
Answer 1: Ultrasound is not used in the staging of Kienbock's disease.
Answer 3: Angiography would not be warranted in this scenario.
Answer 4: A clenched fist AP radiograph of the wrist is used to evaluate widening of the
scapholunate interval.
Answer 5: A bone scan of the wrist is a non-specific test, which would likely be positive in
almost all patients with chronic wrist pain.
Illustrations:

Preisers Disease (Scaphoid AVN)


Author : David Abbasi

Introduction
A condition caused by AVN of scaphoid
Epidemiology
o

rare condition

average age of onset is 45 years

Presentation
Symptoms
dorsoradial wrist pain

Imaging
Radiographs
o

show sclerosis and fragmentation of proximal pole without evidence of


fracture

MRI
o

can further allow classification into complete vs partial involvement

Treatment
Nonoperative
o

immobilization
is effective in 20% of cases

Operative
o

microfracture

drilling,

revascularization

procedure,

or

allograft

replacement
indications
when nonoperative management fails
techniques include
drilling
revascularization
allograft replacements
o

proximal row carpectomy or scaphoid excision with four corner fusion


indications
considered salvage procedures

Gymnasts Wrist (Distal Radial Physeal Stress Syndrome)


Author : Tracy Jones

Introduction
Overuse syndrome of the wrist primarily affecting young gymnasts
o

may lead to premature closure of distal radial physis

Epidemiology
o

up to 25% of non-elite gymnasts

Pathophysiology
o

wrist undergoes supraphysiological loads due to use as a weight bearing joint

repetitive stress causes inflammation at growth plate of distal radius

microtrauma can lead to premature closure of distal radial physis resulting in


secondary overgrowth of ulna

Associated conditions
o

orthopaedic
distal ulnar overgrowth
positive ulnar variance

Prognosis
o

good outcomes associated with early treatment

Presentation
Symptoms
o

wrist pain
usually radial sided
may be chronic in nature

Physical exam
o

inspection

swelling may be present at wrist


tenderness to palpation at distal radius
o

motion
decreased wrist flexion or extension may be present

Imaging
Radiographs
o

recommended views
AP and lateral of the wrist

findings
widened distal radial growth plate with ill-defined borders

positive ulnar variance with chronic cases

MRI
o

indications
chronic or cases non-responsive to treatment

findings
paraphyseal edema

early physeal bridging


bruising of radius

Treatment
Nonoperative
o

NSAIDS, rest, immobilization for 3-6 months


indications
first line of treatment

Operative
o

resection of physeal bridge


indications
small physeal closures

ulnar epiphysiodesis and shortening with radial osteotomy as needed


indications
large physeal closures (roughly 50% of physis)

SNAC (Scaphoid Nonunion Advance Collapse)


Author : Orthobullet Team

Intronduction
A condition characterized by advanced collapse and progressive arthritis of the wrist
that results from a chronic scaphoid nonunion
o

see scaphoid fracture

Pathophysiology
o

pathoanatomy
natural history of degenerative changes first occurs at the
radioscaphoid area followed by pancarpal / midcarpal arthritis

Prognosis
o

patients with scaphoid nonunions of > 5 years duration or proximal pole


necrosis have less favorable outcomes

punctate bleeding of bone during surgery is a good prognostic indicator of


union
92% union with obvious bleeding, 71% with questionable bleeding,
0% with no bleeding
results show decreased rate of arthritis (down to 40-50%)

Anatomy
Scaphoid anatomy
blood supply
o

major blood supply is dorsal carpal branch (branch of the radial artery)
enters scaphoid in a nonarticular ridge on the dorsal surface and
supplies proximal 80% of scaphoid via retrograde blood flow

minor blood supply from superficial palmar arch (branch of volar radial
artery)
enters distal tubercle and supplies distal 20% of scaphoid

motion
o

both intrinsic and extrinsic ligaments attach and surround the scaphoid

the scaphoid flexes with wrist flexion and radial deviation and it extends
during wrist extension and ulnar deviation (same as proximal row)

also see Wrist Ligaments and Biomechanics for more detail


Classification
Radiographic Classification
Stage I

Arthrosis localized to the radial side of the scaphoid and radial styloid

Stage II

Scaphocapitate arthrosis in addition to Stage 1

Stage III

Periscaphoid arthrosis (proximal lunate and capitate may be maintained)

Presentation
Symptoms
o

weakness
reduced grip and pinch strength

stiffness
stiffness with extension and radial deviation

Physical exam
o

palpation
localized tenderness of the radioscaphoid articulation

motion
decreased wrist motion on extension and radial deviation

Imaging
Radiographs
recommended view
o

ap and lateral of wrist

findings

see radiographic classification above

Treatment
Nonoperative
o

observation alone
indications
medically frail and low functioning patients only

Operative
o

radial styloidectomy plus scapholunate reduction and stabilization


indications
stage I

proximal row corpectomy


indications
stage II and III
outcomes
disadvantages
reduction of wrist motion and grip strength
procedure should be avoided if there are capitate head
degenerative changes

four-corner fusion
indications
stage II and III
outcomes
retains 60% of wrist motion and 80% of grip strength

wrist arthrodesis
indications
stage II and III

Questions :
1. A 30-year-old female reports 5 months of wrist pain after a fall onto her wrist. A
radiograph is shown in Figure A. If untreated, which of the following is least likely to
occur during the natural progression of the disease process?

FIGURES:

1.

Radial styloid osteophyte


6% (136/2122)

2.

Radioscaphoid arthritis
7% (150/2122)

3.

Midcarpal arthritis
8% (178/2122)

4.

Scapholunate arthritis
19% (402/2122)

5.

Radiolunate arthritis
59% (1248/2122)
PREFERRED RESPONSE
Radiographs show a scaphoid non-union which can lead to Scaphoid Nonunion
Advanced Collapse (SNAC wrist) and progressive arthritis. The natural history of

degenerative changes first occurs at the radioscaphoid area and progresses to pancarpal
arthritis. All of the answers above are features of a SNAC wrist with radiolunate arthritis only
occurring at the very end stages of disease.
In the cited reference by Schuind et al, they found that professional heavy work, age
of the nonunion of over 5 years, associated radial styloidectomy, and duration of
postoperative immobilization were associated with a significantly decreased likelihood of
healing of the scaphoid nonunion with operative treatment.
The study by Soejima et al found that distal scaphoid resection produces a satisfactory
clinical outcome and should be considered one of the surgical options for patients with longstanding scaphoid nonunion with either radioscaphoid or intercarpal degenerative arthritis.

Scapholunate Ligament Injury & DISI


Author : Evan Watts

Introduction
Scapholunate ligament is important for carpal stability
o

chronic scapholunate deficiency leads to DISI (see below)

Epidemiology
o

incidence
acute injury
occurs in approximately 10-30% of intra-articular distal radius
fractures or carpal fractures
degenerative injury
degenerative tears in >50% of people over the age of 80 years
old

location
ligament has 3 components that span between the scaphoid and lunate
bones
dorsal, proximal and volar components
incomplete tears > complete tears

Pathophysiology
o

mechanism of injury
sudden impact force applied to the hand and wrist causing SLIL injury
and scapholunate dissociation
injury occurs most commonly with wrist positioned in extension, ulnar
deviation and carpal supination

pathoanatomy

osseous
SLIL tearing will position the scaphoid in flexion and lunate
extension
ligamentous
diastasis of the scapholunate complex occurs with complete
SLIL tears and capsule disruption.
Associated injuries
o

DISI

(dorsal

intercalated

segmental

instability)

scapholunate dissociation causes the scaphoid to flex palmar and the


lunate to dorsiflex
if left untreated the DISI deformity can progress into a SLAC wrist
DISI is a form of carpal instability dissociative

Anatomy
Scapholunate interosseous ligament
o

location
c-shaped structure connecting the dorsal, proximal and volar surfaces
of the scaphoid and lunate bones
dorsal fiber thickened (2-3mm) compared to volar fibers

biomechanics

dorsal component provides the greatest constraint to


translation between the scaphoid and lunate bones
proximal fibers have minimal mechanical strength
Overview of wrist ligaments and biomechanics

Presentation
History
o

acute FOOSH injury vs. degenerative rupture


age, nature of injury, duration since injury, degree of underlying
arthritis, level of activity

Symptoms
o

usually dorsal and radial-sided wrist pain

pain increased with loading across the wrist (e.g. push up position)

clicking or catching in the wrist

may be associated with wrist instability or weakness

Physical exam
o

inspection
may see swelling over the dorsal aspect of the wrist

palpation
tenderness in the anatomical snuffbox or over the dorsal scapholunate
interval (just distal to Lister's tubercle)

motion
pain increased with extreme wrist extension and radial deviation

provocative tests
Watson test
when deviating from ulnar to radial, pressure over volar aspect
of scaphoid produces a clunk secondary to dorsal subluxation
of the scaphoid over the dorsal rim of the radius
dorsal wrist pain or a clunk during this maneuver may
indicate instability of scapholunate ligament

Imaging
Radiographs

recommended views
AP and lateral views of the wrist

additional views
radial and ulnar deviation views
flexion and extension views
clenched fist (can attenuate the diastasis)

findings
AP radiographs
SL gap > 3mm with clenched fist view (Terry Thomas sign)

cortical ring sign (caused by scaphoid malalignment)

humpback deformity with DISI associated with an unstable


scaphoid fracture
scaphoid shortening
Lateral radiographs
dorsal tilt of lunate leads to SL angle > 70 on neutral rotation
lateral

capitolunate angle > 20


DISI
normal carpal alignment

increased SL angle

Arthrography
o

indications
may be used as screening tool for arthroscopy

views
radiocarpal and midcarpal views
always assess the contralateral wrist for comparison

findings
may demonstrate the presence of a tear but cannot determine the size
of the tear
positive finding of a tear may indicate the need for wrist arthroscopy

MRI
o

indications
often overused as a screening modality for SLIL tears

findings
requires careful inspection of the SLIL by a dedicated radiologist to
confirm diagnosis
low sensitivity for tears

Arthroscopy
o

indications
considered the gold standard for diagnosis

Treatment
Nonoperative
o

NSAIDS, rest +/- immobilization


indications
acute, undisplaced SLIL injuries
chronic, asymptomatic tears
technique
splinting and close follow-up with repeat imaging and clinical
response with acute injuries
outcomes
most people feel casting alone is insufficient
may be effective with incomplete tears

Operative
o

scapholunate ligament repair or SL reconstruction (if repair not possible)


indications

acute scapholunate

ligament

injury

without

carpal

malalignment
chronic but reducible scapholunate ligament injuries
primary repair can be performed up to 18 months from the time
of injury
techniques
primary repair
SL joint pinning with k-wires
suture anchors with k-wires
Blatt dorsal capsulodesis
often added to a ligament repair and remains a
viable alternative for a chronic instability when
ligament repair is not feasible
tendon reconstruction
FCR tendon transfer (direct SL joint reduction)
ECRB tendonosis (indirect SL joint reduction)
weave not recommended due to high incidence of late
failure
if pathoanatomy is scaphoid fx than ORIF vs. CRPP (+/arthroscopic assistance)
o

stabilization with wrist fusion (STT or SLC)


indications
rigid and unreducible DISI deformity
DISI with severe DJD
technique
scaphotrapezialtrapezoidal (STT) fusion
scapholunocapitate (SLC) fusion
scapholunate fusion alone has highest nonunion rate

Techniques
Direct repair SLIL with k-wires
approach
o

small incision is made just distal to the radial styloid

care to avoid cutting the radial sensory nerve branches

technique
o

place two k-wires in parallel into the scaphoid bone

reduce the SL joint by levering the scaphoid into extension, supination and
ulnar deviation and lunate into flexion and radial deviation

pass the k wires into the lunate

confirm reduction of the SL joint under fluoroscopy

place patient in short arm cast

post-operative care
o

remove k-wires in 8-10 weeks

no heavy labor for 4-6 months

Questions :
1. A 32-year-old professional baseball player presents with wrist pain after a fall on his
outstretched wrist 10 days ago. He initially thought it was a sprain, but presents due to
continued pain worsened by push-ups. His physical exam shows dorsal wrist tenderness
and is positive for the provocative test shown in Figure V. Standard PA radiograph of the
wrist is normal. Which of the following radiographic views shown in Figures A to E
would be most helpful in establishing the diagnosis?
FIGURES:

1.

A
77% (877/1146)

2.

B
8% (97/1146)

3.

C
8% (92/1146)

4.

D
4% (47/1146)

5.

E
2% (24/1146)
PREFERRED RESPONSE
The clinical description and video of the patient's physical exam are consistent with
an acute scapho-lunate ligament tear. The video shown in the question stem demonstrates the
Watson test. When positive, the patient will feel dorsal wrist pain and/or a "clunk" when the
wrist is brought from extension/ulnar deviation to radial deviation. If plain radiographs are
normal, a PA clenched fist radiograph as seen in Figure A should be performed.

In patients with a acute scapho-lunate ligament tear, initial radiographs may not show
the characteristic "Terry Thomas" sign, or widening of the SL gap > 3mm. When making a
clenched fist, the capitate is drawn proximally, stressing the SL ligament. This is an easy
view to obtain during the initial patient visit and should strongly be considered if this
diagnosis is suspected.
Walsh et al review the various aspects of scapholunate ligament injuries. While they
agree imaging is helpful in establishing the diagnosis, they emphasize that wrist arthroscopy
is the gold standard in the diagnosis of SL injuries.
Illustration A shows demonstrates a clenched fist view with obvious widening of the
scapho-lunate gap.
Incorrect Answers:
Answer 2: Shows a lateral radiograph in 30 degrees of supination. It is excellent for
assessment of pisotriquetral arthrosis.
Answer 3: Shows a PA of the wrist in radial deviation. This view will actually close the SL
gap.
Answer 4: Shows a a carpal tunnel view, used for assessment of hook of hamate fractures.
Answer 5: Shows a a stardard PA wrist in neutral aligment.
Illustrations:

Lunotriquetral Ligament Injury & VISI


Author : Mark Vitale

Introduction
Instability of the lunotriquetral joint caused by rupture of the
o

lunotriquetral ligament and

dorsal radiocarpal ligament (aka radiotriquetral ligament)

Epidemiology
o

LT ligament injury is less common than SL ligament injury

Mechanism
o

LT ligament injury occurs with


wrist hyperextension or
extension and radial deviation

scaphoid induces the lunate into further flexion while triquetrum extends

VISI Deformity
o

stands for volar intercalated segment instability


a type of Carpal Instability Dissociative (CID)

caused by advanced injury with injury to


lunotriquetral ligament
dorsal radiotriquetral ligament
volar radiolunate ligament

VISI may occasionally be seen in uninjured wrists in patients with


ligamentous laxity
this is in contrast to DISI deformity, which is always a pathologic
condition

Anatomy
Lunotriquetral ligament
o

C-shaped intrinsic ligament spanning the dorsal, proximal and palmar edges of
the joint

comprised of thick dorsal and volar regions and weak membranous portion

dorsal LT ligament
most important as a rotational constraint
volar LT ligament

thickest and strongest portion of the LT ligament


transmits extension moment of the triquetrum
Dorsal radiocarpal ligament (aka dorsal radiotriquetral ligament)

extrinsic ligament that serves as a secondary restraint to VISI deformity, and


loss of integrity allows lunate to flex more easily

Volar long and short radiolunate ligaments

extrinsic ligament that may be torn in advanced injury

Presentation
Symptoms
o

ulnar sides pain that is worse with pronation and ulnar deviation (power grip)

Physical exam

LT shuck test (aka ballottement test)


grasp the lunate between the thumb and index finger of one hand while
applying alternative dorsal and palmar loads across the triquetrum with
the thumb and index of the other hand
positive test elicits pain, crepitus or increased laxity, suggesting LT
interosseous injury

Kleinman's shear test


stabilize the radiolunate joint with the forearm in neutral rotation and
with the contralateral hand load the triquetrum in the AP plane,
producing shear across the LT joint
positive test produces pain or a clunk

Lunotriquetral compression test


displacement of triquetrum ulnarly during radioulnar deviation which
is associated with pain

Imaging
Radiographs
o

lateral
volar flexion of lunate leads to SL angle < 30 (normal is 47)
and VISI deformity
capitolunate zigzag deformity seen with capitolunate angle increase to
> 15 (lunate and capitate normally co-linear)

AP
unlike scapholunate dissociation, may not be widening of LT interval
break in Gilula's arc
may see proximal translation of triquetrum and/or LT overlap

Arthroscopy
o

helpful in making diagnosis, as radiographs may be normal

Treatment
Nonoperative
o

observation
indications
may be attempted initially

Operative
o

CRPP (multiple K-wire fixation) with acute ligament repair +/- dorsal
capsulodesis
indications
acute instability
technique
ligament reconstructions with bone-ligament-bone autograft
and LT fusion have fallen out of favor in acute setting

LT fusion
indications
chronic instability
complications
nonunion is a known complication

arthroscopic debridement of LT ligament with ulnar shortening


indications
chronic instability secondary to ulnar positive variance
long ulna chronically impacts the triquetrum, resulting
in LT tear with instability
often

associated

with

degenerative

fibrocartilage complex (TFCC)

tear

of

triangular

SLAC (Scaphoid Lunate Advanced Collapse)


Author : Mark Vitale

Introduction
A condition of progressive instability causing advanced arthritis of radiocarpal and
midcarpal joints
o

describes the specific pattern of degenerative arthritis seen in chronic


dissociation between the scaphoid and lunate

Pathoanatomy
o

chronic SL ligament injury creates a DISI deformity


scaphoid is flexed and lunate is extended as scapholunate ligament no
longer restrains this articulation
scapholunate angle > 70 degrees
lunate extended > 10 degrees past neutral

resultant scaphoid flexion and lunate extension creates


abnormal distribution of forces across midcarpal and radiocarpal joints
malalignment of concentric joint surfaces

initially affects the radioscaphoid joint and progresses to capitolunate joint

Classification
Watson classification
o

describes predictable progression of degenerative changes from the radial


styloid to the entire scaphoid facet and finally to the unstable capitolunate
joint, as the capitate subluxates dorsally on the lunate

key finding is that the radiolunate joint is spared, unlike other forms of wrist
arthritis, since there remains a concentric articulation between the lunate and
the spheroid lunate fossa of the distal radius

Watson Stages
Stage I

Arthritis between scaphoid and radial styloid

Stage II

Arthritis between scaphoid and entire scaphoid facet of the radius

Stage III

Arthritis between capitate and lunate

note: radiolunate joint spared

Stage 1

Stage 2

Stage 3

While original Watson classification describes preservation of radiolunate joint in all


stages of SLAC wrist, subsequent description by other surgeons of "stage IV"
pancarpal arthritis observed in rare cases where radiolunate joint is affected
o

validity of "stage IV" changes in SLAC wrist remains controversial and


presence pancarpal arthritis should alert the clinician of a different etiology of
wrist arthritis

Presentation
Symptoms
o

difficulty bearing weight across wrist

patients localize pain in region of scapholunate interval

progressive weakness of affected hand

wrist stiffness

Physical exam
o

tenderness directly over scapholunate ligament dorsally

decreased wrist ROM

weakness of grip strength

Watson scaphoid shift test

patients may have positive Watson scaphoid shift test early in the
process,
will not be positive in more advanced cases as arthritic changes
stabilize the scaphoid
technique
with firm pressure over the palmar tuberosity of the scaphoid,
wrist is moved from ulnar to radial deviation
positive test seen in patients with scapholunate ligament injury
or patients with ligamentous laxity, where the scaphoid is no
longer constrained proximally and subluxates out of the
scaphoid fossa resulting in pain
when pressure removed from the scaphoid, the scaphoid
relocates back into the scaphoid fossa, and typical snapping or
clicking occurs
must compare to contralateral side

Evaluation
Radiographs
o

obtain standard PA and lateral radiographs


PA radiograph will reveal greater than 3mm diastasis between the
scaphoid and lunate
Stage I SLAC wrist
PA radiograph shows radial styloid beaking, sclerosis
and joint space narrowing between scaphoid and radial
styloid
Stage II SLAC wrist
PA radiograph shows sclerosis and joint space
narrowing between scaphoid and the entire scaphoid
fossa of distal radius
Stage III SLAC wrist
PA radiograph shows sclerosis and joint space
narrowing between the lunate and capitate, and the
capitate will eventually migrate proximally into the
space created by the scapholunate dissociation

lateral radiograph
will reveal DISI deformity and subluxation of capitate dorsally
onto lunate
o

stress radiographs unnecessary

MRI
o

unnecessary for staging, but will show


thinning of articular surfaces of the proximal scaphoid
scaphoid facet of distal radius and capitatolunate joint with synovitis in
radiocarpal and midcarpal joints

Treatment
Nonoperative
o

NSAIDs, wrist splinting, and possible corticosteroid injections


indications
mild disease

Operative
o

radial styloidectomy and scaphoid stabilization


indications
Stage I
technique
prevents impingement between proximal scaphoid and radial
styloid
may be performed open or arthroscopically via 1,2 portal for
instrumentation

PIN and AIN denervation


indications
Stage I
technique
since posterior and anterior interosseous nerve only provide
proprioception and sensation to wrist capsule at their most
distal branches, they can be safely dennervated to provide pain
relief
can be used in combination with below procedures for Stage II
or III

proximal row carpectomy


indications
Stage II
contraindicated if there is an incompetent radioscaphocapitate
ligament
contraindicated

with

caputolunate

arthritis

(Stage

III)

because capitate articulates with lunate fossa of the distal radius


technique
excising entire proximal row of carpal bones (scaphoid, lunate
and

triquetrum)

while

preserving radioscaphocapitate

ligament (to prevent ulnar subluxation after proximal row


carpectomy)
outcomes
provides relative preservation of strength and motion
o

scaphoid excision and four corner fusion


indications
Stage II or III
technique
also provides relative preservation of strength and motion
wrist motion occurs through the preserved articulation between
lunate and distal radius (lunate fossa)
outcomes
similar long term clinical results between
scaphoid excision/ four corner fusion and
proximal row carpectomy

wrist fusion
indications
Stage III
any form of pancarpal arthritis
outcomes
wrist fusion gives best pain relief and good grip strength at the
cost of wrist motion

Questions :
1. A 65-year-old man fell and injured his right wrist. Radiographs taken in the emergency
room are seen in Figure A. He was treated as a sprain and no further follow-up was
planned. He sustained 2 minor falls over the next 6 years and his wrist pain recurred.
Recent radiographs are seen in Figure B. Surgical treatment that will best address his
symptoms and preserve wrist motion consists of
FIGURES:

A
1.

Anterior and posterior interosseous neurectomy


3% (75/2709)

2.

Scaphotrapezialtrapezoidal (STT) fusion


10% (281/2709)

3.

Complete wrist arthrodesis


1% (36/2709)

4.

Proximal row carpectomy


34% (914/2709)

5.

Four-corner fusion with scaphoidectomy


51% (1393/2709)

PREFERRED RESPONSE
Four-corner fusion with scaphoidectomy is indicated for Stage III SLAC
wrist. Surgical treatment of SLAC wrist is stage dependent. Stage I disease (scaphoid-radial
styloid arthritis) is treated with AIN/PIN neurectomy. This procedure can also be done in
addition to other bony procedures for Stages II-III disease. Stage II (scaphoid-entire scaphoid
facet) is treated with PRC or scaphoid excision with 4-corner fusion (4CF). Stage III
(capitolunate arthritis with proximal migration of the capitate into the scapholunate interval)
is treated with either scaphoidectomy with 4CF or total wrist fusion.
Some other conditions exist: If capitolunate arthritis exists, PRC is contraindicated
and 4CF is performed. If radiolunate arthritis exists, both PRC and 4CF are contraindicated
and total wrist fusion is performed. If both radiolunate and capitolunate surfaces are
preserved, then either PRC or a 4CF may be performed.
Cohen et al. compare PRC with 4-corner fusion plus scaphoid excision. PRC is
technically easier, but leads to shortening of the carpus with weakness and incongruity exists
between the capitate and lunate fossa of the distal radius. Scaphoid excision and four-corner
fusion maintains carpal height and preserves the radiolunate relationship, but is more
technically demanding, there is risk of nonunion, and it requires longer postop
immobilization. Pain relief is more reliable following 4-corner fusion.
Figure A shows scapholunate ligament disruption. Figure B shows late stage SLAC
wrist. There is capitolunate arthritis but no radiolunate arthritis. Illustration A shows an
example of PRC. Illustration B shows an example of 4CF and scaphoidectomy.
Incorrect Answers:
Answer 1. Neurectomy of AIN and PIN is performed for Stage I disease and can also be done
in addition to other bony procedures for Stages II-III.
Answer 2. STT fusion is indicated for chronic scapholunate instability, STT arthritis and
Kienbock's disease. It is not appropriate for Stage III SLAC wrist as it does not address
capitolunate arthritis.
Answer 3. Complete wrist arthrodesis is indicated for pancarpal arthritis in a young patient. It
is less appropriate for this 71-year-old patient. It sacrifices wrist motion. Wrist arthrodesis
would be performed if BOTH capitolunate and radiolunate arthritis were present
Answer 4. Proximal row carpectomy is indicated for Stage II disease. It is contraindicated
where capitolunate arthritis is present (Stage III).

Illustrations:

2. A 45-year-old male sustained a fall onto his right wrist 2 weeks ago. A radiograph is
shown in figure A. What joint is first affected if left untreated with subsequent
development of a SLAC (scapholunate advanced collapse) wrist?
FIGURES:

1.

Capitolunate joint
7% (30/456)

2.

Radioscaphoid
76% (347/456)

3.

Radioulnar

1% (4/456)
4.

Radiolunate
12% (53/456)

5.

STT (scaphotrapezotrapezoidal)
4% (18/456)
PREFERRED RESPONSE
The clinical presentation is consistent with a SLAC wrist. The radioscaphoid joint is
the first to be affected in this process.
The radiographs of the right wrist demonstrate a scapholunate dissociation, as
evidenced by an increased scapholunate joint space, referred to as scapholunate diastasis
(abnormal when the gap is greater than 2 mm and increased from the opposite extremity and
other intercarpal spaces).
If left untreated, the wrist may progress to a "SLAC" wrist, as originally described by
Watson and Ballet in 1984, which is the most common form of wrist arthritis. The repetitive
sequence of degenerative changes is based on and caused by articular alignment problems
between the scaphoid, the lunate and the radius.
Kuo et al. review the stages of SLAC wrist. They report stage I SLAC wrist involves
changes limited to an area of abnormal contact between the abnormally rotated scaphoid and
the radial styloid. In stage II the remaining radioscaphoid joint is affected, as persistent
abnormal load transfer and shear across the cartilaginous surfaces leads to degeneration of the
proximal scaphoid facet. In stage III, the dorsally translated capitate migrates proximally into
the widened scapholunate interval, and degenerative changes occur at the capitolunate joint.
The relative congruency of the radiolunate joint in all positions of lunate rotation due to the
spherical shape of the lunate facet preserves this articulation, and at all stages of SLAC wrist
the radiolunate joint is not involved. The lunate is congruently loaded in every position and,
thus, highly resistant to degenerative changes.
Illustration A below shows the stages of involvement in the SLAC wrist.

Illustrations:

Basilar Thumb Arthritis

Introduction
-metacarpal (CMC) joint
common arthritis of the hand
o 2nd only to DIP arthritis

theorized to be due to attenuation of anterior oblique ligament (Beak ligament)


o leading to instability, subluxation, and arthritis of CMC joint
Anatomy

Trapezial metacarpal joint is a biconcave saddle joint


Trapezium has a palmar groove for flexor carpi radialis (FCR) tendon
Ligaments

anterior oblique ligament (Beak ligament)


primary stabilizing restraint to subluxation of CMC joint
o intermetacarpal ligaments
o posterior oblique ligament
o dorsal-radial capsule (injured in dorsal CMC dislocation)
Biomechanics
o CMC joint reactive force is 13X applied pinch force
o

Classification

Stage I
Stage II
Stage III
Stage IV

Eaton and Littler Classification of Basilar Thumb Arthritis


slight joint space widening (pre-arthritis)
slight narrowing of CMC joint with sclerosis, osteophytes <2mm
marked narrowing of CMC joint with osteophytes, osteophytes >2mm
pantrapezial arthritis (STT involved)

Stage 1

stage 2

Stage 3

Presentation
Symptoms
o pain at base of thumb
o difficulty pinching and grasping
o concomitant carpal tunnel syndrome

Stage 4

up to 50% incidence
Physical exam
o painful CMC grind test
combined axial compression and circumduction
o swelling and crepitus
o metacarpal adduction and web space contractures
are later findings
o may have adjacent MCP fixed hyperextension
during pinch
Imaging
Radiographs
o technique
X-ray beam is centered on trapezium and metacarpal with thumb flat
on cassette and thumb hyperpronated

findings
joint space narrowing
osteophytes
may show MCP hyperextension

Treatment
Nonoperatie
o NSAIDS, bracing, symptomatic treatment
indications
indicated as first line of treatment for mild symptoms
technique
splints (thumb spica orthosis)
o hyalgan injections
show no difference for the relief of pain and improvement in function
when compared to placebo and corticosteroids
Operative
o trapezial resection with LRTI (ligament reconstruction and tendon
interposition)
indications
Stage II-IV disease
most common procedure and favored in most patients
technique
there are many different surgical options available

trapezial excision appears to be the most important step


to this procedure, regardless of the other specifics of
each individual style of CMC arthroplasty
FCR tendon most commonly used in reconstruction to
suspend metacarpal
alternatively, APL may be used for suspension
or PL around FCR to correct subluxation

outcomes
can expect 25% subsidence postoperatively
with no change in outcomes
results in improved grip and pinch strengths
ligament reconstruction with FCR
indications
Stage I disease when joint is hypermobile and unstable (pain
with varus valgus stress)
CMC arthroscopy and debridement
indications
early stages of disease
extension osteotomy of the first metacarpal
indications
early Stage disease and minimal arthritic degeneration of CMC
joint
technique
redirects the force to the dorsal, more uninvolved portion of the
first carpometacarpal joint
outcomes
has gained in popularity and studies show that 93% are
improved at seven years out
trapeziometacarpal arthrodesis and fusion

indications
Stage II and Stage III disease in young male heavy laborers
technique
TM joint fused in
35 radial abduction
30 palmar abduction
15 pronation
outcomes
good pain relief, stability, and length preservation
decreased ROM; inability to put hand down flat
nonunion rate of 12%
volar capsulodesis, EPB tendon transfer, sesamoid fusion, or MCP fusion
indications

if thumb MCP hyperextension instability (joint can


hyperextend > 30)
otherwise a Swan neck deformity will arise
silicone replacements
indications
not recommended due to complications of prosthesis fracture,
subluxation, or silicone synovitis

1. (OBQ13.95) A 55-year-old female patient presents with pain along the thumb ray and
increasing deformity of her right hand. Key pinch causes her pain. The appearance of her
hand is seen in Figure A. Range of motion of her thumb is seen in Figure B. What is the most
likely cause of her deformity?

1.

Type II hypoplastic thumb

1% (13/1955)
2.

Median nerve neuropathy

4% (69/1955)
3.

Lupus thumb deformity

4% (75/1955)
4.

Extensor tendon rupture

5% (103/1955)
5.

Osteoarthritis of the trapeziometacarpal joint

86% (1686/1955)
PREFERRED RESPONSE 5
The patient has 1st carpometacarpal (CMC) arthritis.
With 1st CMC arthritis, the patient avoids painful thumb abduction and an adduction
deformity gradually develops, with 1st webspace contracture. With progressive 1st CMC
stiffness, the thumb metacarpophalangeal joint (MCP) develops hyperextension deformity to
compensate for the loss of motion, leading to a secondary "Z" deformity.
Rozental et al. reviewed hand and wrist reconstruction. They believe that arthrosis arises
from loss of the anterior oblique ("beak") ligament. Compensatory MCP hyperextension
should
be
treated
with
MCP
capsulodesis
or
arthrodesis.
Van Heest et al. reviewed thumb CMC arthritis. Treatment for Eaton stage I/II arthritis is
open/arthroscopic debridement, volar ligament reconstruction (with APL or FCR tendons), or
metacarpal extension osteotomy. For stage III/IV arthritis, treatment options include implant
arthroplasty or resection arthroplasty +/- LRTI (with APL, FCR or palmaris longus), and
fusion (young patients).
Figure A shows adduction contracture of the 1st webspace, with hyperextension deformity of
the 1st MCP joint. Figure B illustrates decreased thumb abduction because of adduction
contracture with decreased palmar abduction (normal, 45deg) and decreased radial abduction
(normal, 60deg). Illustration A is a radiograph showing thumb CMC arthritis with Z

deformity. Illustration B shows lupus thumb deformity ("hitchhiker thumb"). Illustration C


shows hand changes in inflammatory arthritis.
Incorrect Answers:
Answer 1: Type II thumb hypoplasia does not present with "Z" deformity.
Answer 2: Chronic median nerve neuropathy leads to flattening of the thenar eminence and
an ape hand deformity from loss of opponens pollicis. There is no loss of thenar bulk or
thumb opposition in Figure A.
Answer 3: There is no lupus thumb deformity (Illustration A). Lupus thumb deformity is
characterized by flexion at the MCP and hyperextension at the interphalangeal joint.
Answer 4: Extensor tendon rupture will not lead to hyperextension deformity of the MCP
joint.

2. (OBQ11.246) A 68-year-old female office assistant reports left thumb pain that has
progressively worsened over the past 2 years. She is left hand dominant and reports difficulty
with opening jars and holding a coffee cup. On examination of the left hand she has a positive
thumb carpometacarpal grind test and has a fixed deformity at the thumb
metacarpalphalangeal joint. Figure A demonstrates the left hand grasping an object and
Figure B shows a radiograph of the left thumb. What is the most appropriate next step in
treatment?

PREFERRED RESPONSE 3
The patients history, examination, and images are consistent with thumb CMC (basilar) joint
arthritis with associated MCP joint arthritis. At the MCP joint there is hyperextension of the
thumb metacarpophalangeal (MCP) joint and adduction involving the first web space of the

hand (Z deformity). Arthrodesis of the MCP joint is the treatment of choice when thumb
MCP hyperextension exceeds 40, the deformity is not passively correctable, or advanced
degenerative changes are noted to affect the articulation.
The review article by Armbruster and Tan state that when MCP joint hyperextension is:
0 to 10= Surgical intervention is not necessary when MCP hyperextension is less than 10.
10 to 20= Percutaneous pinning of the MCP joint in 25 to 35 of flexion for 3-4 weeks
may be performed independently or as an adjunct to EPB transfer.
20 to 40= Capsulodesis of the volar aspect of the MCP joint is recommened to provide a
check rein for hyperextension and Sesamoidesis has also been investigated as an adjunctive
procedure.
Cooney et al performed a Level 4 review of their CMC arthroplasty patients and found 15
patients with 17 revision arthroplasties in the treatment of mechanical pain related to
instability or bone impingement. The revisions included soft-tissue interposition alone or
soft-tissue interposition with ligament reconstruction and found that this provided satisfactory
patient outcomes in more than 75% of the cases.
Illustration A depicts the forces accounting for the observed adduction and hyperextension
deformities. The arrowhead indicates the direction of subluxation of the base of the thumb
metacarpal (due to incompetent volar beak ligament). The arrow represents the force vector
of the EPB potentiating the MCP hyperextension deformity

3. (OBQ09.122) A 60-year-old man has chronic pain at the base of this thumb and weakness
on attempted thumb pinch. A radiograph is shown in Figure A. Which injection would likely
reduce his pain and increase his function?

1.

Saline

1% (8/1501)

2.

Steroid

40% (607/1501)
3.

Hylan

1% (20/1501)
4.

All of the above are equally effective

55% (832/1501)
5.

All of the above are detrimental

2% (28/1501)
PREFERRED RESPONSE 4
The patient has basal joint arthritis of the thumb and randomized controlled trials have failed
to
demonstrate
an
advantage
of
steroid
or
hylan
over
saline.
Heyworth et al demonstrated that all three injections were similarly effective for
approximately 3 months at reducing pain and increasing thumb function over baseline levels.
Stahl found that steroid and hylan were equally effective, but did not control with saline.
Henderson found no advantage of hyaluronan over saline for knee arthritis during a 5 week
treatment course.

DIP and PIP Joint Arthritis


Introduction

Forms include
o

primary osteoarthritis
DIP
highest joint forces in hand
undergoes more wear and tear
associated with Heberden's nodules (caused by
osteophytes)
mucous cysts
can lead to draining sinus
septic arthritis
nail ridging
nail can be involved
splitting
deformity
loss of gloss
PIP
Bouchard nodes
joint contractures with fibrosis of ligaments
erosive osteoarthritis
condition is self limiting, patients are relatively asymptomatic,
but can be destructive to joint
more common in DIP
seen in middle aged women with a 10:1 female to male ratio

Presentation
Symptoms of primary osteoarthritis
o pain

deformity
Symptoms of erosive osteoarthritis
o intermittent inflammatory episodes
o articular cartilage and adjacent bone destroyed
o synovial changes similar to RA but not systemic
o

Imaging
Radiographs
o recommended views
AP, lateral and oblique of hand
o

findings
erosive osteoarthritis will show cartilage destruction,
osteophytes, and subchondral erosion (gull wing deformity)

Treatment
DIP Arthritis
o nonoperative
observation, NSAIDs
indications
first line of treatment for mild symptoms
o operative
fusion
indications
debilitating pain and deformity
technique
fusion with headless screw is most reliable
(nonunion in 10%)
2nd and 3rd digit fused in extension, 4th and
5th fused in 10-20 of flexion
Mucous Cyst
o nonoperative
observation
indications
first line of treatment as 2060% spontaneously resolve
o operative
cyst excision and osteophyte resection
indications
impending rupture
may need to do local rotational flap for skin
coverage
PIP Arthritis
o nonoperative
observations, NSAIDs

indications
first line of treatment in mild symptoms
operative
fusion
indications
border digits
poor bone stock
technique
headless screw fixation has highest fusion
rates
recreate normal cascade of fingers
index- 30, long- 35, ring- 40, small45
silicone arthroplasty
indications
long and ring finger
good bone stock
no angulation or deformity
outcomes
results are similar for both dorsal and volar
approaches
collateral ligament excision, volar plate release,
osteophyte excision
indications
predominant contracture
minimal joint involvement
Erosive osteoarthritis
o nonoperative
splints, NSAIDs
indications
tolerable symptoms
o operative
fusion
indications
intolerable deformity
technique
position of fusion same as above
o

Wrist Arthritis
Author: Mark Karadsheh
Introduction

SLAC wrist (scapholunate advanced collapse)


o most common
STT arthrosis
o second most common

SNAC (scaphoid nonunion advanced collapse)


DRUJ arthrosis

Pisotriquetrial arthrosis

degenerative
o primary OA
posttraumatic
o leads to SLAC/SNAC/DRUJ
inflammatory
o Rheumatoid arthritis
congenital
o may be secondary to Madelung's deformity
idiopathic
o may secondary to Kienbock's or Preiser's disease

SLAC
o

Injury to SL ligament --> palmar rotary subluxation of scaphoid -->


incongruency of joint surfaces --> arthrosis of radiocarpal joint --> arthrosis of
capitolunate joint
radiolunate typically spared

SNAC
proximal portion of scaphoid remains attached to lunate while distal scaphoid
flexes
o leads to early arthritis between radial styloid and distal scaphoid
o like SLAC, radiolunate typically spared
Rheumatoid arthritis
o wrist becomes supinated, palmarly dislocated, radially deviated, and ulnarly
translocated
o early disruption of DRUJ leads to dorsal subluxation of ulna (Caput-ulna)
o

Anatomy
Wrist ligaments and biomechanics
Imaging
Radiographs
o obtain standard hand series with additional views to visualize specific joints
o pisotriquetral joint (pisotriquetral arthrosis) obtain lateral in 30 degrees of
supination
Treatment

NSAIDs, bracing, intra-articular steroid injections


o indications
first line of treatment for mild to moderate symptoms

aimed at addressing diseased area


o SLAC
o SNAC
o Pisotriquetrial arthritis
excision of pisiform in refractory cases
o DRUJ abutment syndrome & arthrosis
distal ulna resection (Darrach procedure)
Sauv-Kapandji procedure
partial ulna resection and interposition

ulnar head replacement


can be used as primary procedure, or as salvage for failed
Darrach
early results are promising, long-term results pending
Rheumatoid arthritis

Radial Clubhand (radial deficiency)


Author: Terrill Julien
Introduction

A longitudinal deficiency of the radius


o likely related to sonic hedgehog gene
o thumb usually deficient as well
o bilateral in 50-72%
o incidence is 1:100,000
Associated with
o TAR
autosomal recessive condition with thrombocytopenia and absent
radius

different in that thumb is typically present

o
o

Fanconi's anemia
autosomal recessive condition with aplastic anemia
Fanconi screen and chromosomal breakage test to screen
treatment is bone marrow transplant
Holt-Oram syndrome
autosomal dominant condition characterized by cardiac defects
VACTERL Syndrome
vertebral anomalies, anal atresia, cardiac abnormalities,
tracheoesophageal fistula, renal agenesis, and limb defects)
VATER Syndrome
vertebral anomalies, anal atresia, tracheoesophageal fistula, esophageal
atresia, renal agenesis)
Classification

Type I
Type II
Type III
Type IV

Bayne and Klug Classification


Deficient distal radial epiphysis
Deficient distal and proximal radial epiphyses
Present proximally (partial aplasia)
Completely absent (total aplasia - most common)

Presentation
Physical exam
deformity of hand with perpendicular relationship between forearm and wrist

absent thumb
perform careful elbow examination
Imaging

entire radius and often thumb is absent

must order CBC, renal ultrasound, and echocardiogram to screen for associated
conditions

Treatment
Nonoperative
o passive stretching
target tight radial-sided structures
o observation
indicated if absent elbow motion or biceps deficiency
hand deformity allows for extra reach to mouth in presence of a
stiff elbow
Operative
o hand centralization
indications
good elbow motion and biceps function intact
done at 6-12 months of age
followed by tendon transfers

contraindications
older patient with good function
patients with elbow extension contracture who rely on radial
deviation
proximate terminal condition
technique
involves resection of varying amount of carpus, shortening of
ECU, and, if needed, an angular osteotomy of the ulna (be sure
to spare ulnar distal physis)
may do as two stage procedure in combination with a
distraction external fixator
if thumb deformity then combine with thumb reconstruction at
18 months of age

Ulnar Club Hand


Author: Chad Krueger
Introduction

A congenital upper extremity deformity


characterized by
o deficiency of the ulna and/or the ulnar sided carpal structures
o unstable elbow and stable wrist or vice versa
elbow abnormalities more common than wrist abnormalities
Epidemiology
o 5-10 times less common than radial club hand
Associated conditions
o medical
not associated with systemic conditions like radial club hand
o orthopaedic conditions
PFFD
fibula deficiency
scoliosis
phocomelia
multiple hand abnormalities
almost all patients have absent ulnar sided digits
Presentation
Symptoms
o limited function
o usually painless
Physical exam
o shortened, bowed forearm
o decrease in elbow function
o loss of ulnar digits
Classification

Bayne Classification
Type 0 Deficiencies of the carpus and/or hand only
Type 1 Undersized ulna with both growth centers present
Type II Part of the ulna is missing (typically the distal ulna is absent)
Type III Absent ulna
Type IV Radiohumeral synostosis

There is a subtype of each classification that is based on the first webspace


A = Normal
B = Mild deficiency of the webspace
C = Moderate to severe deficiency of the webspace
D = Absent webspace

Treatment
Goals
treatment depends on multiple factors including
hand position, thumb function, elbow stability, syndactyly
thumb condition is most important factor to consider for treatment
Nonoperative
o stretching and splinting
indications
used in early stages of treatment
Operative
o syndactyly release and digital rotation osteotomies
indications
done at 12-18 months of age
o radial head resection and creation of a one-bone forearm
indications
Stage II to provide stability at the expense of forearm motion
there is no good option for restoring elbow motion
corrective procedures should not be performed until the child is
at least 6 months old
o osteotomy of the synostosis
indications
may be required in Stage IV to obtain elbow motion
o

Congenital Dislocation of Radial Head


Author: David Abbasi
Introduction

Congenital dislocation of radial head


can bedifferentiated from a traumatic dislocation by
o bilateral involvement
o hypoplastic capitellum
o convex radial head
o other congenital anomalies
o lack of history of trauma
o difficult to reduce
Pathoanatomy
almost always posterior dislocation of radial head
often combined with bowing and shortening of radius

may have concurrent congenital anomalies

Anatomy
Elbow Anatomy & Biomechanics
Presentation
Symptoms
o patients often asymptomatic
o limited elbow ROM
Physical exam
o radial head prominence
o can have limited elbow ROM
especially in extension and supination
usually painless
Imaging
Radiographs

o
o
o

radial head posterior to capitellum


radial head can be large and convex
radius is short and bowed

Treatment
Nonoperative
o observation
indications
first line of treatment
Operative
o radial head resection
indications
usually done in adulthood if patient has
significant pain
restricted motion
cosmetic concern of elbow
outcomes
reduces pain
may improve some elbow ROM

Madelung's Deformity
Author: Heeren Makanji
Introduction
congenital dyschondrosis of the distal radial physis that leads to
partial deficiency of growth of distal radial physis
excessive radial inclination and volar tilt
ulnar carpal impaction

occurs predominantly in adolescent females


o common in gymnasts

caused by disruption of the ulnar volar physis of the distal radius


o repetitive trauma or dysplastic arrest
one hypothesis is due to tethering by Vickers ligament
o Vickers ligament is a fibrous band running from the distal radius to the lunate
on the volar surface of the wrist (short radio-lunate ligament)
o may be accompanied by anomalous palmar radiotriquetral ligament

autosomal dominant

Leri-Weill dyschondrosteosis
o rare genetic disorder caused by mutation in the SHOX gene
SHOX stands for short-statute homeobox-containing gene
anatomically at the tip of the sex chromosome
o causes mesomelic dwarfism (short stature)
o associated Madelung's deformity of the forearm
Presentation
Symptoms
o most are asymptomatic until adolescence
o symptoms include
symptoms of ulnar impaction
median nerve irritation
Physical exam
o leads to radial and volar displacement of hand

restricted forearm rotation

Imaging
Radiographs
o can see proximal synostosis
o characteristic undergrowth of the volar, ulnar corner of the radius
o increased radial inclination
o increased volar tilt
MRI
o indications
concern for pathologic Vickers ligament
o views
thickening ligament from the distal radius to the lunate

Treatment
Nonoperative
o observation
indications
if asymptomatic
o restricted activity
indications
activities with repetitive wrist impaction
recommend cessation of weight-bearing activities until pain
decreases

Operative
o physiolysis with release of Vickers ligament
indications
wrist pain or decreased range of motion
efficacy of prophylactic release of Vickers ligament in mild
deformity in skeletally immature patients unknown
o radial corrective osteomy +/- distal ulnar shortening osteotomy
indications
wrist pain or decreased range of motion
cosmetic deformity
functional limitations
o DRUJ arthroplasty
indications
highly controversial
painful DRUJ instability and limited supination/pronation
significant deformity may require staged procedures
Techniques
Physiolysis and release of Vickers ligament
o approach
volar approach to the distal radius
o technique
release a pathologically thick ligament
ligament approximately 0.5 to 1.0 cm in diameter
bar resection and fat grafting in the physis
Corrective radial osteotomy +/- distal ulnar shortening osteotomy
o goals
restore mechanics of distal radius
o approach
volar approach to the distal radius
o technique
severe deformities may benefit from a staged procedure with initial
distraction external fixation to avoid neurovascular stretching injury of
a single procedure
codome osteotomy allows correction of coronal and sagittal deformity
Complications
Incomplete physiolysis or premature growth arrest
Violation of radiocarpal or ulnocarpal joint
Incomplete deformity correction
Recurrent deformity
Nonunion of the osteotomy site
Continued ulnar impaction (if radial osteotomy done alone)

Congenital Radial Ulnar Synostosis


Author: Jan Szatkowski

Introduction

In normal development the radius and ulna divide from distal to proximal
therefore the synostosis is usually in proximal half
Epidemiology
bilateral in 60%
Genetics
familial cases with autosomal dominant inheritance has been reported
patients frequently have duplication in sex-chromosome

Presentation
Physical exam
o children often present at 3-5 years of age
no pronation or supination
fixed in varying degree of pronation (50% of patients have >
50 of pronation)

Imaging
Radiographs
o recommended view
AP and lateral of forearm and elbow
o findings
can see proximal synostosis

radius is heavy and bowed

Treatment
Nonoperative
o observation
indications
usually preferred treatment, especially if deformity
is unilateral

Operative
o osteotomy with fusion
surgery rarely indicated
indications
indicated to obtain functional degree of pronation
unilateral
fix the forearm in pronation of 30
bilateral
fix dominant forearm in pronation (1020)
nondominan forearm in neutral
technique
use percutaneous pins to aid fusion
perform at ~ 5 years of age
cannot recreate proximal radial-ulnar joint
with excision alone as it will reossify and
recur

Cleft Hand
Author: Colin Woon

Introduction
Definition
typical (central) cleft hand is characterized by absence of 1 or more central digits of
the hand or foot
o also known as lobster-claw deformity
Swanson type I failure of formation (longitudinal arrest) of central ray, leaving Vshaped cleft in the center of the hand
types
o unilateral vs bilateral
o isolated vs syndromic
Epidemiology
incidence
o rare (1:10,000 to 1:90,000)
demographics
o male:female ratio is 5:1 (more common in male)
location
o hands, usually bilateral
associated with absent metacarpals (helps differentiate from
symbrachydactyly)
missing middle finger
on the ulnar side, small finger is always present
o often involves feet as well
Pathogenesis
wedge-shaped degeneration of central part of apical ectodermal ridge
(AER) because of loss of function of certain genes expressed in that part of the AER

inheritance pattern
o autosomal dominant with reduced penetrance (70%)
o inherited forms become more severe with each generation

mutations
o deletions, inversions, translocations of 7q
split hand-split foot syndrome
affected families should undergo genetic counseling

Ectrodactyly-ectodermal dysplasia-cleft (EEC) syndrome


sensorineural hearing loss
syndactyly and polydactyly

functional limitation dependent on involvement of 1st webspace


aesthetically displeasing, but not functionally limiting
Classification
Manske and Halikis Classification
Characteristics
Thumb space not narrowed
Thumb space mildly narrowed

Type
Description
I
Normal web
IIA Mildly narrowed
web
IIB Severely narrowed Thumb space severely narrowed
web
Thumb and index rays syndactylized, web space
III Syndactylized web
obliterated
Index ray suppressed, thumb web space merged with
IV Merged web
cleft
Thumb elements suppressed, ulnar rays remain, thumb
V
Absent web
web space no longer present

Presentation

aesthetic limitation
functional limitation

absent or shortened central (third) ray


may have absent radial digits

Images

may have syndactyly of ulnar digits


o may involve feet

Symptoms
aesthetic limitation
functional limitation

absent or shortened central (third) ray


may have absent radial digits
may have syndactyly of ulnar digits
o may involve feet
Imaging
Radiographs
o recommended views
AP, lateral, oblique views of bilateral hands
foot radiographs if involved
Treatment
Nonoperative
o observation
indications
types I (normal web) and IV (merged web), no functional
impairment
Operative
o thumb web space, thumb, and central cleft reconstruction
indications
types IIA, IIB, III and V webs
Technique
Thumb, thumb web space reconstruction
o web space deepening, tendon transfer, rotational osteotomy, toe-hand transfer
o thumb web reconstruction has greater priority over correction of central cleft
o thumb reconstruction should not precede cleft closure as it might compromise
skin flaps
Central cleft reconstruction
o depends on characteristic of thumb web space
o close the cleft proper with local tissues from the cleft and stabilize and close
intermetacarpal space

Symphalangism
Author: Daniel Hatc
Introduction
Congenital digital stiffness that comes in two forms
hereditary symphalangism
nonherediatry symphalangism

location
o more common in ulnar digits
hophysiology
failure of IP joint to differentiate during development

inheritance pattern (hereditary type)


o autosomal dominant

syndactyly (nonhereditary type)


Apert's syndrome (nonhereditary type)
Poland's syndrome (nonhereditary type)
correctable hearing loss (hereditary type)

correctable hearing loss (hereditary type)

Presentation
Physical exam
inspection
o absence of flexion and extension creases
motion
o stiff digits
Imaging
Radiographs
IP joint space may appear narrow

Treatment
Nonoperative
o observation
no indication for surgery in children
Operative
o capsulectomy
outcome
limited success
o IP joint arthroplasty
outcome
limited success
o angular osteotomy
indications
rarely needed due to adequate digital function
o arthrodesis
indications
may be considered during adolesence to improve function and
cosmesis
rarely needed due to adequate digital function

Camptodactyly
Author: Rachel Frank
Introduction

Congenital digital flexion


deformity that usually occurs in the PIP joint of the small finger
Epidemiology
o prevalence
less than 1%
o location
can be unilateral or bilateral
if bilateral, can be symmetric or assymetric
Pathophysiology
o typically caused by either
abnormal lumbrical insertion/origin
abnormal (adherent, hypoplastic) FDS insertion
other less common causes include
abnormal central slip
abnormal extensor hood
abnormal volar plate
skin, subcutaneous tissue, or dermis contracture
Genetics
o most often sporadic
o can be familial with possible autosomal dominant inheritance with variable
penetrance
Associated conditons
o can be associated with more widespread developmental dysmorphology
syndromes

Classification

Type I
Type II
Type III

Camptodactyly Classification
Presents in infancy and affects males and females equally
Most common form
Presents in adolescence
Affects girls more often than boys
Multiple digits involved
More severe form
Usually associated with a syndrome

Kirner's
Deformity

Specific deformity of small finger distal phalanx with volarradial curvature (apex dorsal-ulnar)
Often affects preadolescent girls
Often bilateral
Usually no functional deficits

Presentation
Symptoms
o often goes unnoticed as usually only affects small finger and is very rarely
associated with any significant compromise in function
o typically painless and without motor/sensory deficits
Physical exam
o flexion deformity of small finger PIP joint
flexible (correctable) or fixed (non-correctable) deformity
progressively worsens over time if untreated
may rapidly worsen during growth spurts
o normal strength, sensation, perfusion
o usually normal DIP and MCP joint alignment, however compensatory
contractures can develop
o no swelling, erythema, or warmth; not associated with inflammation
Imaging
Radiographs
o often normal, especially in early stages
o later stages: possible decrease in P1 head convexity; possible volar
subluxation and flattening of base of P2
Treatment

passive stretching, splinting


o indications
nonoperative treatment is favored in most cases
best for PIP contracture < 30 degrees
o technique
passive stretching + static splinting
o outcomes
variable outcomes
best outcomes with early intervention

FDS tenotomy +/- FDS transfer


o indications

reserved for cases of progressive deformity leading to functional


impairment
o technique
must address all abnormal anatomy
passive (correctable) deformities
FDS tenotomy, or
FDS transfer to radial lateral band if full active PIP extension
can be achieved with MCP flexion
osteotomy vs. arthrodesis
o indications
severe fixed deformities
o outcomes
variable outcomes

Clinodactyly
Author: David Abbasi

Introduction
Congenital curvature of digit in radioulnar plane
found in 25% of children with Down's syndrome and 3% of general
population
Pathoanatomy
autosomal dominant inheritance
middle phalanx of small finger most commonly affected

Anatomy
Anatomy of ligaments of the fingers

Classification
Clinodactyly Classification
Type I Minor angulation with normal length (most common)
Type Minor angulation with short length
II
Type Significant angulation and delta phalanx (c-shaped epiphysis and
III
longitudinal bracketed diaphysis)

Presentation
Physical exam
o function rarely significantly compromised
o daily activities can be affected if deformity reaches 30-40 degrees

Imaging
Radiographs
o

C-shaped physis can result in a delta phalanx

Treatment
Nonoperative
observation
o indications
favored in most cases
splinting is not indicated
Operative
phalanx opening wedge osteotomy +/- bone excision
o indications
Type III (delta phalanx)
when deformity (delta phalanx) encroaches digit space of
neighboring short digit
o technique
excision of extra bone

Syndactyly
Author: Tracy Jones
Introduction
Most common congenital malformation of the limbs
Epidemiology
o incidence
1 in 2,000 - 2,500 live births
o demographics
M>F
Caucasians > African Americans
o ray involvement
50% long-ring finger
30% ring-small finger
15% index-long finger
5% thumb-index finger
Pathophysiology
o failure of apoptosis to separate digits
Genetics
o autosomal dominant in cases of pure syndactyly
reduced penetrance and variable expression
positive family history in 10-40% of cases
Associated conditions
o acrosyndactyly
digits fuse distally and proximal digit has fenestrations (e.g.,
constriction ring syndrome)
o

Poland Syndrome

o
o

Apert Syndrome
Carpenter syndrome
acrocephalopolysyndactyly

Classification

Simple
Complex
Complicated
Complete vs.
Incomplete

Syndactyly Classification
Only soft tissue involvement, no bony connections
Side to side fusion of adjacent phalanges
Accessory phalanges or abnormal bones involved in fusion
Complete syndactyly the skin extends to finger tips; with
incomplete, skin does not extend to fingertips

Treatment
Operative
o digit release
indications
syndactyly
perform at ~ 1 year of age
acrosyndactyly
perform in neonatal period
Technique
Digit Release
o if multiple digits are involved perform procedure in two stages to avoid
compromising vasculature
o release digits with significant length differences first to avoid growth
disturbances
o zigzag flaps are created during release to avoid longitudinal scarring
Complications
Web creep
o most common complication of surgical treatment
o treatment
reconstruct web space with local skin flaps
Nail deformities

Poland Syndrome
Author: Michael Hughes MD

Introduction
unilateral chest wall hypoplasia
o due to absence of sternocostal head of pectoralis major
hypoplasia of the hand and forearm
symbrachydactyly and shortening of middle fingers
o
o

result of absence or shortening of the middle phalanx


simple complete syndactyly of the short digits

1 in 32,000 live births


occurs in 10% of syndactyly cases

thought to be linked to subclavian artery hypoplasia

Presentation
Physical exam
extent of hand and chest involvement varies
chest deformities
o hypoplasia or absence of the pectoralis major, pectoralis minor, deltoid,
serratus anterior, external oblique, and latissimus dorsi
o Sprengels deformity
o scoliosis
o dextrocardia
o absence or underdevelopment of the breast
hand deformities
o syndactyly
o hypoplasia or absence of metacarpals or phalanges
o absence of extensors or flexor tendons
o carpal coalition or hypoplasia

o
o

radioulnar synostosis
nail agenesis

Imaging
CT scan
o will show absent perctoralis major
CT scan
o will show absent perctoralis major
Operative
o syndactyly release
indications
performed in most patients
technique
complete syndactyly release produces skin deficiency that
requires skin grafting
perform only one side of the digit at a time to avoid vascular
complications
local flap is created for commisure reconstruction followed by
interdigitating zigzag dorsal and palmar flaps along the medial
and lateral aspect of the digit

Complications
Skin graft failure
Excessive tension
Improper flap planning
Digital artery injury
Web creep
Nail deformity

Apert Syndrome
Author: Jason McKean

Introduction
Syndrome characterized by

bilateral complex syndactyly of hands and feet


o index, middle, and ring fingers most affected
symphalangism
premature fusion of cranial sutures (craniosynostosis) results in
flattened skull and broad forehead (acrocephaly)
hypertelorism (increased distance between paired body parts, as in
wide set eyes)
normal to moderately disabled cognitive function
glenoid hypoplasia
radioulnar synostosis
Genetics
autosomal dominant, but most new cases are sporadic

mutation of FGFr2 gene


Epidemiology
incidence is 1/80,000 live births
Prognosis
spectrum of normal to moderately disabled cognitive function

Presentation
Physical exam
dysmorphic face
o craniosynostosis results in flattened skull and facial features
rosebud hands (complex syndactyly where the index, middle, and ring finger share a
common nail)
Imaging

Radiographs

will show complex syndactyly

Treatment
Operative
surgical release of border digits
o indications
perform ~ 1 year of age
digit reconstruction
o indications

perform ~ 1.5 years of age


o

to convert central three digits into two digits

Polydactyly of Hand
Author: Ujash Sheth

Topic updated on 10/19/15 7:19pm


Introduction
A congential malformation of the hand
Three forms exist
o preaxial polydactlyly
thumb duplication
o postaxial polydactlyly
small finger duplication
o central polydactlyly

Preaxial Polydactyly (Thumb Duplication)


Epidemiology
o incidence

1 per 1,000 to 10,000 live births


Type IV most common (43%)
Type II second most common (15%)
o demographics
M>F
caucasian > African Americans
Genetics
o inheritance pattern
usually unilateral and sporadic
except for Type VII which is associated with several syndrome
including
Holt-Oram syndrome
Fanconi's anemia
Blackfan-Diamond anemia
imperforate anus
cleft palate
tibial defects
Associated conditions
o pollex abductus

abnormal connection between EPL and FPL tendons, seen in


approximately 20% of hypoplastic and duplicated thumbs
suggested by abduction of affected digit + absence of IP joint
crease
Classification

Type I
Type II
Type III
Type IV
Type V
Type VI
Type VII

Wassel Classification of Preaxial Polydactyly


Bifid distal phalanx
Duplicated distal phalanx
Bifid proximal phalanx
Duplicated proximal phalanx (most common)
Bifid metacarpal
Duplicated metacarpal
Triphalangia

Treatment
o

operative
goals of treatment
to construct a thumb that is 80% of the size of the
contralateral thumb
resect smaller thumb (usually radial component)
preserve / reconstruct medial collateral structures in
order to preserve pinch function
reconstruction of all components typically done in one
procedure
type 1 combination procedure (Bilhaut-Cloquet)
indications
type I, II, or III
technique
involves removing central tissue and combining
both digits into one
outcomes
approximately 20% have late deformity

problems include stiffness, angular and size


deformity, growth arrest, and nail deformities
type 2 combination procedure
indications
usually favored approach for type III and IV
type V and VI usually require more complex
transfer of intrinsics and collateral ligaments
technique
preserve skeleton and nail of one component and
augment with soft tissue from other digit and
ablation of lesser digit (radial digit most
commonly)
type 3 combination procedure
indications
when one digit has superior proximal component
and one digit has superior distal fragment (type
V, VI, and VII)
technique
a segmental distal transfer (on-top plasty)
Postaxial Polydactyly (Small Finger Duplication)
Epidemiology
o demographics
10X more common in African Americans

Genetics
o inherited as autosomal dominant (AD) in African Americans
o more complex genetics in caucasians and a thorough genetic
workup should be performed
Classification
o Type A - well formed digit
o Type B - rudimentary skin tag (vestigial digits)
Treatment
o operative
formal reconstruction with a Type 2 combination
indications

Type A
technique
preserve radial digit
preserve or reconstruct collateral ligaments
from ulnar digit remnant
preserve muscles
tie off in nursery or amputate before 1 year of age
indications
Type B
Central Polydactyly
Epidemiology
o commonly associated with syndactyly

extra digit may lead to angular deformity or impaired


motion
Treatment
o osteotomy and ligament reconstructions
indications
perform early to prevent angular growth deformities

Questions

Which of the following congenital hand deformities displayed in figures A-E is more
prevalent in patients of African-American ancestry?
FIGURES: A B C D E

1.

Figure A

2% (25/1415)

2.

Figure B

74% (1049/1415)

3.

Figure C

12% (170/1415)

4.

Figure D

6% (84/1415)

5.

Figure E

5% (71/1415)

PREFERRED RESPONSE 2
Image B is consistent for postaxial polydactyly, which is more prevalent in patients of
African-American ancestry.
The cohort study by Woolf found the incidence of postaxial polydactyly in African americans
is 12.42 per 1,000 (1.2%) compared to the Caucasian incidence of 0.91 per 1,000 (0.09%). If
postaxial polydactyly is found in a patient of Caucasian ancestry then further workup for
underlying syndromes (chondroectodermal dysplasia or Ellis-van Creveld syndrome) is
needed.
The article by Orioli is a case-control study that hypothesizes that a sex-linked recessive
modifier gene occurs more frequently in African americans and this gene then promotes the
autosomal dominant polydactyly gene.
Incorrect Answers: Constriction band syndrome or amniotic band syndrome is a type of
pseudosyndactyly (Figure A) and is not the result of failure of differentiation during
embryogenesis, but a result of injury by bands after the fingers are formed. Preaxial
polydactyly (Figure C) is more common in caucasians and is usually sporadic except for
triphalangism which is associated with Holt-Oram and Fanconi's Anemia. Syndactyly (Figure
D) is defined as an abnormal interconnection between adjacent digits and syndactyly
variations are associated with Apert syndrome and Poland syndrome. Macrodactyly (Figure
E) represents overgrowth of all structures of the involved digit and is associated with
neurofibromatosis and Klippel-Trenaunay-Weber syndrome.

Macrodactyly (local gigantism)


Author: Ben Taylor

Topic updated on 02/13/14 7:27pm

Introduction

Nonhereditary congenital digit enlargement


Epidemiology
o demographics
very rare
o location
90% are unilateral
70% involves more than one digit

index involved most frequently


in order of decreasing frequency, the long finger, thumb, ring,
and small are also involved
can involve digits of the hand or foot
70% involves more than one digit
index involved most frequently
in order of decreasing frequency, the long finger, thumb, ring, and
small are also involved
can involve digits of the hand or foot
90% are unilateral
risk factors
none known
Pathophysiology
o

etiology unknown
o no genetic correlations known to date
o affected digits correspond with neurologic innervation
the median nerve being the most common
Associated conditions
o lipfibromatous hamartoma of the median nerve is the adult homolog
o has been associated with:
Proteus syndrome
Banayan-Riley-Ruvalcabe's disease
Maffucci syndrome
Olliers disease
Milroys disease
Prognosis
o if static, asymmetry does not worsen
o if progressive, asymmetry worsens with time
o

Classification
Functional Classification
Static
Present at birth and growth is linear with other digits
Progressive Not as noticable at birth but shows disproportionate growth over
time

Presentation
History
o asymmetry to digits can be present at birth or appearing over
time
Symptoms
o pain
o inability to use digits
o complaints of cosmetic issues
Physical exam
o inspection & palpation
thick, fibrofatty tissue involving enlarged digits
o ROM & instability
often limited ROM due to soft tissue constraints
Imaging
Radiographs
o recommended views
biplanar hand radiographs
o findings
enlarged phalanges to involved digits

may see malalignment of joints or angled phalanges


CT, MRI
o not typically needed
Studies
Angiography
o only needed if used for surgical planning
Treatment
Nonoperative
o observation
in mild cases
Operative
o

epiphysiodesis
indications
single digit
perform once digit reaches adult length of same sex
parent
most common approach
postoperative care
soft tissue care
early ROM
osteotomies and shortening procedures
indications
thumb involvement
multiple digit involvement
severe deformity
postoperative care
local soft tissue care
early ROM
amputations

indications
severe involvement of digit
non-reconstructable digit

Question

(OBQ08.215) An 8-year-old boy's parents are concerned about the appearance of the child's middle
finger. The child denies pain and his digital neurovascular status is normal. A clinical photograph and
radiograph are provided in figures A and B. For children with this condition, which of the following is
the best intervention to achieve a finger that is proportional to the rest of the hand?
FIGURES: A B

1.

Epiphysiodesis now

13% (246/1837)

2.

Epiphysiodesis when the finger reaches adult length of the father

77% (1422/1837)

3.

Compression wrapping until proportional size is achieved

1% (22/1837)

4.

Resection of hypertrophic nerves

4% (66/1837)

5.

Osteotomy at skeletal maturity.

4% (76/1837)

PREFERRED RESPONSE 2
Clinical photograph and radiographs demonstrate macrodactyly of the middle finger, a rare congenital
malformation enlarging all structures of the digit.
Ishida et al reviews 23 cases of surgically treated macrodactyly finding favorable results with
epiphysiodesis/epiphysiodectomy while resection of hypertrophic nerves was unsuccessful in
preventing overgrowth. The epiphysiodesis is performed once the finger reaches the length of the
same sex parent, using their digit as a template for final growth.

Constrictive Ring Syndrome


Author: Joshua Blomberg

Topic updated on 02/14/14 4:36pm


Introduction

A malformation due to intrauterine rings or bands which constrict fetal


tissue
o the anatomy promximal to the constriction or amputation is normal
o also referred to as Streeter dysplasia
Epidemiology
o incidence
reported incidence varies between 1/1200 and 1/15000 live
births
o location
usually affects distal extremities
rare for only one ring to be present as an isolated malformation
o risk factors
prematurity
maternal illness
low birth weight
drug exposure
Genetic
o sporadic condition with no evidence of hereditary disposition
Pathoanatomy
o exact etiology unknown but theories include
intrinsic anomaly in germ plasm resulting in the defects
intrauterine disruption during pregnancy
intrauterine trauma
Associated conditions
o club foot
most common
Prognosis
o in rare cases, can cause limb amputation or death
Classification
Degrees of Constrictive Ring Syndrome
Simple constriction rings
Mild ring with no distal deformity or
lymphedema

Rings with distal deformity


Acrosyndactyly

Amputations

Ring may cause distal lymphedema in


association with deformity
Fusion between the more distal portions
of the digits with the space between the
digits varying from broad to pinpoint in
size.
Loss of limb distal to ring

Presentation
Symptoms
o

most patients get diagnosed at birth

Physical exam
o

check for distal pulses and perfusion

Imaging

Ultrasound
o intrauterine diagnosis can be made with ultrasound at end of first
trimester

Treatment
Operative
o surgical release with multiple circumferential Z-plasties
indications
if circulation is compromised by edema or limb has
contour deformity

perform early (neonatal)


technique
acrosyndactyly is treated with distal release early in
neonatal period
intrauterine band release can be done if limb is found to
be at risk of amputation (rare)

Question

Figure A depicts a child with a congenital abnormality. Which of the following is true regarding this
condition?
FIGURES: A

1.

Circumferential trunk involvement is more common than distal extremities involvement

0% (6/1659)

2.

Risk factors include late gestation (>44 weeks) and high birth weight (>3500g)

7% (114/1659)

3.
Incomplete circumferential bands not directly interfering with lymphatic circulation should be
resected

2% (26/1659)

4.

There is a strong correlation with anterolateral tibial bowing

1% (16/1659)

5.
Complete circumferential bands that interfere with lymphatic drainage can be treated with
band excision and z-plasty.

90% (1486/1659)

PREFERRED RESPONSE 5
The image and vignette describe a patient with constriction band syndrome (CBS). In the case of
lymphatic obstruction or vascular compromise, the treatment of CBS is band excision. There are
many terms used to describe this phenomenon. However, the etiology is the entanglement of fetal
parts in the amniotic membrane.
Foulkes et al reviewed 71 cases of congenital constriction band syndrome (CCBS). They found the
average patient had three involved limbs, with a predilection for distal, central digits of the upper
extremity. There was a strong correlation with abnormal gestation and clubfoot. Treatment included
distraction osteogenesis and free osteocutaneous transfer.
Goldfarb et al reviewed amniotic constriction band syndrome (ABS), highlighting its association with
annular constriction of multiple extremities. They classified ABS into classic (disruptions and
deformations) and non-classic (malformations). ABS is due to disruptions (amputations,
acrosyndactyly), deformations (oligohydraminos, scoliosis, talipes equinovarus) and malformations
(body-wall defects, cleft lip/palate). As there is moderate overlap between the classic and nonclassic,
additional research into the underlying cause is being investigated.
Green described a one-stage release of circumferential constriction bands in three patients. The
advantages of this technique are the decreased need for anesthesia and subsequent procedures as well
as facilitating postoperative care.

Kawakura et al reviewed the intrinsic and extrinsic theories of (CBS). The most common
manifestations are distal extremity involvement, intrauterine amputations and acrosyndactyly.
Excision of bands and mobilization of subcutaneous adipose tissue as described by Upton is seen in
Illustration A.
Incorrect Answers:
Answer 1: Distal extremities are more affected than the trunk
Answer 2: Risk factors include low birth weight (<2500g), prematurity (<37wks), maternal drug
exposure, trauma during pregnancy and attempted fetal termination during the first trimester.
Answer 3: Shallow bands that do not interfere with circulation or lymphatic drainage do not need to
be released.
Answer 4: There is a strong correlation with clubfoot, not anterolateral bowing.
Illustrations:

Streeter's Dysplasia

Author: Tracy Jones


Topic updated on 11/17/15 2:33pm

Introduction

Amniotic band syndrome occurs when loose fibrous bands of


ruptured amnion adhere to and entangle the normal developing
structures of the fetus.
o also referred to as
amniotic disruption sequence
constriction ring syndrome
premature amnion rupture sequence
Streeter's dysplasia
Epidemiology
o incidence
1:15,000 live births
o demographics
affects males and females equally
o location
occurs in hands and fingers 80% of the time
greater than 90% occur distal to wrist
Pathophysiology
o no firmly established etiology
o most accepted theory is that the disrupted amnion releases
fibrous membranous strands which wrap around the
developing limb in a circumferential fashion
Genetics

inheritance pattern
sporadic and not hereditary
Associated conditions
o orthopaedic
clubfoot
syndactyly
o nonorthopaedic
cleft palate
cleft lip
craniofacial defects
Prognosis
o related to location and severity of constricting bands
o

Classification

Type I
Type II

Patterson Classification
Simple constriction ring
Deformity distal to ring (hypoplasia, lymphedema)
Edema may or may not be present

Type III
Type D

Fusions distally (syndactyly, acrosyndactyly)


Amputation

Presentation
Physical exam
o normal anatomy proximal to constriction ring
o bands perpendicular to longitudinal axis of the digit or limb
most common presentation
o central digits more commonly affected
o amputations distal to constriction site can be found
o when no amputations present look for
secondary syndactyly
bony fusions
may observe sinus tracts proximally between digits

Treatment
Nonoperative
o observation
indications
Type I (simple constriction ring)
Operative
o excision or release of constriction band

indications
Type I with compromise of digital circulation
circumferential Z-plasties
indications
Type II
distal deformities present
surgical release of syndactyly
indications
Type III with distal fusions
reconstruction of involved digits or limb (i.e., lengthening
of bone, deepening of web space)
indications
Type IV to improve function

Thumb Hypoplasia
Author: Mark Karadsheh

Topic updated on 02/14/14 4:37pm

Introduction

Congenital underdevelopment of the thumb frequently associated


with partial or complete absence of the radius
Epidemiology
o incidence
1/100,000 live births
o demographics
male = female
o location
bilateral involvement in ~60% of patients
right hand more common than left
Pathophysiology
o exact cause during embryologic development has yet to be
elucidated
Associated anomalies
o greater than 80% of patients will have associated anomalies
including
VACTERL
Holt-Oram
thrombocytopenia-absent radius (TAR)
Fanconi anemia
Classification & Treatment
Treatment algorithm depends on presence of carpometacarpal joint
stability
Type
Type
I

Blauth Classification
Description
Minor hypoplasia
All musculoskeletal and
neurovascular components of
the digit are present, just small

Treatment
No surgical treatment
required

in size
Type
II

All of the osseous structures are


present (may be small)
MCP joint ulnar collateral
ligament instability
Thenar hypoplasia

Type
IIIA

Musculotendinous and osseous


deficiencies
CMC joint intact
Absence of active motion at the
MCP or IP joint

Type
IIIB

Musculotendinous and osseous


deficiencies.
Basal metacarpal aplasia with
deficient CMC joint
Absence of active motion at the
MCP or IP joint.

Type
IV

Floating thumb
Attachment to the hand by the
skin and digital neurovascular
structures

Stabilization of MCP
joint
Release of first web
space
Opponensplasty

Thumb amputation &


pollicization

Type
Complete absence of the thumb
V
Presentation
Physical exam
o inspection
extrinsic tendon abnormalities
pollex abductus
flexor pollicus longus attaches to normal insertion and
the extensor tendon

hypoplasia of thenar musculature

absence of skin creases indicates muscle or tendon abnormalities


excessive abduction of MCP joint
range of motion and instability
ulnar collateral ligament laxity
web-space tightness
evaluation for associated anomalies is essential
cardiac
auscultation
echocardiography
kidneys
ultrasound
abdomen
ultrasound

Imaging
Radiographs
o recommended views
bilateral films of hand, wrist and forearm
Studies
Labs
o

peripheral blood smear and complete blood count

important to rule out Fanconi anemia


Additional studies
o chromosomal challenge test
detects Fanconi anemia before bone marrow failure
Treatment
Nonoperative
o observation
indications
Type I hypoplasia where augmentation of thenar musculature
(thumb abduction) is not necessary
Operative
o opposition tendon transfer (opponensplasty)
indications
Type I hypoplasia with insufficient thumb abduction
o release of first web space, opposition transfer, stabilization of MCP joint
indications
Type II and IIIA hypoplasia
o pollicization
indications
Type IIIB, IV, V hypoplasia
Surgical Techniques
Opponensplasty (opposition transfer)
o technique
performed using
flexor digitorum superficialis or
abductor digiti minimi
First web space deepening
o technique
usually performed with Z-plasty
Stabilization of MCP joint
o technique
three options
fusion
reconstruction of UCL with FDS
reconstruction of UCL with free tendon graft
Pollicization

technique
plan skin incision to avoid skin grafts
isolate index finger on its neurovascular bundles
detach first dorsal and palmar interosseous muscles
shorten digit by removing index finger metacarpal and epiphyseal plate
stabilize index MCP joint
reattach and balance musculotendinous units
reconstruct long extensor tendons
rebalance flexor tendons

Question
(OBQ10.6) The parents of a newly adopted 3-year-old boy bring the child to the office for
evaluation of his thumb. A clinical photograph is provided in figure A. Which of the
following is the most important factor in determining thumb reconstruction versus ablation
and pollicization?
FIGURES: A

1.

Stability of the carpometacarpal joint

89% (1410/1581)

2.

Functional ROM of the wrist

0% (4/1581)

3.

Functional ROM of the index, middle, ring, and small fingers

2% (38/1581)

4.

Skin contracture of the first web space

2% (29/1581)

5.

Absence of a thumb interphalangeal joint

6% (95/1581)

PREFERRED RESPONSE 1
The clinical photograph demonstrates a hypoplastic thumb. The incidence of thumb
hypoplasia is 1 in every 100,000 births and associated anomalies including radial aplasia,
thrombocytopenia, and renal/cardiovascular/CNS anomalies are frequent. Stability of the
carpometacarpal joint is essential for success of thumb reconstruction procedures. If CMC
stability is deficient, then ablation and pollicization is preferred.
Light et al describe the evaluation and surgical technique involved in treating the hypoplastic
thumb. They note that severe thumb hypoplasia and aplasia are best treated by thumb ablation
and pollicization of the index finger.

Congenital Trigger Thumb


Author: Evan Watts

Topic updated on 04/06/15 6:37pm

Introduction

Pediatric condition of the thumb that results in abnormal flexion at


interphalangeal (IP) joint
Epidemiology
o prevalence
3 per 1,000 children are diagnosed by the age of 1 years
o demographics
separate entity to adult acquired trigger thumb
male and females affected equally
o location
25% are bilateral
o risk factors
etiology of pediatric trigger thumb remains unknown
Pathophysiology
o pathoanatomy
flexor pollicis longus (FPL) tendon is thickened due to
abnormal collagen degeneration and synovial proliferation
increased FPL tendon diameter, compared to the A1
pulley, causes disruption of normal tendon gliding
Genetics
o most commonly an acquired condition
o some reports suggest autosomal dominance with variable penetration
o term congenital trigger thumb is now considered a misnomer
Prognosis
o natural history
usually begins with notable thumb triggering that progresses to
a fixed contracture
spontaneous resolution unlikely after age of 2 years old
Presentation

History
o presenting complaint is usually fixed thumb flexion deformity at
the IP joint
o history of trauma is rare
o family history of disease is rare
Symptoms
o usually painless
o may be bilateral
Physical exam
o inspection
flexion deformity at the IP joint
o motion
prominence of the flexor tendon nodule, referred to as
"Notta's node"
deformity may be fixed with loss of IP joint extension
o neurovascular
usually preserved
Imaging
Radiographs
o recommended views
AP and lateral views of the hand
o additional views
dedicated thumb views
o indications
recommended only if history of trauma
o findings
usually diagnosed based on clinical presentation
radiographs are usually normal
Treatment
Nonoperative
o passive extension exercises and observation
indications
usually considered first line of treatment
not recommended for fixed deformities in older
children
technique
passive thumb extension exercises
duration based on clinical response
outcomes
30-60% will resolve spontaneously before the age
of 2 years old
<10% will resolve spontaneously after 2 years old

extension splinting
indications
consider alongside stretching regime
not recommended with fixed deformities in older
children
technique
splints maintain IP joint hyperextension and
prevent MCP joint hyperextension
duration for 6-12 weeks
outcomes
50-60% resolution in all age groups
high drop out rate from therapy
Operative
o A1 pulley release
indications
fixed deformity beyond age of 12 months of age
failed conservative treatment
outcomes
65-95% resolution in all age groups
o

Techniques
A1 Pulley Release
o

open release
small transverse incision in the thumb MCP flexion crease,
extending over the A1 pulley
protect the radial digital nerve
sharp dissection of the A1 pulley
identify the Notta nodule in the FPL tendon
watch nodule under direct vision during passive IP extension
of the thumb to ensure there is smooth FPL tendon gliding

Complications
Digital nerve injury
o caution must be performed during release as digital nerves at high
risk due to proximity to flexor tendon and A1 pulley
Wound complications
o scar contracture
o abscess
o infection
IP flexion deficit
Bow-stringing of flexor tendon
o usually related to release of the oblique pulley

Congenital Clasped Thumb


Author: Evan Watts

Topic updated on 08/29/15 2:53pm

Introduction

Congenital flexion-adduction deformity of the thumb that persists


beyond the 3rd or 4th month of life
Epidemiology
o demographics
male-to-female ratio is approx 2.5:1
o risk factors
exact causative factors are not well known
possible pre-disposing factors include
consanguinity
family history
Pathophysiology
o genetics
autosomal dominance inheritance of variable
expressivity
may be sporadic
o pathoanatomy
attenuation or deficiency of extensor pollicis longus or
brevis, or both
usually associated with contracture of the 1st web space
contracture of adductor pollicis or first dorsal
interosseous muscle
global instability of first MP joint
abnormal articular cartilage of first MP joint
Orthopedic considerations
o common manifestations associated with disease
lower limb anomalies
congenital vertical talus
congenital talipes equinovarus (bilateral)
upper limb anomalies

flexion deformities of the four fingers


Associated conditions
o arthrogryphosis (congenital joint contractures affect two or
more areas in the body)
o digitotalar dysmorphisms
o Freeman-Sheldon syndrome
o X-linked MASA syndrome
Classification
Tsuyuguchi Classification of Clasped Thumb

Type
Type I (Supple clasped
thumb)

Feature
Thumb able to be passively abducted and
extended against resistance of thumb flexors. No
other digital anomly present.
Type II (Clasped thumb Thumb cannot be passively extended and abducted.
This may occur with or without other digital anomaly.
with contracture)
Type III (Rigid clasped
Clasped thumb that is associated with arthrogryposis
thumb)
and marked soft-tissue deficits.

Presentation
History
o persistent flexion-adduction deformity beyond 3rd or 4th month of
life, usually bilateral
o family history
o pre-natal history
Symptoms
o pain usually with a contracture
o associated with other musculo-skeletal deformities

Examination
o type of clasped thumb
o associated anomalies
Treatment
Nonoperative
o serial splinting and stretching for 3-6 months
indications
first-line treatment over a trial period of 3-6 months
for all types
begin treatment around the age of 6 month old
outcome
good definitive results with Type I congenital
deformities when one of the EPL or EPB tendons
are present
poor results with Type I deformities when both
EPL/EPB tendons are absent
poor results with Type II or III deformities
Operative
o tendon transfer to EPL
indications
Type I or II with residual deficiency in active
extension
technique
extensor indicis transfer to remnant of extensor
tendon
o thumb reconstruction
indications
failed conservative treatment
soft-tissue deficiency in the thumb-index finger
webspace (Type III)
Type II or III deformity with significant MCP joint
contractures
technique
o arthrodesis
indications
severe deformities when skin release and tendon
trasnfer cannot overcome joint deformity.
Techniques

Thumb reconstruction
o usually delayed until the age of 3 to 5 years old
o procedure based on amount of contracture and may include
widening the first webspace

transposition flap of skin (dorsal rotational advancement flap)


four-flap or five-flap Z plasty of skin
deepening the first webspace by releasing soft-tissue
releasing origins of thenar musculature from transverse carpal ligmant
releasing joint capsule of first MP joint
tendon transfer
FPL Z-lengthening in the forearm
isolated EPB tendon will not usually require tendon transfer
EPB and EPL absence is best reconstructed with tendon transfer

Ganglion Cysts
Author: Michael Day

Topic updated on 08/29/15 5:03pm

Introduction

A mucin-filled synovial cyst caused by either


o trauma
o mucoid degeneration
o synovial herniation
Epidemiology
o incidence
it is the most common hand mass (60-70%)
o location
dorsal carpal (70%)
originate from SL articulation
volar carpal (20%)
originate from radiocarpal or STT joint
volar retinacular (10%)
originate from herniated tendon sheath fluid
dorsal DIP joint (mucous cyst, associated with
Heberden's nodes)

Pathophysiology
o filled with fluid from tendon sheath or joint
o no true epithelial lining

Associated conditions
o median or ulnar nerve compression
may be caused by volar ganglion
o hand ischemia due to vascular occlusion
may be caused by volar ganglion
Presentation
Symptoms
o usually asymptomatic
o may cause issues with cosmesis
Physical exam
o inspection
transilluminates (transmits light through tissue)
o palpation
firm and well circumscribed
often fixed to deep tissue but not to overlying skin
o vascular exam
Allen's test to ensure radial and ulnar artery flow for volar
wrist ganglions
Imaging
Radiographs
o normal
MRI
o
o

indications
not routinely indicated
findings
shows well marginated mass with homogenous fluid signal
intensity

Ultrasound
o useful for differentiating cyst from vascular aneurysm
o may provide image localization for aspiration while avoiding artery

Histology

Biopsy
o indications
not routinely indicated
o findings
will show mucin-filled synovial cell lined sac
Treatment
Nonoperative
o observation
indications
first line of treatment in adults
children
76% resolve within 1 year in pediatric
patients
o closed rupture
home remedy
high recurrence
o aspiration
indications
second line of treatment in adults with dorsal
ganglions
aspiration typically avoided on volar aspect of wrist
due to radial artery
outcomes
higher recurrence rate (50%) than surgical
resection but minimal risk so reasonable to
attempt
Operative
o surgical resection
indications
severe symptoms or neurovascular manifestations
technique
requires adequate exposure to identify origin and
allow resection of stalk and a portion of adjacent
capsule
at dorsal DIP joint: must resect underlying
osteophyte
results
volar ganglions have higher recurrence after
resection than dorsal ganglions (15-20%
recurrence)

Complications
With aspiration
o infection (rare)
o neurovascular injury
With excision
o infection
o neurovascular injury (radial artery most common)
o injury to scapholunate interosseous ligament
o stiffness

Questions

1. (OBQ12.64) A 54-year-old male presents with a slowly enlarging mass on the dorsum of
his left wrist which has been present for 3 years. He denies any significant symptoms.
Physical exams shows a 1 cm palpable mass. A MRI is shown in Figure A. A biopsy of this
lesion would most likely show?
FIGURES: A

1.

Synovial cells with mucin accumulation

93% (2435/2625)

2.

Proliferating histiocytes of moderate cellularity and frequent multinucleated giant cells

3% (87/2625)

3.

Polymorphonuclear neutrophils

0% (11/2625)

4.

Spindle cells arranged in intersecting bundles

1% (20/2625)

5.

Lipocytes, spindle cells, and scattered atypical giant cells

2% (60/2625)

PREFERRED RESPONSE 1
The clinical presentation is consistent with a ganglion cyst. Histology of a ganglion cyst
would show a mucin filled synovial cyst.
Ganglion cysts are the most common mass found on the hand or wrist. Dorsal ganglions
originating from the scapholunate (SL) ligament are the most common (60%). They are
caused by trauma, mucoid degeneration, or synovial herniation. On exam, they appear fixed
to the underlying deep tissue, but not to the skin and are commonly translucent to light
illumination. Radiographs of a ganglion will be normal, although a T2-weighted MRI axial
image of the wrist will show increased signal where the cyst is located.
Nahra et al. give a thorough review of ganglion cysts including known epidemiology,
etiology, and treatment. He notes that dorsal ganglions are the most common (60% to 70%)
and are found between the third and fourth dorsal compartments arising most commonly from
the scapholunate ligament. Supportive splints and anti-inflammatories in conjunction with
aspiration are an important part of nonoperative management. Aspiration of dorsal ganglions
(not recommended for volar ganglions) yields a recurrence rate of around 50%. Surgical
intervention is not common, but there is a low recurrence rate when performed.
Peh et al. reviewed the MRI features found for benign soft tissue masses in the hand,
especially ganglion cysts. They noted that ganglion cysts show an increased signal intensity
on T2-weighted MRI images.
Figure A is a T2 weighted MRI showing signal intensity in the fluid of the cyst consistent
with a ganglion cyst. Illustration A is a clinical photo of a ganglion cyst.
Illustration B shows a histologic specimen of a ganglion cyst
Incorrect Answers:
Answer 2: Proliferating histiocytes of moderate cellularity and frequent multinucleated giant
cells would be consistent with Giant-cell tumor of tendon sheath.
Answer 3: Polymorphonuclear neutrophils would be the primary cell type with infection and
are the predominant cells in pus, accounting for its whitish/yellowish appearance.
Answer 4: Spindle cells arranged in intersecting bundles would be consistent with a
Schwannoma.
Answer 5: Lipocytes, spindle cells, and scattered atypical giant cells are the histologic
findings of a pleomorphic lipoma.
Illustrations: A B

2. (OBQ09.67) A 10-year-old boy presents with a painless mass on the dorsal aspect of his wrist that
has been present for 3 weeks. A clinical image is shown in Figure A. T1 and T2 magnetic resonance
images are shown in Figure B and C, respectively. On your exam, the mass transilluminates and Allen
test reveals patent radial and ulnar arteries. What is the most appropriate next step in management?
FIGURES: A B C

1.

Referral to a orthopaedic oncologist

2% (26/1697)

2.

Surgical excision with wide margins

3% (56/1697)

3.

Observation

94% (1597/1697)

4.

Autologus bone marrow aspirate injection

0% (0/1697)

5.

Injection of N-Butyl-Cyanoacrylate

1% (12/1697)

PREFERRED RESPONSE 3
This child has a ganglion cyst on the dorsal aspect of his wrist. Imaging provided shows a wellmarginated, homogenous signal intensity mass consistent with a ganglion cyst. Physical examination
findings of a mass transilluminating corroborate the MRI findings of a ganglion cyst. Performing an
Allen's test to evaluate radial and ulnar artery collateral blood flow is especially important when
evaluating ganglion cysts on the volar aspect of the wrist as they are often adjacent to the radial artery.
Wang et al. peformed a Level 4 review of 14 children with hand and wrist ganglion cysts and found
that 79% of these cysts resolved spontaneously within 1 year. Autologus bone marrow aspirate
injection is a treatment option for unicameral bone cysts and N-Butyl-Cyanoacrylate injections have
been described for treatment of hemangiomas. Referral to an orthopaedic oncologist is not indicated.

Epidermal Inclusion Cyst


Author: Deborah Allen

Topic updated on 02/15/14 6:50pm

Introduction

A painless, benign, slow-growing soft tissue


tumor that often occurs in the hand
o occurs months to years after a
traumatic event
Epidemiology
o incidence
third most common hand tumor
o demographics
more common in men than
women
occurs in the third to fourth
decade
o location
the distal phalanx is commonly
involved
Pathophysiology
o results from a penetrating injury that drives keratinizing
epithelium into subcutaneous tissues or bone
o cells grow slowly to produce an epithelial cell-lined cyst filled
with keratin
Prognosis
o excision is curative
o malignant transformation has not been reported
Presentation
Symptoms
o painless mass, most commonly occurring in the fingertip
o although less common, erythematous, painful lesions have been
reported
Physical exam
o inspection & palpation
flesh-colored, yellow, or white in appearance

well-circumscribed, firm, slightly mobile lesions

lesions are firmer than ganglion cysts and do not


transilluminate
often superficial and tethered to overlying skin

range of motion
there may be loss of ROM when lesions are large and occur

near IP joints
neurovascular exam
sensory deficits may be evident with 2-point discrimination
testing secondary to digital nerve compression

Imaging
Radiographs
o recommended views
AP, lateral, and oblique views of the involved digit or hand
o findings
soft tissue mass may be evident
a lytic lesion of the distal phalanx may be present if the cyst
erodes into bone

may mimic a malignant or infectious process


Studies
Biopsy
o indications
should be considered before surgical excision to rule out
neoplasm or infection if a lytic bony lesion is present in the
distal phalanx
Histology
o gross appearance
cysts contain a thick, white keratinous material
o characteristic findings
cysts filled with keratin and lined with epithelial cells

Differential

Tophaceous gout
Foreign body granuloma
Sebaceous cyst
Giant cell tumor
Ganglion cyst
Enchondroma
Glomus tumor
Treatment
Nonoperative
o observation
indications
not recommended
Operative
o marginal excision
indications
diagnosis of epidermal inclusion cyst
painful lesions
loss of function
cosmetic concerns
technique
careful dissection to remove the entire capsule
local curettage and bone graft may be required for
lesions eroding bone
amputation is an alternative with advanced bony
destruction in rare circumstances
outcomes

marginal excision is curative


low recurrence rate
Complications
Wound complications
Infection
Digital neurapraxia
Recurrence
o recurrence rate is low even with bony involvement

Anomalous Extensor Tendon


Author: Colin Woon

Topic updated on 09/05/13 6:40pm

Introduction

Definition
o variations of extensor tendons of the hand
o usually discovered incidentally during surgery for other reasons
(e.g. ganglion excision)
Epidemiology
o incidence
not uncommon
Mechanism
o

symptoms arise because of increased muscle volume within


small muscle compartment
pain from synovitis or ischemia

Anatomy
Normal EIP
o occupies 4th dorsal extensor compartment (8-10mm wide)
o ratio of 1:1 for muscle:tendon length
o origin - posterior surface of distal third of ulna and adjacent interosseous
membrane
o insertion - dorsal expansion of index finger on ulnar side of EDC

Classification
Anomalous Extensor Muscle Forms
Anomalous extensor indicis
Most common cause of symptoms
proprius (aEIP)
Extensor digitorum brevis manus Less common cause of symptoms because
(EDBM)
muscle belly is proximal to extensor retinaculum
Extensor medii proprius (EMP)
Extensor indicis et medii
EIP and EIMC unlikely to be symptomatic
communis (EIMC)
because of narrow width

Presentation

Symptoms
o usually asymptomatic
discovered incidentally during surgery (e.g. ganglion
removal)
o mass on the dorsum of the hand

intermittent dorsal wrist pain if muscle bellies impinge on and


occupy the narrow dorsal compartments of the wrist
Physical Exam
o inspection
mass does not transilluminate
moves with movement of local muscles (flexion and
extension of hand and wrist(
becomes firmer with grasp
o provocative tests
resisted extension triggers pain
o

Imaging
MRI
o
o

indications
exclude other more common conditions e.g. ganglion
findings
mass is isointense with muscle tissue

anomalous extensor indicis proprius (aEIP)


extensor digitorum brevis manus (EDBM)

extensor medii proprius (EMP)


extensor indicis et medii communis (EIMC)
Differential
Ganglion
Synovitis
o both produce dorsal wrist pain
Treatment
Nonoperative
o observation
indications
first line treatment
Operative
o surgical decompression of 4th dorsal compartment (aEIP) or reduction of
muscle belly (EDBM)

indications
failed conservative treatment, and symptoms, signs and
imaging point to anomalous muscle, with no associated
conditions (e.g. ganglion)

Giant Cell Tumor of Tendon Sheath


Author: Joshua Blomberg

Topic updated on 09/14/13 3:08pm

Introduction
A benign nodular tumor that is found on the tendon sheath of the
hands and feet
Also known as pigmented villonodular tumor of the tendon sheath
(PVNTS)
Epidemiology
o present in 3rd-5th decade of life
o incidence
second most common soft-tissue tumor seen in the hand,
following ganglion cyst
o location
it is most common on palmar surface of radial three digits
near DIPJ
o no reports of metastisis in literature
Presentation
Symptoms
o enlarging mass
o pain, worse with activity (or wearing shoes, for foot lesions)
Physical exam
o firm, nodular mass that does not transilluminate
Differential diagnosis
o ganglion cyst
cystic component
o pigmented villonodular synovitis
histologically identical
involves larger joints
o desmoid tumor
o fibroma/fibrosarcoma
o glomangioma
Imaging
Radiographs

pressure-type bone erosion can be seen in up to 5% of patients on radiographs

Ultrasound
o able to demonstrate relationship of lesion with adjacent tendon
o
o

homogeneously hypoechoic, although some heterogeneity may be


seen in echo-texture in a minority of cases
most have some internal vascularity

MRI
o
o

MRI may be helpful diagnostically


appearance of the focal form is generally decreased signal intensity on both
T1-and T2-weighted MR imaging

Histology
Characterized by
o proliferating histiocytes, moderately cellular (sheets of rounded or polygonal
cells)

o
o

hemosiderin (brown color) may be present, but typically less than seen with
PVNS
multinucleated giant cells are common

Treatment
Operative

marginal excision
5-50% recurrence rate
more common if tumor extends into joints and deep to the volar
plate
local recurrence is usually treated with repeat excision
operative approach is dependant on location and extent of the tumor

Melanoma
Author: Colin Woon

Topic updated on 01/24/16 9:15pm

Introduction

An aggresive skin malignancy of melanocytic origin


o

types include
acral lentiginous melanoma
subungual melanoma is a subtype of ALM

Epidemiology
o demographics
slightly more common in men (male:female ratio = 1.2:1)
age bracket is 50-70yrs
o location
thumb > great toe > index finger
sun exposed areas
o risk factors

sun exposure
UV radiation suppresses skin immunity, induces
melanocyte cell division, produces free radicals,
damages melanocyte DNA
family history
skin characteristics
blue eyes, fair hair and complexion, freckling
xeroderma pigmentosa
familial atypical mole or melanoma (FAMM) syndrome

multiple benign and dysplastic nevi


dysplastic nevi are a precursor
immunesuppression
Pathophysiology
o progresses through phases of growth

Prognosis
o depth is the most important prognostic factor

< 0.7 mm - survival is 96%


> 4.0 mm - survival is 47%
poor prognostic factors for melanoma
deep lesion
male sex
lesion on neck or scalp
positive lymph nodes and metastases
ulceration
subungual melanoma has poor prognosis overall with 5yr
survival 40-60%

Anatomy
Melanocytes
o derived from neural crest cells
o found in deepest layer of epidermis, separated from dermis by
basement membrane
o dermis is divided into papillary dermis and reticular dermis
o subcutaneous tissue is deep to reticular dermis
Classification
Breslow classification
o thickness =< 0.75mm
o thickness 0.76 - 1.5mm
o thickness 1.51 - 4mm
o thickness >4mm
Clark classification
o Level I - involves epidermis (in situ melanoma), no invasion
o Level II - invades papillary dermis
o Level III - invades papillary dermis up to papillary-reticular interface
o Level IV - invades reticular dermis

Level V - invades subcutaneous tissue

Presentation
History
o pigmented lesion with recent change in shape or size
o

nail trauma
subungual melanoma renders the nail dystrophic and vulnerable
to trauma

Symptoms
o

itching or bleeding

characterized by (ABCDEs)
Asymmetry
Border irregularity
Color variation
Diameter (<6mm benign)
Elevation
Enlargement

Physical exam
o brown-black pigmented lesion, may ulcerate
o extension of brown-black pigment of the nail bed or nail plate to
the cuticle and nail folds (Hutchinson sign)

Imaging
CXR
indications
lungs are often first site of metastases
Ultrasound
o indications
diagnose lymph node involvement
PET or CT
o

indications
detect metastases

Studies
Labs
CBC
o AST and ALT
liver metastases
o LDH
predictive for poor prognosis
Histolology
o melanocytes with
marked cellular atypia
invasion into the dermis
vacuolated cytoplasm
hyperchromatic nuclei with prominant nucleoli
o

Differential
Differentials for melanoma
o nevi
o seborrheic keratosis
o basal cell carcinoma
Subungual melanoma is mistaken for
o trauma
o subungual hematoma
o onychomycosis
Treatment
Operative
o local resection with a 1cm margin
indications
lesion is < 1mm thick
o local resection with 1-2cm margin, sentinel node biopsy
indications
lesion is 1-2mm thick
technique
if sentinel node biopsy positive perform radical node
dissection
o local resection, lymph node dissection, chemotherapy
indications
evidence of metastasis
o amputation
indications
subungual melanoma
outcomes

distal amputation with sufficient margins has similar


recurrence rates and survival to proximal
(carpometacarpal/tarsometacarpal) amputations
may include lymph node dissection and isolated limb
perfusion
Prevention
o prevent melanoma with sunscreen and avoiding sun exposure
Complications
Recurrence
o usually regional lymph nodes

Hypothenar Hammer Syndrome


Author: Colin Woon

Topic updated on 01/22/16 11:15pm

Introduction

Post-traumatic digital ischemia


from thrombosis of ulnar
artery at Guyon's canal.
Epidemiology
o incidence
rare
o demographics
male: female ratio is 9:1
age bracket is 40s-50s
o location
unilateral, dominant ring
finger +/- small finger
less commonly, index and middle fingers
thumb is spared
o risk factors
occupations using vibrating tools such as carpenters,
machinists, mechanics
sports such as baseball catchers, mountain biking, golf,
volleyball, karate
Pathophysiology
o mechanism
single or repetitive blunt impact on hypothenar eminence
leads to ulnar artery thrombosis or aneurysm

hook of hamate functions as an anvil, causing thrombosis


distal embolisation leads to ulceration, gangrene
Anatomy
Ulnar artery
o ulnar artery branches into 2 branches as it exits Guyon's canal
deep branch
superficial palmar arch in Guyon's canal
o relation to hook of hamate
over distal 2cm, the artery is directly anterior to the hook of the
hamate, covered by palmaris brevis, subcutaneous tissue and skin

Presentation
History
o occupational or sporting risks (see above)
Symptoms
o pain over hypothenar eminence and ring finger
may involve small, middle and index fingers
o cold sensitivity
o paresthesia
Physical exam
o inspection
blanching, mottling, cyanosis, pallor, gangrene
tenderness over hypothenar eminence
prominent callus (calloused skin over hypothenar eminence)

pulsatile mass if aneurysm is present


fingertip ulcerations over ulnar digits
splinter hemorrhages over ulnar digits
provocative tests
Allen's test
positive if occlusion is present
negative if aneurysm is present

Imaging
Doppler ultrasound
o indications
first line test
Angiogram, CT angiogram or MR angiogram

o
o

indications
mandatory for diagnosis
findings
tortuous "corkscrew" ulnar artery
occlusion or aneurysm at the hook of the hamate

Differential
Raynaud's disease involves the thumb but hypothenar hammer syndrome does not

Treatment
Nonoperative
o lifestyle modifications, symptomatic treatment, and
vascular consult
indications
thrombosis without aneurysm > 2 weeks
asymptomatic
no threat of digital loss
lifestyle modifications
smoking cessation
avoid recurrent trauma
outcomes
80% success
Operative
o endovascular fibrinolysis
indications
thrombosis without aneurysm < 2 weeks
o excision of involved segment and reconstruction with or
without a vein graft

indications
thrombosis with aneurysm
ischemia in multiple digits
failed conservative treatment with recurrent
symptoms

Raynaud's Syndrome
Author: Deborah Allen

Topic updated on 08/16/13 11:06pm

Introduction

Raynaud's Syndrome consists of both


o Raynaud's Phenomenon
vasospastic disease with a
known cause
o Raynaud's Disease
vasospastic disease with
no known cause
(idiopathic)
Raynaud's Phenomenon
Vasospastic disease with a known underlying disease
o epidemiology
demographics
occasional female predominance
age >40 years (generally older than patients with
Raynaud's disease)
location
affects the distal aspect of digits
o pathophysiology
periodic digital ischemia induced by cold temperature or
sympathetic stimuli including pain or emotional stress
triphasic color change (white-blue-red progression)
digits turn white from vasospasm and interruption of
blood flow

blue discoloration follows from cyanosis and venous


stasis

finally digits turn red as a result of rebound hyperemia


dysesthesias often follow color changes
o associated conditions
connective tissue disease
scleroderma (80-90% incidence of Raynaud's
phenomenon)
SLE (18-26%)
dermatomyositis (30%)
RA (11%)
CREST syndrome
calcinosis, Raynaud's phenomenon, esophageal
dysmotility, sclerodactyly, telangiectasias
neurovascular compression (thoracic outlet syndrome)
Presentation
o symptoms
asymmetric findings

rapid progression
physical exam
peripheral pulses often absent
frequent trophic skin changes (including ulceration and
gangrene)

abnormal Allen test


Studies
o labs
o

blood chemistry - often abnormal


invasive studies
microangiology - often abnormal
angiography - often abnormal

Treatment

nonoperative
lifestyle modifications, treat underlying cause
indications
mainstay of treatment
modalities
smoking cessation and avoidance of cold
exposure is critical

Raynaud's Disease
Vasospastic disease with no known cause (idiopathic)
o epidemiology
seen in young premenopausal women (age <40 years)
o pathophysiology
similar to Raynaud's phenomenon
Presentation
o symptoms
often bilateral
slow progression
o physical exam
peripheral pulses usually present
trophic skin changes are uncommon
normal Allen test
Studies
o labs usually normal
o invasive studies usually normal
o diagnosis
based on Allen and Brown criteria
Allen and Brown Criteria for Raynaud's Disease
Intermittent attacks with discoloration of acral
parts
Bilateral involvement
Absence of clinical arterial occlusion
Gangrene and trophic changes are rare
Symptoms present for >2 years
Absence of other disease to explain findings
Predominance in women

Treatment
o nonoperative
medical management
indications
first line of treatment
modalities
smoking cessation and avoidance of cold

exposure is critical
thermal biofeedback techniques
medications include
calcium channel blockers
ASA
intra-arterial reserpine
dipyridamole (Persantine)
pentoxifylline (Trental)
o

operative
digital sympathectomy
indications
severe cases that fail conservative treatment
microvascular reconstruction
indications
may be indicated in rare situations

Thromboangiitis Obliterans (Buerger's


disease)
Author: Mark Karadsheh

Topic updated on 02/16/14 7:55am

Introduction

A nonatherosclerotic, segmental, inflammatory disease in the small and medium-sized


vessels of the hands and feet
o occurs predominantly in smokers
Epidemiology
o incidence
12.6 per 100,000 in the United States

demographics
3:1 male: female ratio
typically affects patients < 45 years old
o risk factors
smoking
chewing tobacco
Pathophysiology
o inflammation and clotting of the small vessels of hands and feet
o 3 phases
acute
o

thrombus including neutrophils and giant cells occludes the


vessel lumen while sparing the wall
subacute

progressive organization of the thrombus


chronic

inflammation has subsided


organized thrombus and vascular fibrosis remain
Prognosis
o depends on smoking status
94% who quit smoking avoid amputation
43% chance of amputation within 8 years if smoking is continued
Presentation
Symptoms
o early disease
intermittent claudication of feet, legs, hands or arms
numbness and/or tingling in the limbs
o late disease
symptoms of critical limb ischemia
rest pain
Physical exam
o inspection
ulcerations

large, erythematous, superficial blood vessels


necrotic distal digits in hands and feet

o
o

palpation
decreased temperature in hands and feet
neurovascular
diminished or absent pulses
sensory findings in up to 70% of patients
provocative tests
positive Allen test in young smoker with digital ischemia is suggestive
of disease

Imaging
Arteriography
o indications
useful for ruling-out other conditions that may mimic Buerger's disease
o findings
"corkscrew" vessels

collateral circulation giving a "spider leg" appearance


Studies
Labs
o used to exclude alternative diagnoses
Echocardiogram
o used to exclude proximal source of emboli

Treatment

Nonoperative
o smoking cessation and symptomatic treatment
indications
all patients with Buerger's disease that use tobacco
techniques
smoking cessation
patient education
pharmacotherapy
smoking cessation groups
symptomatic treatment
avoid exposure to cold
gentle exercise
daily aspirin
vasodilators
outcomes
smoking cessation is the only treatment known to decrease the
risk of future amputation
Operative
o surgical sympathectomy
indications (controversial)
refractory pain and digital ischemia
technique
cut nerves to the affected areas
o amputation
indications
gangrene
non-healing ulcers
refractory pain

Questions

(OBQ12.126) A 45-year-old male smoker presents with the clinical appearance shown in Figure A.
Which of the following statements is true regarding his condition?
FIGURES: A

1.

Anticoagulation with aspirin has been shown to decrease the incidence of amputation

30% (836/2775)

2.

Arteriography is useful in the diagnosis of his condition

42% (1161/2775)

3.

Prophylactic amputation of unaffected digits leads to improved patient outcomes

1% (37/2775)

4.

The condition is usually painless

23% (632/2775)

5.

Involves proximal vessels first and distal vessels last

3% (84/2775)

PREFERRED RESPONSE 2
The clinical description and photograph are most consistent with a diagnosis of Buerger's disease, or
thromboangiitis obliterans. Arteriography is the best method for diagnosis of this condition.
Buerger's disease is an inflammatory occlusive disorder of small and medium-sized vessels of the
digits most frequently occurring in male smokers. The condition may mimic other autoimmune and
vascular diseases. If the diagnosis is uncertain, arteriogram is the study of choice. The only treatment

that has been found to reduce the risk of amputation is the cessation of smoking.
Phillips et al. review vascular conditions of the upper extremity. They discuss that patients with
Buerger's disease present with rest pain, claudication, and ulceration, and that cessation of smoking
decreases disease progression and the incidence of amputation.
Figure A shows the classic clinical appearance of a patient with Buerger's disease. Illustration A
shows an arteriogram with "corkscrew" arteries that result from vascular destruction. In the
appropriate clinical setting, this finding is diagnostic of Buerger's disease.
Incorrect Answers:
Answer 1: Anticoagulation has not been shown to halt progression of Buerger's disease.
Answer 3: Prophylactic amputation of uninvolved digits has not been described for the treatment of
Buerger's disease.
Answer 4: Buerger's disease is frequently painful.
Answer 5: Buerger's disease involves distal vessels first and proximal vessels last.
Illustrations: A

Digital Artery Aneurysm


Author: Evan Watts

Topic updated on 03/14/16 5:42am

Introduction

Aneurysm is defined as a permanent dilation of an artery with a 50%


increase in its normal diameter
Incidence
o rare
Pathophysiology
o traumatic
true aneurysm
blunt trauma weakens the arterial wall causing it to
dilate
appear more uniform in shape
false aneurysm (e.g. pseudoaneurysm)
pentrating trauma to arterial wall replaced by organized
hematoma and fibrous wall
appear more 'sac-like' in shape
o non-traumatic
inflammatory
atherosclerotic
Presentation
History
o recent blunt or penetrating hand trauma
Symptoms
o slow-growing painful mass

many be sensory disturbance due to compression of adjacent digital


nerve
Physican exam
o palpable mass
o

o
o

may be pulsatile in ~ 50% of cases


may occur in any of the 5 digits
most common in thumb > index > ring finger

Imaging
Radiographs
o indication
usually not helpful
concern of destructive lesion
o findings
usually normal
Doppler ultrasound or angiocomputed tomography (CT) scan

o
o

indication
pre-operative confirmation
findings

size and location of lesion


thrombus formation
collateral circulation
Differential

Often misdiagnosed as
o epidermoid cysts
o arteriovenous fistulas
o forieign body granulomas
o ganglions
o neurilemmomas

Treatment
Nonoperative
o observation and analgesics
indications
small, asymptomatic lesions
o ultrasound-guided thrombin injection
indications
some reports use this techique in lesions arising more
proximal in the hand or wrist.
Operative
o surgical exploration and ligation

indications
symptomatic lesions with adequate collateral circulation
technique
ligation performed proximal and distal
repair with interpositional grafting

indications
symptomatic lesions with inadeaquate collateral
circulation
Techniques
Digital artery aneurysm repair
o

end-to-end anastomosis and an autogenous interpositional vein or


arterial graft

Complications

digital ischemia
chronic pain

Question

(OBQ06.208) A 47-year-old female presents with a pulsatile mass in the palm of the hand
and intermittent paresthesias a few weeks after a traumatic laceration in a kitchen accident. A
contrast-enhanced MRA of the mass is seen in Figure A. What is the most appropriate
treatment?
FIGURES: A

1.

Aspiration

1% (4/423)

2.

Surgical exploration

78% (330/423)

3.

Compression bandage

14% (58/423)

4.

Steroid injection

1% (4/423)

5.

Strict elevation and observation

6% (25/423)

PREFERRED RESPONSE 2
The contrast-enhanced MRA shown in Figure A depicts a false aneurysm of the superficial
palmar arch. The most appropriate treatment for this symptomatic lesion is surgical
exploration.
Traumatic palmar artery aneurysms and pseudoaneurysms are rare and can be caused by
penetrating or blunt trauma (e.g., during endoscopic carpal tunnel release). The case reports
in the literature all support surgical exploration with either ligation, excision, or repair
depending on the extent of the lesion.
Yajima et al. report on the management of three cases of digital artery aneurysms after
traumatic lacerations. Two cases were treated with excision only, while one was treated with
excision and vascular reanastomosis.
Figure A is a contrast-enhanced MRA showing a false aneurysm of the superficial palmar
arch.
Incorrect Answers:
Answer 1, 3-5: Aspiration, compression bandage, steroid injection and observation are not
the most appropriate modalities of treatment for false aneurysms of the superficial palmar
arch.

Vous aimerez peut-être aussi